X



トップページ数学
1002コメント403KB
分からない問題はここに書いてね456
レス数が1000を超えています。これ以上書き込みはできません。
0002132人目の素数さん
垢版 |
2019/09/07(土) 23:31:40.27ID:0LV/IqJq
ここは分からない問題を書くスレです。
お願いごとをするスレでも分からない問題に答えてもらえるスレでもありません。
0004132人目の素数さん
垢版 |
2019/09/08(日) 16:36:41.65ID:CWD01bYT
xx+yy=1…@
xx-8x+yy+12=0…A
の共通接線Lを求めよという問題で
各円の中心を出してから、
L:2ax+2by+c=0…B 
とおいて点と直線の距離の公式を使えば解けるのはわかるのですが、式変形でやってみようと思い

@−Bで整理して、
(x-a)^2+(y-b)^2=aa+bb+c+1=D、これが唯一つ(x,y)の解を持てばいいから、D=0かつ、点(a,b)がLの上にあればよい、

同様にA-Bで、
(x-a-4)^2+(y-b)^2=c-12+(a+4)^2+b^2=E、これが唯一つ(x,y)の解を持てばいいから、E=0かつ、点(a+4,b)がLの上にあればよい、

となって、y=bでLが2通りのx座標を取るので、Lは傾き無限のy軸に並行な直線、となってしまったのですが、図を書けばこれは誤りです

とんでもないアホすぎミスをしてると思うのですが私の実力ではどこでミスしたのか分からないのでここの達人方お願いいたします
0005132人目の素数さん
垢版 |
2019/09/08(日) 17:00:52.24ID:snRYW362
>>4
D=0とかE=0とかしてるところがおかしいです

x^2+y^2=1…@
2x+2y=2√2…A

この連立方程式考えてみましょう

グラフ書けばこれらは接していますね

@-Aすると
(x-1)^2+(y-1)^2=3-2√2
あなたは勝手に右辺0にしてますよね

@とA両方満たすのがひとつしかないわけですよね
@-AとA両方満たすのは一つしかないと言えても、@-A満たすのが一つとは限りませんね
0006132人目の素数さん
垢版 |
2019/09/08(日) 18:26:59.32ID:NPxrtGxy
 xx + yy = 1     … @
 (x-4)^2 + yy = 4  … A

@の接線で傾きがmのもの
 y = mx ±√(1+mm),
Aの接線で傾きがmのもの
 y = m(x-4) ±2√(1+mm),
y切片が一致することから
 ±√(1+mm) = -4m ±2√(1+mm),
これより
 m/√(1+mm) = -3/4, -1/4, 1/4, 3/4,
 m = -3/√7, -1/√15, 1/√15, 3/√7,

 y = ±(x+4)/√15,
 y = ±(3x-4)/√7,
0007132人目の素数さん
垢版 |
2019/09/08(日) 19:26:42.67ID:53eQNOXe
全校生徒x人のうち24%がバスを使って学校に通っています
バスを使わず通学している生徒は何人ですか?
0008132人目の素数さん
垢版 |
2019/09/08(日) 19:38:21.04ID:NPxrtGxy
中心(0,0)で半径1  … @
中心(4,0)で半径2  … A
直線L:  y = mx+k,   … B
(x,y)〜L の距離  |y-(mx+k)|/√(1+mm),

(0,0)〜L の距離が1
 |k|/√(1+mm) = 1,
(4,0)〜L の距離が2
 |4m+k|/√(1+mm) = 2,
これより
 k = -(4/3)m, 4m,
 m = ±3/√7, ±1/√15,

 y = ±(x+4)/√15,
 y = ±(3x-4)/√7,

>>7
 (1-0.24)x 人
0009132人目の素数さん
垢版 |
2019/09/08(日) 22:02:29.73ID:n06H5EG5
平面状に非可算無限個のT字型の集合(大きさや縦横比や向きはそれぞれバラバラでもよい)を配置した時、重ならないように置く事は不可能である事を示せ
というのはどうやって解くのでしょうか


T字型の集合とはR^2の部分集合で
あるa,b>0を用いて
{(x,0)| -a<x<a}∪ {(0,y)| -b<y<0}
という集合の回転と移動で得られるものであると定義されます
0010 【大吉】
垢版 |
2019/09/09(月) 04:11:40.15ID:pmEy2ZnY
>>7
1-0.24=0.76
∴バスを使わずに通学する人は0.76x人
0013132人目の素数さん
垢版 |
2019/09/09(月) 11:38:26.77ID:GeJx+nRu
  告 示
このスレが正統の「分かスレ456」です。
[前スレ.960] にて 2019/09/07(土) 23:29:22.15 認知
0014132人目の素数さん
垢版 |
2019/09/09(月) 15:20:33.07ID:n4l/eJIN
質問です

A=Σ(k=0→n) (nCk)^2
B=Σ(k=0→n) (nCk)^3

それぞれどうやれば求まりますか?
Aだけでもお願いします

なるべく高校程度の範囲でお願いします
0015132人目の素数さん
垢版 |
2019/09/09(月) 17:19:58.45ID:pr21JHmw
数検の問題集を読んでいます。

1 / cos(x) の x = 0 でのテイラー展開を x^8 の項まで計算せよ

という問題の解答ですが、 1 / cos(x) は偶関数だから、

1 / cos(x) = a_0 + a_1*x^2 + a_2*x^4 + a_3*x^6 + a_4*x^8 + …

とおき、

cos(x) = 1 - (1/2)*x^2 + (1/24)*x^4 - (1/720)*x^6 + (1/40320)*x^8 + …

との積が 1 になるように漸化式を作ると、

1/ cos(x) = 1 + (1/2)*x^2 + 5/24*x^4 + (61/720)*x^6 + (277/8064)*x^8 + …
0016132人目の素数さん
垢版 |
2019/09/09(月) 17:20:16.84ID:pr21JHmw
Σ a_n
Σ b_n

がともに絶対収束ならば、

コーシー積 Σ c_n

も絶対収束して

Σ c_n = Σ a_n * Σ b_n



cos(x) = 1 - (1/2)*x^2 + (1/24)*x^4 - (1/720)*x^6 + (1/40320)*x^8 + …

は (-∞, +∞) で絶対収束する。

1 / cos(x) = a_0 + a_1*x^2 + a_2*x^4 + a_3*x^6 + a_4*x^8 + …

はその収束半径を R とすると、 (-R, R) で絶対収束する。

↑の命題により、

x ∈ (-R, R) のとき、

a_0 + ((-1/2)*a_0 + a_1) * x^2 + …

は絶対収束して、

a_0 + ((-1/2)*a_0 + a_1) * x^2 + … = cos(x) * (1/cos(x)) = 1

が成り立つ。

x = 0 を代入すると、

a_0 = 1

両辺を2回微分すると、

2! * ((-1/2)*a_0 + a_1) + … = 0

x = 0 を代入すると、

a_1 = 1/2



というように、 a_0, a_1, … を決定できる。

なので、そもそも、

1 / cos(x)

がテイラー展開できるのか?

ということに答えなければならないはずです。
0017132人目の素数さん
垢版 |
2019/09/09(月) 22:16:35.73ID:cY7hnB/s
>>14
>A=Σ(k=0→n) (nCk)^2
A=ΣnCk・nC(n-k)=2nCn
0018132人目の素数さん
垢版 |
2019/09/09(月) 23:15:25.31ID:cY7hnB/s
>>14
>B=Σ(k=0→n) (nCk)^3
No good answer
0020132人目の素数さん
垢版 |
2019/09/10(火) 00:44:19.43ID:yUERuKgH
>>14
B(n) = Σ(k=0→n) (nCk)^3
   = Σ(k=0→n) {(n+r)C(k+r)・(k+r)Cr}^3 / {(n+r)Cr}^3, (r≧0)

B(n) = Franel number

〔漸化式〕 (J.Franel)
 (n+1)(n+1)・B(n+1) = {7n(n+1)+2}B(n) + 8nn・B(n-1),

〔生成函数〕 (P.D.Hanna)
Σ(n=0→∞) B(n)(x^n)/(n!)^3 = { Σ(k=0→∞) (x^k)/(k!)^3 }^2.

〔公式〕 (V.Strehl)
 B(n) = Σ(n/2≦k≦n) (nCk)^2・(2k)Cn,

http://oeis.org/A000172
0021132人目の素数さん
垢版 |
2019/09/10(火) 01:00:43.92ID:lO8Z+bdf
コインを表n枚と裏n枚で合わせる組み合わせは何通りか考える

そのまま考えると
2nCn通りだろう

これを別の数え方で数えあげる


始めのn枚を並べた時に表が何枚あるかで場合分けして数えると

最初のn枚を並べたときk枚あるパターンはnCk通りで

この時残りのn枚は表のコインがn-k枚しか残っていないから
並べ方の組み合わせはnC(n-k)通り

合わせると最初のn枚に表がk枚あるような2n枚の並べ方の組み合わせは
nC(n-k) ・nC(n-k)


これを全部合わせると全パターンの並べ方の総数になるわけで、つまり2nCn
に等しいわけだ
0022132人目の素数さん
垢版 |
2019/09/10(火) 01:05:35.05ID:lO8Z+bdf
>>21
1行目は
合わせて2n枚並べる組み合わせ、だった
0023132人目の素数さん
垢版 |
2019/09/10(火) 01:34:38.92ID:yUERuKgH
>>20 (補足)

B(n) = Σ(k=0→[n/2]) (n+k)!/{(k!)^3・(n-2k)!} ・ 2^(n-2k)

〔生成函数〕
y(x) = Σ(n=0→∞) B(n)・x^n = 1 + 2x + 10xx + 56x^3 + 346x^4 + 2252x^5 +
は2階線形微分方程式
 x(x+1)(8x-1)y " + (24xx+14x-1)y ' + 2(4x+1)y = 0,
の解。
 G.Coserea(2018)
0024132人目の素数さん
垢版 |
2019/09/10(火) 07:32:24.82ID:KQ2SjNf6
>>19
(1+t)^(m+n)=Σ(m+n)Ck・t^k
(1+t)^m(1+t)^n=ΣmCi・t^iΣnCj・t^j=ΣΣmCi・nCj・t^(i+j)=Σ(Σ[i+j=k]mCi・nCj)t^k
Σ[i+j=k]mCi・nCj=(m+n)Ck

Σ[h+i+j=k]lCh・mCi・nCj=(l+m+n)Ck

Σ[Σi_j=k]Πm_jCi_j=(Σm_j)Ck
0026132人目の素数さん
垢版 |
2019/09/10(火) 12:50:01.18ID:AIUcxeUm
Michael Spivak著『Calculus』を読んでいます。

lim_{n → ∞} n * cos(n^2 * x) does not always exist(for example, it does not exist if x = 0).

などと書いてあります。

lim_{n → ∞} n * cos(n^2 * x) が収束するような x って存在するんですか?
0028132人目の素数さん
垢版 |
2019/09/10(火) 13:25:00.34ID:AIUcxeUm
>>27

証明をお願いします。
0030おいら
垢版 |
2019/09/10(火) 15:13:37.90ID:yUERuKgH
1/cos(x) = Σ[k=0,∞) (-1)^k・{E_(2k)/(2k)!}・x^(2k),

と展開される。 E_(2k) はオイラー数。
0031132人目の素数さん
垢版 |
2019/09/10(火) 20:30:52.41ID:AIUcxeUm
Michael Spivak著『Calculus』を読んでいます。
以下が成り立つことの証明を読みました。
非常に重要かつ興味深い結果だと思いました。

ところが、微分積分学の教科書でこのことが書いてある本はほとんどないように思います。

藤原松三郎以外の本で、このことが書いてある本を教えてください。


f(x) = Σ_{n = 0}^{∞} a_n * x^n
a_0 ≠ 0
Σ_{n = 0}^{∞} a_n * (x_0)^n ∈ R for some x_0 ∈ R - {0}

とする。

このとき、正の収束半径を持ったべき級数 g(x) = Σ_{n = 0}^{∞} b_n * x^n

で、

f(x) * g(x) = 1 for all x ∈ (-R, R) for some R > 0

が成り立つようなものが存在することを証明せよ。
0032132人目の素数さん
垢版 |
2019/09/10(火) 21:40:56.73ID:lO8Z+bdf
>>31
Robert Strichartzの
The Way of Analysis

とかにもある
0033132人目の素数さん
垢版 |
2019/09/10(火) 22:37:54.04ID:AIUcxeUm
>>32

ありがとうございます。

Closure Properties of Analytic Functionsというセクションにある定理ですね。

ぱっと見、分かりやすくていい本っぽいですね。

ところで、それほどメジャーな本ではないと思いますが、この本を読んだきっかけは何ですか?
0034132人目の素数さん
垢版 |
2019/09/10(火) 22:45:35.32ID:AIUcxeUm
>>32

探せばいい本はたくさんあるようですね。

>>32

の本もパソコンにあったのですが、全く見たことがありませんでした。
0035おいら
垢版 |
2019/09/10(火) 23:40:20.82ID:yUERuKgH
>>30 から

1 = cos(x) {1/cos(x)}
 = {Σ(j=0→∞) (-1)^j・{1/(2j)!} x^(2j)} {Σ(k=0→∞) |E_(2k)|/(2k)! x^(2k)},
x^(2n) の係数から
 Σ(k=0→n) (2n)C(2k)・E_(2k) = 0,

〔漸化式〕
E_(2n) = - Σ(k=0→n-1) (2n)C(2k)・E_(2k)
と E_0 =1 により整数。

〔漸近形〕
 E_(2k) / (2k)! 〜 (-1)^k・(2/π)^(2k+1)  (k→∞)
0036おいら
垢版 |
2019/09/10(火) 23:57:47.86ID:yUERuKgH
>>35
1 = cos(x) {1/cos(x)}
 = {Σ(j=0→∞) (-1)^j・{1/(2j)!} x^(2j)} {Σ(k=0→∞) |E_(2k)|/(2k)! x^(2k)}
 = Σ(n=0→∞) {Σ(k=0→n) (2n)C(2k)・E_(2k)}/(2n)! x^(2n)
から
 Σ(k=0→n) (2n)C(2k)・E_(2k) = δ_(n,0)
0037おいら
垢版 |
2019/09/11(水) 10:14:47.61ID:1IJ7FHVd
>>36
E_0 = 1,        (1.27323954)
E_2 = -1,       (-1.0320491)
E_4 = 5,        (5.019285)
E_6 = -61,       (-61.02719)
E_8 = 1385,      (1385.0697)
E_10 = -50521,     (-50521.284)
E_12 = 2702765,     (2702766.69)
E_14 = -199360981,   (-199360994.9)
E_16 = 19391512145,   (19391512295)
E_18 = -2404879675441, (-2404879677510)
E_20 = 370371188237525, (370371188272932)
……
( )内は (-1)^(2k)・2(2k)!・(2/π)^(2k+1)
0038132人目の素数さん
垢版 |
2019/09/13(金) 00:19:16.41ID:Bk4U8Cej
[脇スレ.026]

(1) (√2 + √3)^n は負でない整数 a_n, b_n, c_n, d_n を用いて
 a_n + (√2)b_n + (√3)c_n + (√6)d_n,
と表せることを示せ。

(2) d_(2n-1) を求めよ。
0039132人目の素数さん
垢版 |
2019/09/13(金) 00:22:29.77ID:Bk4U8Cej
>>38
(略解)
漸化式
 a_(n+1) = 2b_n + 3c_n,
 b_(n+1) = a_n + 3d_n,
 c_(n+1) = a_n + 2d_n,
 d_(n+1) = b_n + c_n,

 a_1=0, b_1=1, c_1=1, d_1=0
から。
(2)
 a_(2n-1) = d_(2n-1) = 0,
 b_(2n) = c_(2n) = 0,
後者は
 (√2 + √3)^(2n) = (5+2√6)^n
 = Σ(k=0,n) C(n,k) (5^k)(2√6)^(n-k)
 = a_(2n) + (√6)d_(2n),
0040132人目の素数さん
垢版 |
2019/09/13(金) 00:28:38.43ID:Bk4U8Cej
[脇スレ.044]
Aは実数を成分とする2次正方行列で A≠E、A≠O とする。
このとき、Aによる一次変換αの不動点
 ( x ) ≠ ( 0 ) を求めよ。
 ( y )  ( 0 )
0041132人目の素数さん
垢版 |
2019/09/13(金) 00:34:52.04ID:Bk4U8Cej
>>40
(略解)
A = (a b)
  (c d)
A-E ≠ O,
|A-E| = (a-1)(d-1) - bc = |A| - (a+d) +1 = 0,
とするとき、不動点は
 ( x ) = ( b ), (1-d)
 ( y )  (1-a), ( c )
0042132人目の素数さん
垢版 |
2019/09/13(金) 00:49:24.44ID:Bk4U8Cej
[脇スレ.082]
実数を成分とする2次正方行列Aは、
 A = [a,b]
   [c,d]
 ad-bc = 1
 3以上のある自然数nに対して A^n = E,
を満たす。Aを求めよ。
0043132人目の素数さん
垢版 |
2019/09/13(金) 01:10:35.06ID:Bk4U8Cej
>>42
(略解)
 A = ±E
pはnの素因数として
 A = [ cos(2qπ/p), ±sin(2qπ/p) ]
   [ 干sin(2qπ/p), cos(2qπ/p)]

>>41 不動軸と云うべきか....
0045132人目の素数さん
垢版 |
2019/09/13(金) 01:55:03.44ID:Bk4U8Cej
[脇スレ.083]
分からない問題
(1) スターリングの公式の導出

 log(k) ≧ ∫[k-1/2,k+1/2] log(x)dx,
 (1/2){log(k-1)+log(k)} ≦ ∫[k-1,k] log(x)dx,
を k=2,・・・・,n について足すと
 log(n!) = Σ[k=2,n] log(k) 〜 (n+1/2)log(n) -n +c,
  0.89180 = (3/2)log(2e/3) < c < 1,
は出るが、
 c = (1/2)log(2π) = 0.91894
が出ませぬ....orz  どうやるんだろ??
 
*) log(n+1/2) = log(n) - log{1 - 1/(2n+1)} > log(n) + 1/(2n+1),
0046132人目の素数さん
垢版 |
2019/09/13(金) 02:05:25.03ID:Bk4U8Cej
[脇スレ.083]
分からない問題
(2) オイラー定数γを規則的な連分数で表示する。

代数的数どころか無理数かどうかも怪しい。
もう、サパーリですよね。
0047132人目の素数さん
垢版 |
2019/09/13(金) 02:32:45.96ID:Bk4U8Cej
[脇スレ.083]
分からない問題
(3)  円周率πを計算するニュートンの公式の導出

(略解)
二項公式
 (1-xx)^n = Σ(k=0,n) (-1)^k C(n,k) x^(2k),
で n=-1/2 のとき、
 1/√(1-xx) = Σ(k=0,∞) (-1)^k C(-1/2,k) x^(2k),
xで積分して
 arcsin(x) = Σ(k=0,∞) (-1)^k {C(-1/2,k)/(2k+1)} x^(2k+1),
x=1/2 のとき
 π/6 = Σ(k=0,∞) (-1)^k {C(-1/2,k)/(2k+1)} (1/2)^(2k+1),

ここに、一般化二項係数
 C(-1/2,k) = (-1)^k {(2k-1)!!/(2^k・k!)},  (k≧1)
       = 1     (k=0)
0048132人目の素数さん
垢版 |
2019/09/13(金) 02:54:43.43ID:Bk4U8Cej
>>46
 サパーリでもないか。

ネイピア数eにはある。
 e = 2 + [1, 2, 1, 1, 4, 1, 1, 6, 1, 1, 8, 1, 1, 10, ...]

円周率πにもある。(正則ぢゃないけど)
 π = 3 + (1^2)/{6 + (3^2)/[6 + (5^2)/(6 +(7^2)/(6 + ・・・・))]}
0049132人目の素数さん
垢版 |
2019/09/13(金) 03:01:40.15ID:Bk4U8Cej
>>42

A = [cosθ+ p sinθ, -q(1+pp)sinθ]
   [(1/q)sinθ, cosθ- p sinθ]
p,q≠0:実数, θ=2πk/n, k:整数
らしい。。。
[脇スレ.090]
0050132人目の素数さん
垢版 |
2019/09/13(金) 03:31:46.20ID:Bk4U8Cej
>>49
n = 3 の場合で
p = 18q = ±3√3,
1+pp = 28,
k = ±1,
θ = ±2π/3,
cosθ = -1/2,
sinθ = 3q = 1/(4q) = 9/(2p),
とおけば >>44 を含む。
0051132人目の素数さん
垢版 |
2019/09/13(金) 03:42:56.23ID:ELyka2CP
いや、>>49も不十分。
ジョルダンセルのサイズが1で固有値がともに1の冪根でtrace、determinantが実数が条件だから固有値が+1,-1となるケース、つまりtr=0、det=-1のケースも入れないといかん。
0052132人目の素数さん
垢版 |
2019/09/13(金) 03:51:18.13ID:BXZbPjlz
>>51
>det=-1のケース
ad-bc=1
0054132人目の素数さん
垢版 |
2019/09/13(金) 12:15:04.50ID:Bk4U8Cej
>>49
固有値λ = e^(iθ) に対する固有ヴェクトルu
 ( p+i )
 ( 1/q )

固有値λ = e^(-iθ) に対する固有ヴェクトルu
 ( p-i )
 ( 1/q )

[脇スレ.090]
0055132人目の素数さん
垢版 |
2019/09/13(金) 12:26:59.54ID:Bk4U8Cej
>>35
〔漸近形〕
 E_(2k) / (2k)! 〜 (-1)^k 2(2/π)^(2k+1)  (k→∞)

>>37
 ( )内は (-1)^k 2(2k)!・(2/π)^(2k+1)
0056132人目の素数さん
垢版 |
2019/09/14(土) 07:20:12.67ID:MvteciNZ
>>45
n! = Γ(n+1) = ∫[0,∞] x^n e^(-x) dx (置換: x=n+(√n)t)
 = ∫[-n^2,∞] (n+(√n)t)^n e^(-n-(√n)t) (√n)dt
 = n^(n+1/2) e^(-n) ∫[-n^2,∞] (1+t/(√n))^n e^(-(√n)t) dt

n→∞ で (1+t/(√n))^n e^(-(√n)t) → e^(-t^2/2),
∫[-n^2,∞] (1+t/(√n))^n e^(-(√n)t) dt → ∫[-∞,∞] e^(-t^2/2) dt = √(2π)
0057132人目の素数さん
垢版 |
2019/09/14(土) 15:15:04.46ID:cBoFKc15
被積分函数 x^n e^(-x) は x=n で極大 (n/e)^n となる
x=n のまわりにテーラー展開して
 x^n e^(-x) = (n/e)^n {1 - tt/2 + ttt/(3√n) + (n-2)t^4 /(8n) + (6-5n)t^5 /(30n√n) + ・・・・ }
  = (n/e)^n e^(-tt/2 + (t^3 /√n) -(t^4)/(4n) + (t^5)/(5n√n) - ・・・・ )
  〜 (n/e)^n e^(-tt/2)    ・・・・ 正規分布
これは t=±√2 の辺りでも極大値の 1/e であり、t<-√n ではひじょうに小さい。
∴積分の下限を -∞ としてもよい。
n! = ∫[0,∞] x^n e^(-x) dx
 〜 (n/e)^n ∫[-√n, ∞] e^(-tt/2) (√n)dt
 〜 n^(n+1/2) e^(-n) ∫[-∞, ∞] e^(-tt/2) dt
 = n^(n+1/2) e^(-n) √(2π),
でござるな。
0058132人目の素数さん
垢版 |
2019/09/14(土) 19:12:21.54ID:0nc5ufbc
https://youtu.be/89d5f8WUf1Y

↑は、

lim_{n → ∞} (n!/n^n)^(1/n)

の値を求めている動画です。

40万回以上も再生されている動画ですが、いい加減すぎやしないでしょうか?

こういう解答を何の疑いもなく受け入れてしまう人もいると思います。

質が悪いですよね。
0059132人目の素数さん
垢版 |
2019/09/14(土) 19:50:54.73ID:cBoFKc15
〔問題102〕
f(x,t) = exp(-xx) cos(t),
g(x,t) = exp(-xx) sin(t),
F(x,t) = [[f, -g] [g, f]],
D = [[∂/∂x, -(∂/∂t)] [∂/∂t, ∂/∂x]],  … 微分演算子
X(x,t) = {F(x,t)^(-1)} D F(x,t),
とおく。
自然数nに対して X^n をx,tで表わせ。

[脇スレ.102]
0060132人目の素数さん
垢版 |
2019/09/14(土) 20:08:49.38ID:cBoFKc15
T(t) = [cos(t), -sin(t)] [sin(t), cos(t)]],
とおくと
F(x,t) = exp(-xx) T(t),
F^(-1) = exp(xx) T(-t),
D T(t) = T(t) (∂x - 1),
より
X^n = {F^(-1) D F}^n
 = F^(-1) D^n F
 = F^(-1) (∂x - 1)^n exp(-xx) T(t)
 = exp(xx) (∂x - 1)^n exp(-xx) E,
さて、どうする??
0062132人目の素数さん
垢版 |
2019/09/14(土) 20:51:09.08ID:TM+svlHx
>>58
字で書いて
0063132人目の素数さん
垢版 |
2019/09/14(土) 21:31:20.43ID:bw1hEwcz
>>58
なんか間違ってるのかこれ
0064132人目の素数さん
垢版 |
2019/09/14(土) 22:05:44.10ID:TrVaXrFL
たぶん「極限が存在するかどうか示してないのにlogの連続性使って計算してる」とかそんな感じよくだらない部分だと思う
0066132人目の素数さん
垢版 |
2019/09/14(土) 22:06:48.59ID:0nc5ufbc
>>63
リーマン和のようなものを作っていて、それが広義積分に収束するということを勝手に仮定しています。
0067132人目の素数さん
垢版 |
2019/09/14(土) 22:08:15.15ID:0nc5ufbc
自分で考えた解答は以下です:

a_n := log((n!/n^n)^(1/n))

=

(1/n) * log(n!/n^n) = (1/n) * log(1/n * 2/n * … * n/n)

=

(1/n) * (log(1/n) + log(2/n) + … + log(n/n))

とおく。

log(x) は単調増加関数だから、

(1/n) * (log(1/n) + log(2/n) + … + log((n-1)/n)) ≦ ∫_{1/n}^{1} log(x) dx ≦ (1/n) * (log(2/n) + … + log(n/n))

が成り立つ。

よって、

(1/n) * (log(1/n) + log(2/n) + … + log(n/n)) ≦ ∫_{1/n}^{1} log(x) dx + (1/n) * log(n/n) = ∫_{1/n}^{1} log(x) dx

∫_{1/n}^{1} log(x) dx + (1/n) * log(1/n) ≦ (1/n) * (log(1/n) + log(2/n) + … + log(n/n))

が成り立つ。

まとめると、

∫_{1/n}^{1} log(x) dx + (1/n) * log(1/n) ≦ a_n ≦ ∫_{1/n}^{1} log(x) dx

が成り立つ。
0068132人目の素数さん
垢版 |
2019/09/14(土) 22:08:31.86ID:0nc5ufbc
∫ log(x) dx = x * log(x) - x

であり、

lim_{ε → +0} ε * log(ε) = lim_{ε → +0} - log(1/ε) / (1/ε) = 0

であるから、

lim_{ε → +0} ∫_{ε}^{1} log(x) dx

=

lim_{ε → +0} [(1 * log(1) - 1) - (ε * log(ε) - ε)]

=

lim_{ε → +0} [- 1 - ε * log(ε) + ε]

=

-1

である。

したがって、

lim_{n → ∞} ∫_{1/n}^{1} log(x) dx + lim_{n → ∞} (1/n) * log(1/n) ≦ lim_{n → ∞} a_n ≦ lim_{n → ∞} ∫_{1/n}^{1} log(x) dx

∴ -1 ≦ lim_{n → ∞} a_n ≦ -1

∴ lim_{n → ∞} a_n = -1
0069132人目の素数さん
垢版 |
2019/09/14(土) 22:08:48.45ID:0nc5ufbc
exp(x) は連続関数だから、

lim_{n → ∞} (n!/n^n)^(1/n)

=

lim_{n → ∞} exp(a_n)

=

exp(-1)
0070132人目の素数さん
垢版 |
2019/09/14(土) 22:09:20.00ID:0nc5ufbc
>>58

誤魔化しですが、誤魔化されたことに気づかない人も多いと思います。

そこが質が悪いですよね。
0071132人目の素数さん
垢版 |
2019/09/14(土) 22:09:38.50ID:0nc5ufbc
Michael Spivak著『Calculus 3rd Edition』をチェックしてみたら、

同じ問題がありました。
0072132人目の素数さん
垢版 |
2019/09/14(土) 22:09:54.57ID:0nc5ufbc
13

(a)

f を [1, ∞) で狭義単調増加関数とする。

f(1) + … + f(n-1) < ∫_{1}^{n} f(x) dx < f(2) + … + f(n)

を示せ。

(b)

f = log とし、

n^n / exp(n-1) < n! < (n + 1)^(n + 1) / exp(n)

を示せ。

したがって、

lim_{n → ∞} (n!)^(1/n) / n = 1 / e

が成り立つ。
0073132人目の素数さん
垢版 |
2019/09/14(土) 22:10:10.41ID:0nc5ufbc
(a) 明らか。

(b)

log(1) + … + log(n - 1)

=

log((n - 1)!)


∫_{1}^{n} log(x) dx

=

n * log(n) - n + 1


log(2) + … + log(n)

=

log(n!)

よって、

log((n - 1)!) < n * log(n) - n + 1 < log(n!)

(n - 1)! < n^n / exp(n - 1) < n!



n^n / exp(n - 1) < n! < (n + 1)^(n + 1)
0074132人目の素数さん
垢版 |
2019/09/14(土) 22:10:26.23ID:0nc5ufbc
Spivakさんの解答のほうが理解するのは簡単だと思いますが、思いつくのは大変かもしれませんね。
0075◆Q/LEoOxAUk
垢版 |
2019/09/14(土) 22:25:05.86ID:9H1jzk/l
単調収束定理使ってんじゃないの?
今電車の中でイヤホン持ってきてないから声聞こえないけど説明はなんて言ってんの?
0076◆Q/LEoOxAUk
垢版 |
2019/09/14(土) 22:26:45.66ID:9H1jzk/l
あ、単調収束定理じゃなくて一様可積分性による収束定理ね。
0077132人目の素数さん
垢版 |
2019/09/14(土) 22:29:34.68ID:bw1hEwcz
解くだけなら積分とか使わなくても
分解して

(n!/n^n)=((1/2)^1)*((2/3)^2)*((3/4)^3)*・・・((n-1)/n)^(n-1))*n/n


と見れば
各項の((k-1)/k)^(k-1)という数列、これは定義式からeに収束する数列そのもの、の逆数なので
それらを掛け合わせた(n!/n^n)も(積の)チェザロ平均の法則から同じ値に収束する、とシンプルに求める事はできる

一応、数列a_kが収束するなら (a_1a_2・・・a_n)^(1/n)も同じ値に収束するという法則ね
0078132人目の素数さん
垢版 |
2019/09/14(土) 22:36:02.49ID:bw1hEwcz
ん?積の場合はチェザロ平均とは言わないか
単に幾何平均だな
0079132人目の素数さん
垢版 |
2019/09/14(土) 23:09:42.28ID:0nc5ufbc
>>77

a_k := (k/(k+1))^k とおく。

n!/n^n = a_1 * … * a_{n-1}

n!/n^n = (1/a_n) * (a_1 * … * a_n)

(n!/n^n)^(1/n)

=

(1/a_n)^(1/n) * (a_1 * … * a_n)^(1/n)

=

((n+1)/n) * (a_1 * … * a_n)^(1/n)

lim_{n → ∞} (n!/n^n)^(1/n)

=

lim_{n → ∞} ((n+1)/n) * (a_1 * … * a_n)^(1/n)

=

lim_{n → ∞} ((n+1)/n) * lim_{n → ∞} (a_1 * … * a_n)^(1/n)

=

1 * lim_{n → ∞} a_n

=

lim_{n → ∞} a_n

=

lim_{n → ∞} (n/(n+1))^n

=

lim_{n → ∞} (1 - 1/(n+1))^n

=

lim_{n → ∞} (1 - 1/(n+1))^(n+1) / (1 - 1/(n+1))

=

lim_{n → ∞} (1 - 1/(n+1))^(n+1) / lim_{n → ∞} (1 - 1/(n+1))

=

(1/e) / 1

=

1/e
0080132人目の素数さん
垢版 |
2019/09/14(土) 23:21:22.43ID:0nc5ufbc
(a_n) を正の実数のみからなる数列とし、正の実数 α に収束するとする。

log(x) は連続関数だから、

lim_{n → ∞} log(a_n) = log(α)

である。

log((a_1 * … * a_n)^(1/n))

=

(1/n) * log(a_1 * … * a_n)

=

(log(a_1) + … + log(a_n)) / n

である。

lim_{n → ∞} log((a_1 * … * a_n)^(1/n))

=

lim_{n → ∞} (log(a_1) + … + log(a_n)) / n

=

lim_{n → ∞} log(a_n)

=

log(α)

である。

exp(x) は連続関数だから、

lim_{n → ∞} (a_1 * … * a_n)^(1/n)

=

lim_{n → ∞} exp(log((a_1 * … * a_n)^(1/n)))

=

exp(log(α))

=

α

である。
0081132人目の素数さん
垢版 |
2019/09/14(土) 23:23:19.71ID:0nc5ufbc
>>77

積分を使わないというだけで、この解法のほうが複雑で難しいですね。
0082132人目の素数さん
垢版 |
2019/09/14(土) 23:33:33.55ID:cSRsxQKn
>>78
対数取って
でしょ
チェザロ平均でいいと思う
0083132人目の素数さん
垢版 |
2019/09/14(土) 23:54:46.43ID:bw1hEwcz
>>81
いやさね、どれだけシンプルに答えが出るか
って事なんだ

(n!/n^n)が((1/2)^1)*((2/3)^2)*((3/4)^3)*・・・((n-1)/n)^(n-1))*n/nと分解されるのは行間とかなしで一目みれば明らかだろう

それぞれの項の((k-1)/k)^(k-1)がeの逆数なんてのも見れば明らかだろう

そして数列の調和平均、相乗平均、相加平均が元の数列と同じ値に収束するというのもそういう法則を知っていれば明らか

つまりこの問題の為に必要な計算は1行でいいという事だ


別に積分が悪いってわけじゃないけどね
誤差評価には役立つし
0084132人目の素数さん
垢版 |
2019/09/15(日) 01:24:02.03ID:rJnv6NVW
binary512以降のフォーマット

binary512はExponentが23bit
binary1024はExponentが27bit
binary2048はExponentが31bit
binary4096はExponentが35bit
binary8192はExponentが39bit
binary16384はExponentが43bit
binary32768はExponentが47bit
binary65536はExponentが51bit

である場合の問題点。
0085132人目の素数さん
垢版 |
2019/09/15(日) 02:24:01.12ID:4zTXU1DN
>>60 >>61

(∂/∂x - 1)^n = {exp(x)(∂/∂x)exp(-x)}^n
         = exp(x) (∂/∂x)^n exp(-x),
より
exp(xx) (∂/∂x - 1)^n exp(-xx)
 = exp{x(x+1)} (∂/∂x)^n exp{-x(x+1)}
 = exp{(x+1/2)^2} (∂/∂x)^n exp{-(x+1/2)^2}
 = (-1)^n H_n(x+1/2),   (← ロドリーグの公式)

H_n(x) はn次のエルミート多項式。
0086132人目の素数さん
垢版 |
2019/09/15(日) 03:14:42.71ID:4zTXU1DN
何か変だ。
X = F^(-1) (DF)       (←直後のFにのみ作用)
 = exp(xx)T(-t) D T(t) exp(-xx)
 = exp(xx) (∂/∂x -1) exp(-xx) E
 = (-2x-1) E,
より
X^n = (-2x-1)^n E.
0087132人目の素数さん
垢版 |
2019/09/15(日) 03:27:10.68ID:4zTXU1DN
>>60 >>61 は次の問題の解

〔類題〕
F(x,t) = exp(-xx) [ [ cos(t), -sin(t) ] [ sin(t), cos(t) ] ],
D = [ [∂/∂x, -(∂/∂t)] [∂/∂t, ∂/∂x] ],  … 微分演算子
X_n(x,t) = {F(x,t)^(-1)} D^n F(x,t),
とおく。
自然数nに対して X_n をx,tで表わせ。
0088132人目の素数さん
垢版 |
2019/09/16(月) 22:05:30.66ID:YGXkvnLC
松坂和夫著『解析入門上』を読んでいます。

以下の定理3は、実数値関数についての定理として証明されています。この証明を読むと、複素関数についてもそのまま
通用するのではないかと思うのですが、この定理3の38ページ後ろのページに、「定理3の記述はやや実変数に“局限”
された形になっているから、証明には多少の補正を要しよう。」と書いてあります。

以下の証明のどの部分が「多少の補正を要」するのでしょうか?

なお、証明中の定理1とは一様収束に関するコーシーの条件です。

定理3

I を1つの区間とし、 x_0 を I の1つの点( I の端点でもよい)、 I から x_0 をとり除いた集合を E とする。
(f_n) を E で定義された関数列とし、 (f_n) は E において関数 f に一様収束するとする。また、 n = 1, 2, …
について、有限の極限 lim_{x → x_0} f_n(x) = A_n が存在するとする。そのとき、数列 (A_n) は収束し、その極限を
A とすれば、 lim_{x → x_0} f(x) = A である。

証明

f_n は E で一様収束するから、定理1により、与えられた ε > 0 に対し、ある N が存在して、 m ≧ N, n ≧ N ならば、
すべての x ∈ E に対して |f_m(x) - f_n(x)| < ε が成り立つ。ここで x → x_0 とすれば、 f_m(x) → A_m, f_n(x) → A_n
であるから、 |A_m - A_n| ≦ ε。ゆえに数列 (A_n) はコーシー列である。したがって (A_n) は収束する。その極限を A とする。

f_n は f に E で一様収束し、また A_n → A であるから、自然数 n を十分大きく選んで、すべての x ∈ E に対し
|f(x) - f_n(x)| < ε/3 が成り立ち、かつ |A_n - A| < ε/3 が成り立つようにすることができる。さらにこの n に対し、
lim_{x → x_0} f_n(x) = A_n であるから、 δ > 0 を、 |x - x_0| < δ, x ∈ E ならば、 |f_n(x) - A_n| < ε/3 が
成り立つように選ぶことができる。そうすれば、 |x - x_0| < δ, x ∈ E のとき
|f(x) - A| ≦ |f(x) - f_n(x)| + |f_n(x) - A_n| + |A_n - A| < ε/3 + ε/3 + ε/3 = ε。

これは lim_{x → x_0} f(x) = A であることを意味する。
0089132人目の素数さん
垢版 |
2019/09/16(月) 23:01:08.02ID:FF+PWEgn
不定積分を求めよ。

(1)∫1/(3^x+1) dx

(2)∫log(log(x))/{x・log(x)} dx

(3)∫e^(2x)/√(e^x + 1) dx

[脇スレ.216]
0090132人目の素数さん
垢版 |
2019/09/16(月) 23:39:31.17ID:FF+PWEgn
(1)
3^x + 1 = u とおく。
log(3) 3^x dx = du
(与式1) = ∫{1 - (3^x)/(3^x + 1)} dx
 = x -∫(1/u)du /log(3) = x - log(3^x + 1) /log(3),

(2)
log(log(x)) = u とおく。
1/{x・log(x)} dx = du,
(与式) = ∫ u・(du/dx) dx = (1/2)uu = (1/2){log(log(x))}^2,

(3)
e^x +1 = uu とおく。
 (e^x) dx = 2u du
 (与式) = ∫2(uu-1)du = (2/3)u^3 - 2u
 = (2/3)(uu-3)u = (2/3)(e^x - 2)√(e^x + 1),
0093132人目の素数さん
垢版 |
2019/09/17(火) 11:36:35.77ID:4uKSvV0H
川平友規著『入門複素関数』を読んでいます。

∫_{C} f(z) dz の定義ですが、リーマン和の極限によって定義しています。

ところが、 実数変数の複素数値関数 f(t) = g(t) + i * h(t) に対する

∫_{a}^{b} f(t) dt

の定義は、リーマン和の極限によって定義せず、

∫_{a}^{b} g(t) dt + i * ∫_{a}^{b} h(t) dt

によって定義しています。

統一性が全くありませんよね。
0094132人目の素数さん
垢版 |
2019/09/17(火) 11:41:35.06ID:4uKSvV0H
思ったのですが、

実数変数の実数値関数を含めて、すべて、リーマン和の極限によって積分を定義するのが一番統一性もあり、いいように思います。
0095132人目の素数さん
垢版 |
2019/09/17(火) 11:42:00.35ID:4uKSvV0H
思ったのですが、

実数変数の実数値関数を含めて、すべて、リーマン和の極限によって積分を定義するのが一番統一性もあり、いいように思います。
0097132人目の素数さん
垢版 |
2019/09/17(火) 15:23:44.84ID:4uKSvV0H
>>93
>>94

Serge Langの本では、

∫_{C} f(z) dz := ∫_{a}^{b} f(z(t)) * z'(t) dt

などと定義しています。

これでは、

∫_{C} f(z) dz

の意味が分かりづらいですよね?
0098132人目の素数さん
垢版 |
2019/09/17(火) 16:57:38.34ID:4uKSvV0H
なんか複素関数論って基本的なアイディアは難しいとは思いませんが、厳密にやろうとすると、
途端に非常に難しくなりますね。

ジョルダンの定理とか。
0099132人目の素数さん
垢版 |
2019/09/17(火) 17:04:37.29ID:4uKSvV0H
グリーンの定理というのがあります。

∫_{C} P dx + Q dy = ∬_{Ω} (-P_y + Q_x) dx dy

というものです。

証明は、

∫_{C} P dx = ∬_{Ω} -P_y dx dy



∫_{C} Q dy = ∬_{Ω} Q_x dx dy

とをそれぞれ証明して、積分の加法性から、

∫_{C} P dx + Q dy = ∬_{Ω} (-P_y + Q_x) dx dy

が成り立つをことを証明します。

なぜ、

∫_{C} P dx = ∬_{Ω} -P_y dx dy

のみをグリーンの定理と言わないのでしょうか?

なんか冗長なような気がします。
0100132人目の素数さん
垢版 |
2019/09/17(火) 17:07:22.92ID:4uKSvV0H
∫_{C} P dx = ∬_{Ω} -P_y dx dy

をグリーンの定理とよび、

その系として、

∫_{C} P dx + Q dy = ∬_{Ω} (-P_y + Q_x) dx dy

を書けばいいように思います。
0101132人目の素数さん
垢版 |
2019/09/17(火) 17:10:59.65ID:4uKSvV0H
>>98

グリーンの定理の証明も大体のアイディアは難しくありませんが、厳密な証明は
Ω を有限個の縦線領域と横線領域にどうやって分割するのかとか考えると、
おそらくかなり面倒なことになるなと思われます。
0102132人目の素数さん
垢版 |
2019/09/17(火) 17:12:33.21ID:UMEc5f0A
>>99
>∫_{C} P dx + Q dy = ∬_{Ω} (-P_y + Q_x) dx dy
左辺にカッコ要らないの?要らない場合
∫x^2+2x+3dxでもよくない?
これはダメだというのなら
∫x^2dx+2xdx+3dxなら問題なし?
0103132人目の素数さん
垢版 |
2019/09/17(火) 18:22:55.71ID:FU9MbFQ2
2次正方行列
A = [[1 2] [4 4]]
に対し、
A^n = [[a(n) b(n)] [c(n) d(n)]]
を考える。
このとき a(n)d(n) - b(n)c(n) = (-4)^n を示せ。

[脇スレ..242]

ヒント:   |A^n| = |A|^n
0104132人目の素数さん
垢版 |
2019/09/18(水) 03:26:51.37ID:Pu45bTZg
|tE-A| = tt-5t-4 =0 より Aの固有値は
 α = (5-√41)/2, β = (5+√41)/2,
固有ヴェクトルは
   [ 2 ]      [β-4]
   [α-1]     [ 4 ]
これを使うと
 a(n) = (1/2)(α^n + β^n) - 3(β^n - α^n)/(2√41),
 b(n) = 2(β^n - α^n)/√41,
 c(n) = 4(β^n - α^n)/√41,
 d(n) = (1/2)(α^n + β^n) + 3(β^n - α^n)/(2√41),
よって
a(n)d(n) - b(n)c(n) = (1/4)(α^n + β^n)^2 - (1/4)(β^n - α^n)^2
 = (αβ)^n = (-4)^n,
0105132人目の素数さん
垢版 |
2019/09/18(水) 03:55:50.74ID:tnPs3spN
a[n] = Σ[k=0,n] 1/nCk,
b[n] = Σ[k=0,n] (-1)^k/nCk
ただし nCk は二項係数とする。

(1) n(a[n]-2)→2 (n→∞) を示せ。
(2) b[n] を求めよ。

[脇スレ 195]
0106132人目の素数さん
垢版 |
2019/09/18(水) 05:51:36.54ID:Pu45bTZg
(1)
 nC0 = nCn = 1,
 nC1 = nC(n-1) = n,
 nC2 = nC(n-2) = n(n-1)/2,
3≦k≦n-3 のとき
 nCk ≧ nC3 = n(n-1)(n-2)/6,  (*)
よって
 2 < n(a[n]-2) < 2 + 4/(n-1) + 6(n-5)/{(n-1)(n-2)}
n→∞ とする。

*) nCk = {(n-k+1)/k}・nC(k-1),
  k = [n/2] で最大、その両側に単調減少。

(2)
n:奇数のとき nCk = nC(n-k) より 0
n:偶数のとき
 1/(nCk) = {(n+1)/(n+2)}{1/(n+1)Ck + 1/(n+1)C(k+1)}
より
 b[n] = {(n+1)/(n+2)}{1 + (-1)^n}
    = 2(n+1)/(n+2),
0107132人目の素数さん
垢版 |
2019/09/18(水) 09:08:55.82ID:BuAUiqBt
劣等感ババアの餌です

862 名前:755[sage] 投稿日:2019/09/16(月) 22:01:21.56 ID:???
任意の無限小dxに対して、{f(x+dx)-f(x)}/dxがある実数f’(x)に無限に近くなる時、f(x)は微分可能であるといいます
0108132人目の素数さん
垢版 |
2019/09/18(水) 16:12:37.71ID:Pu45bTZg
2次正方行列
 A = [[1 2] [3 4]]
を考える。nを自然数とし、
 A^n = [[a(n) b(n)] [c(n) d(n)]]
と表すとき、
 a(n)d(n) - b(n)c(n) = (-2)^n,
を示せ。

[脇スレ.240,247]

ヒント:  det(A^n) = (det A)^n.
0109132人目の素数さん
垢版 |
2019/09/18(水) 16:28:05.48ID:Pu45bTZg
|tE-A| = tt-5t-2 =0 より Aの固有値は
 α = (5-√33)/2, β = (5+√33)/2,
固有ヴェクトルは
   [ 2 ]      [β-4]
   [α-1]     [ 3 ]
これを使うと
 a(n) = (α^n + β^n)/2 - 3(β^n - α^n)/(2√33),
 b(n) = 2(β^n - α^n)/(√33),
 c(n) = 3(β^n - α^n)/(√33),
 d(n) = (α^n + β^n)/2 + 3(β^n - α^n)/(2√33),
よって
 a(n)d(n) - b(n)c(n) = (1/4)(α^n + β^n)^2 - (1/4)(β^n - α^n)^2
 = (αβ)^n = (-2)^n,
0110132人目の素数さん
垢版 |
2019/09/18(水) 17:04:11.91ID:mmsTqSXM
川平友規著『入門複素関数』を読んでいます。


演習問題に、

∫_{0}^{2*π} 1 / (a^2 * cos^2(x) + b^2 * sin^2(x)) dx

の値を計算させる問題があります。

こういう積分を簡単に計算できるのは素晴らしいですね。

でも、

∫_{0}^{2*π} 1 / (a^2 * cos^2(x) + b^2 * sin^2(x)) dx

↑こういう定積分を見たときに、それに応じてどういう複素線積分を考えればいいかを思いつかないと
いけないですよね。
0111132人目の素数さん
垢版 |
2019/09/18(水) 22:34:14.97ID:Pu45bTZg
(b/a)tan(x) = t とおいて
∫ 1 /{aa*cos(x)^2 + bb*sin(x)^2} dx
 = (1/ab)∫ 1/(1+tt) dt
 = (1/ab)*arctan(t)
 = (1/ab)*arctan((b/a)tan(x))
を思いつかないといけないですよね。
0114132人目の素数さん
垢版 |
2019/09/19(木) 15:01:42.96ID:/rMlpsDI
>>105 (1)
 
n>>1 のとき
Σ[k=0,n] 1/(nCk) = 2(1 + 1/n + 2/n^2 + 8/n^3 + 44/n^4 + 308/n^5 + 2612/n^6 + 25988/n^7 + ・・・・)
0115132人目の素数さん
垢版 |
2019/09/19(木) 23:11:57.62ID:/rMlpsDI
〔問題〕
3辺の長さがそれぞれa,b,cの三角形Tをとる。
0<a≦b≦c<a+b かつ a+b+c=1 の条件下で実数a,b,cを動かすとき、
Tの面積凾最大にするa,b,cを求めよ。
0116132人目の素数さん
垢版 |
2019/09/19(木) 23:22:33.59ID:/rMlpsDI
GM-AM より
刧 = (1/16)(a+b+c)・(-a+b+c)(a-b+c)(a+b-c)
 ≦ (1/16)(a+b+c){(a+b+c)/3}^3
 = (1/16)(1/27)(a+b+c)^4,
≦ {1/(12√3)}(a+b+c)^2,
等号成立は a=b=c =1/3.
0117132人目の素数さん
垢版 |
2019/09/19(木) 23:25:01.32ID:/rMlpsDI
〔問題273〕
等面四面体Sの各側面は合同である。すなわち、3辺の長さがa,b,cの三角形Tである。
0<a≦b≦c<a+b かつ a+b+c=1 の条件下で実数a,b,cを動かすとき、Sの体積Vを最大にするa,b,cを求めよ。

[脇スレ.273]
0118 【末吉】
垢版 |
2019/09/20(金) 00:03:57.12ID:WUyp0FDI
>>10
>>117
a+b+c=1
b=1/3とすると、
a+c=2/3=8/12
a=3/12=1/4,
c=5/12のとき、
a^2+b^2=(1/4)^2+(1/3)^2
=1/16+1/9
=25/144
=(5/12)
=c^2
ピタゴラスの定理より、
長さaの辺と長さbの辺は直角に交わる。
面積のとき考えてこうなんだから、体積のときに拡張し、
S=(1/3)(1/2)abc
=(1/3)(1/2)(1/4)(1/3)(5/12)
=5/864
a=1/4
b=1/3
c=5/12
0120132人目の素数さん
垢版 |
2019/09/20(金) 00:41:19.22ID:ce/riRSP
直角凾フとき
(0,0,0)
(a,0,ε)
(a,b,0)
(0,b,ε)
を頂点とする「4面体」と考える。
ε→0 とすると潰れて長方形になる。
0121132人目の素数さん
垢版 |
2019/09/20(金) 00:52:47.25ID:ce/riRSP
>>117
Sの頂点は、或る直方体の4頂点である。
その稜の長さをp,q,rとすれば
 a = √(qq+rr) ≧ (q+r)/√2,
 b = √(rr+pp) ≧ (r+p)/√2,
 c = √(pp+qq) ≧ (p+q)/√2,
 Tは鋭角△
したがって
 V = (1/3)pqr
  ≦ (1/24)(p+q)(q+r)(r+p)  (← GM-AM)
  ≦ {1/(6√2)}abc
  ≦ {1/(162√2)}(a+b+c)^3,  (← GM-AM)
等号成立は p=q=r, a=b=c=1/3.
0122132人目の素数さん
垢版 |
2019/09/20(金) 05:48:41.67ID:ce/riRSP
〔問題278〕
以下を示せ。

・a[n] = √(3nn + 1) が整数となる自然数nは無数にある。

・任意の正の実数εに対し、ある自然数の組(m,n)が存在して、
 |a[n] - m| < εとなるようにできる。

[脇スレ.278]
0123132人目の素数さん
垢版 |
2019/09/20(金) 13:24:55.51ID:KyAOfC1j
2500
かずきち@dy_dt_dt_dx 8月28日
学コン8月号Sコース1等賞1位とれました!
マジで嬉しいです!
来月からも理系に負けず頑張りたいと思います!
https://twitter.com/dy_dt_dt_dx
https://twitter.com/5chan_nel (5ch newer account)
0124132人目の素数さん
垢版 |
2019/09/20(金) 15:38:01.73ID:ce/riRSP
演習問題に
 ∫ 1/(aa*cos(x)^2 + bb*sin(x)^2}^2 dx
の値を計算させる問題があります。
こういう積分を簡単に計算できるのは素晴らしいですね。
でも、こういう不定積分を見たときに、それに応じて
どういう置換積分を考えればいいかを思いつかないと
いけないですよね。
0125132人目の素数さん
垢版 |
2019/09/20(金) 16:16:43.93ID:ce/riRSP
(b/a)tan(x) = t とおいて
(b/a)dx/cos(x)^2 = dt,
aa・cos(x)^2 + bb・sin(x)^2 = aa・(1+tt)cos(x)^2
 = (ab)^2 (1+tt)/{(at)^2+bb},
より
∫1/{aa・cos(x)^2 + bb・sin(x)^2}^2 dx
 = 1/(ab)^3 ∫ {(at)^2 + bb}/(1+tt)^2 dt
 = 1/{2(ab)^3} ∫{(bb+aa)/(1+tt) + (bb-aa)・(1-tt)/(1+tt)^2} dt
 = 1/{2(ab)^3} [ (bb+aa)・arctan(t) + (bb-aa)・t/(1+tt) ]
 = 1/{2(ab)^3} (bb+aa)・arctan((b/a)tan(x))
  + 1/{2(ab)^2} (bb-aa)・cos(x)sin(x)/{aa・cos(x)^2 + bb・sin(x)^2},
を思いつかないといけないですよね。
0127132人目の素数さん
垢版 |
2019/09/20(金) 16:21:30.50ID:ce/riRSP
演習問題に
 ∫ 1/(aa*cos(x)^2 + bb*sin(x)^2}^3 dx
の値を計算させる問題があります。
こういう積分を簡単に計算できるのは素晴らしいですね。
でも、こういう不定積分を見たときに、それに応じて
どういう置換積分を考えればいいかを思いつかないと
いけないですよね。
0128132人目の素数さん
垢版 |
2019/09/20(金) 16:42:41.56ID:lq2/XEro
>>111

cos(π/2) = cos(3*π/2) = 0

なので、少し面倒ですね。

∫_{0}^{2*π} 1 / (a^2 * cos^2(x) + b^2 * sin^2(x)) dx

=

∫_{-π/2}^{3*π/2} 1 / (b^2 * cos^2(x) + a^2 * sin^2(x)) dx

=

∫_{-π/2}^{π/2} 1 / (b^2 * cos^2(x) + a^2 * sin^2(x)) dx

+

∫_{π/2}^{3*π/2} 1 / (b^2 * cos^2(x) + a^2 * sin^2(x)) dx
0129132人目の素数さん
垢版 |
2019/09/20(金) 16:44:03.42ID:lq2/XEro
>>128

=

2 * ∫_{-π/2}^{π/2} 1 / (b^2 * cos^2(x) + a^2 * sin^2(x)) dx
0130132人目の素数さん
垢版 |
2019/09/20(金) 16:49:43.50ID:lq2/XEro
>>129

∫_{-π/2}^{0} 1 / (b^2 * cos^2(x) + a^2 * sin^2(x)) dx

=

∫_{0}^{π/2} 1 / (a^2 * cos^2(x) + b^2 * sin^2(x)) dx

だから、

2 * ∫_{-π/2}^{π/2} 1 / (b^2 * cos^2(x) + a^2 * sin^2(x)) dx

=

2 * (∫_{0}^{π/2} 1 / (a^2 * cos^2(x) + b^2 * sin^2(x)) dx + ∫_{0^{π/2} 1 / (b^2 * cos^2(x) + a^2 * sin^2(x)) dx)
0131132人目の素数さん
垢版 |
2019/09/20(金) 16:57:12.48ID:lq2/XEro
>>130

g(x) = 1 / (a^2 * cos^2(x) + b^2 * sin^2(x))
f(x) = (1 / (a^2 * cos^2(x))) / (1 + (b/a)^2 * tan^2(x))

とおく。

g(x) = f(x) for all x ∈ [0, π/2)

∫_{0}^{π/2} 1 / (a^2 * cos^2(x) + b^2 * sin^2(x)) dx

=

lim_{c → π/2} ∫_{0}^{c} 1 / (a^2 * cos^2(x) + b^2 * sin^2(x)) dx

=

lim_{c → π/2} ∫_{0}^{c} g(x) dx

=

lim_{c → π/2} ∫_{0}^{c} f(x) dx
0132132人目の素数さん
垢版 |
2019/09/20(金) 17:08:32.70ID:lq2/XEro
∫_{0}^{c} f(x) dx

=

(1/(a*b)) * ∫_{0}^{(b/a)*tan(c)} 1 / (1 + t^2) dt

=

(1/(a*b)) * [arctan((b/a)*tan(c)) - arctan(0)]

=

(1/(a*b)) * arctan((b/a)*tan(c))




lim_{c → π/2} ∫_{0}^{c} f(x) dx

=

lim_{c → π/2} (1/(a*b)) * arctan((b/a)*tan(c))

=

(1/(a*b)) * (π/2)
0133132人目の素数さん
垢版 |
2019/09/20(金) 17:09:26.95ID:lq2/XEro
同様に、

∫_{0^{π/2} 1 / (b^2 * cos^2(x) + a^2 * sin^2(x)) dx

=

(1/(a*b)) * (π/2)
0134132人目の素数さん
垢版 |
2019/09/20(金) 17:10:47.82ID:lq2/XEro


∫_{0}^{2*π} 1 / (a^2 * cos^2(x) + b^2 * sin^2(x)) dx

=

2 * (∫_{0}^{π/2} 1 / (a^2 * cos^2(x) + b^2 * sin^2(x)) dx + ∫_{0^{π/2} 1 / (b^2 * cos^2(x) + a^2 * sin^2(x)) dx)

=

2 * (2 * (1/(a*b)) * (π/2))

=

(2*π) / (a*b)
0135132人目の素数さん
垢版 |
2019/09/20(金) 17:13:00.01ID:lq2/XEro
>>112

高校生でも思いつくとは思いますが、

cos(π/2) = cos(3*π/2) = 0

なので、どうすればいいのか分からないのではないでしょうか?
0137132人目の素数さん
垢版 |
2019/09/20(金) 22:27:12.59ID:lq2/XEro
志賀浩二著『数学が育っていく物語 第2週 解析性』を読んでいます。

テイラーの公式の剰余項が 0 に収束することを証明して、

exp(x), sin(x), cos(x) がテイラー展開可能であることを導いています。

次に、

log(1 + x) のテイラー展開ですが、これについては、

志賀浩二著『数学が育っていく物語 第1週 極限の深み』で、べき級数の理論を使って求めています。

log(1 + x) のテイラーの公式の剰余項が 0 に収束することを直接証明することは難しい理由を以下のように

説明してます。

R_n = (-1)^(n+1) * x^n / (n * (1 + θ*x)^n)

の θ は x と n の関数で 0 < θ < 1 を満たします。

最悪の状況を想定すると、 n を大きくしていったとき θ がずっと 1 に近いままであるかもしれません。

もし、たとえば、 x = -2/3 のときに、そのような状況が起きるとすると、

|R_n| ≒ (1/n) * (2/3)^n / (1 - 2/3)^n = 2^n / n → ∞

となってしまいます。

R_n → 0 であることを証明するには、このような状況が起きないことを証明しなければならず、それは難しい。

950 自分:132人目の素数さん[] 投稿日:2019/09/20(金) 22:26:16.69 ID:lq2/XEro [7/7]
志賀浩二さんの本もたまには少し面白い話が書いてありますね。

log(1 + x) のテイラーの公式の剰余項が 0 に収束することを直接証明することはできますか?
0138132人目の素数さん
垢版 |
2019/09/21(土) 02:47:32.38ID:Nou2F8U6
-1<x<0 のときは
 f(x) = f(0) - ∫[x,0] f '(t) dt
  = ・・・・  (部分積分をn-1回)
  = (n-1次のMcLaurin多項式) + R_n(x),
ここに
 R_n(x) = (-1)^n {1/(n-1)!}∫[x,0] f^(n)(t) (t-x)^(n-1) dt.
   ・・・・・ 積分型剰余 (ベルヌーイの剰余) と云うらしい

さて本問では
f(t) = log(1+t),
f^(n)(t) = (-1)^(n-1) (n-1)! /(1+t)^n,
また
-1<x≦t≦0,
0 ≦ (t-x)/(t+1) ≦ -x = |x|,
1 ≦ 1/(1+t) ≦ 1/(1+x),
よって
| 被積分函数 | < |x|^n /(1+x),
|R_n(x)| < |x|^(n-1) /(1+x) ∫[x,0] dt = |x|^n /(1+x) → 0 (n→∞)
0139132人目の素数さん
垢版 |
2019/09/21(土) 04:09:43.94ID:Nou2F8U6
〔問題〕
nを3以上の自然数とする。
 n次正方行列Aのnn個の成分のうち少なくとも1つは虚数である。
 また、Aの成分はいずれも0でない。
 このとき次の命題Pが成り立つことを示せ。
『命題P』 AA = O となるAが存在する。
ただし、この問題において「虚数」とは実数でない複素数を指す。

[脇スレ.263,274]
0140132人目の素数さん
垢版 |
2019/09/21(土) 04:15:57.24ID:Nou2F8U6
例)
 1のn乗根を ω = e^(2πi/n) として
 A_(j,k) = ω^{(j+k)/2} = e^{iπ(j+k)/n}.
0143132人目の素数さん
垢版 |
2019/09/21(土) 15:42:38.61ID:Nou2F8U6
>>137
コーシー剰余を使えば・・・・
-1 < x < 0 < θ < 1 より
1+θx > 1-θ > 0 かつ 1+θx > 1+x > 0,
| f^(n)(θx) | = (n-1)! /(1+θx)^n < (n-1)! /[(1+x)(1-θ)^(n-1)],
∴ コーシー剰余は
| R_n(x) | = {1/(n-1)!} |f^(n)(θx) (1-θ)^(n-1) x^n |
  < |x|^n /(1+x) → 0  (n→∞)

[脇スレ.336,341]
0144132人目の素数さん
垢版 |
2019/09/21(土) 18:51:50.43ID:Nou2F8U6
>>121 (補足)

四面体の各側面の面積ヴェクトルをp,q,r,sとおく。
等面積だから |p|=|q|=|r|=|s|,
閉じた多面体だから p+q+r+s=o,
∴ p+q = - (r+s), p+r = -(q+s), p+s = -(q+r), …(1)
∴ (p+q)・(p+r) = (-r-s)・p + (p+q)・r
  = -(s・p) + (q・r)
  = {|p|^2 + |s|^2 -|s+p|^2}/2 + {|q+r|^2 -|q|^2 -|r|^2}/2
  = 0,
∴ (1) は直交系をなす。
それらをxyz軸とすれば、Sの2回軸となる。
∴ Sの頂点は、xyz軸に平行な稜をもつ或る直方体の4頂点である。
0145132人目の素数さん
垢版 |
2019/09/21(土) 18:55:11.26ID:Nou2F8U6
〔類題〕
四つの面がすべて等面積の四面体においては、
四つの面をなす三角形は互いに合同であることを証明してください。

数セミ増刊「数学の問題」第(1)集 ●45 (1977)
0146132人目の素数さん
垢版 |
2019/09/22(日) 12:42:18.83ID:Yh7/QcPR
閉じた多面体の面積ベクトルの和が 0 って
どうやって証明するんだ?
4面体なら簡単だが
0147132人目の素数さん
垢版 |
2019/09/22(日) 13:23:51.64ID:2TbS0DPZ
川平友規著『入門複素関数』を読んでいます。

以下の問題の川平さんの解答ですが、非常に長いものになっています。


関数 g(z), h(z) は点 α を含む領域上の正則関数とし、条件

g(α) ≠ 0
h(α) = 0
h'(α) ≠ 0

をみたすものとする。

このとき、点 α は関数 g(z) / h(z) の1位の極であることを示せ。
0148132人目の素数さん
垢版 |
2019/09/22(日) 13:24:08.14ID:2TbS0DPZ
以下の簡単な解答でOKだと思いますがどうでしょうか?


解答:

関数 h(z) は点 α を含む領域上の正則関数であるから、 α の近くで、

h(z) = a_0 + a_1 * (z - α) + a_2 * (z - α)^2 + …

とべき級数展開できる。

0 = h(α) = a_0

であり、

h'(z) = a_1 + 2 * a_2 * (z - α) + …

0 ≠ h'(α) = a_1

であるから、

α の近くで、

h(z) = a_1 * (z - α) + a_2 * (z - α)^2 + …
a_1 ≠ 0

である。

h(z) = (z - α) * [a_1 + a_2 * (z - α) + …]

である。

f(z) := a_1 + a_2 * (z - α) + …



点 α を含む領域上の正則関数であり、 f(α) ≠ 0 であるから、

g(z) / f(z)

も点 α を含む領域上の正則関数である。よって、 α の近くで、

g(z) / f(z) = b_0 + b_1 * (z - α) + b_2 * (z - α)^2 + …

とべき級数展開できる。
0149132人目の素数さん
垢版 |
2019/09/22(日) 13:24:24.07ID:2TbS0DPZ
g(z) / h(z)

=

(1 / (z - α)) * [b_0 + b_1 * (z - α) + b_2 * (z - α)^2 + …]

=

b_0 / (z - α) + b_1 + b_2 * (z - α) + …



g(z) / h(z)

のローラン展開であり、明らかに点 α は関数 g(z) / h(z) の1位の極である。
0150132人目の素数さん
垢版 |
2019/09/22(日) 14:14:29.62ID:2TbS0DPZ
あ、よく考えたら、

川平さんの本では、べき級数の話は付録に登場するだけでした。

べき級数で表される関数が正則であることは証明されていませんね。
0151132人目の素数さん
垢版 |
2019/09/22(日) 14:16:30.40ID:2TbS0DPZ
べき級数で表される関数が正則であること

を使わないで証明するとすると面倒なことになりますね。
0152132人目の素数さん
垢版 |
2019/09/22(日) 14:33:09.84ID:xM7D1Xt7
別にローラン展開の話に持ってっても普通に正しいと思うが

そのまま直接定義通りにやっても
z-aかけりゃ普通に正則だろうし1行で終わるだろうと思うが
0153132人目の素数さん
垢版 |
2019/09/22(日) 15:04:29.81ID:xM7D1Xt7
特異点周りで有界であると実質特異点ではないとか
一致の定理だとか
そう言う基本性質は前提だが
0154132人目の素数さん
垢版 |
2019/09/22(日) 15:25:07.33ID:2TbS0DPZ
>>152

ありがとうございます。

>>153

そういう定理はまだ紹介されていません。



H(z) := h(z) / (z - α)



lim_{z → α} H(z) = lim_{z → α} (h(z) - h(α)) / (z - α) = h'(α) ≠ 0 だから

H(z) は α の近くで有界であると書いてあるのですが、それはなぜですか?


ちなみに、コンパクト集合上で連続な関数は最大値、最小値をもつ

という定理は、付録で紹介されているので、この定理は使えません。
0155132人目の素数さん
垢版 |
2019/09/22(日) 15:32:19.05ID:BxpI4hgl
>>154
>ちなみに、コンパクト集合上で連続な関数は最大値、最小値をもつ

>という定理は、付録で紹介されているので、この定理は使えません。

とんでもないアホを見た
0156132人目の素数さん
垢版 |
2019/09/22(日) 15:33:15.20ID:2TbS0DPZ
>>154

やはり、

「コンパクト集合上で連続な関数は最大値、最小値をもつ」

という定理を暗に使っているようにしか思えません。
0157132人目の素数さん
垢版 |
2019/09/22(日) 15:48:08.19ID:2TbS0DPZ
>>154

あ、分かりました。

lim_{z → α} H(z) = lim_{z → α} (h(z) - h(α)) / (z - α) = h'(α) ∈ C

だから、

ある正の実数 r が存在して、 {z | 0 < |z - α| < r} 上で H(z) は有界である。
0158132人目の素数さん
垢版 |
2019/09/22(日) 15:48:23.65ID:R1QLTtDK
>>150
べき級数で表される関数f(z)が D = { z | |z-α|<R } で収束したとする。
(z-α)^n までの部分和を S_n(z) とすると、△不等式より
 |f(z1)-f(z2)| ≦ |f(z1)-S_n(z1)| + |S_n(z1) - S_n(z2)| + |S_n(z2)-f(z2)|,
右辺の第1項と第3項は n>N に対して <ε,
右辺の第2項も |z1-z2|<δ に対して <ε,
∴ f(z) はD内で連続である。
∴ f(z) はD内で積分可能。

 |f(z) - S_n(z)| → 0  (n→∞)
 Max{ |f(z)-S_n(z)| | z∈C } = M_n → 0  (n→∞)
∴ 「項別積分」してよい。
D内の閉曲線Cに沿ってf(z)を積分すると S_n(z) の部分は消えて
 |∫_C f(z)dz | = |∫_C {f(z)-S_n(z)}dz | + M_n∫_C |dz| → 0 (n→∞)
∴ ∫_C f(z)dz = 0,
CはD内の任意のJordan曲線でよいから、Moreraの定理より、
f(z)はDで正則である。
0159132人目の素数さん
垢版 |
2019/09/22(日) 15:51:17.32ID:xM7D1Xt7
数学書の付録っていうのは
エクストラで紹介するだけ的な意味合いのものもあれば
こういう性質や理論を使って議論するので押さえておいてくださいねっていう物もある
どちらかというと後者が多いんじゃないかな

複素関数の理論話してるのに位相の定理とかから証明しだすとどんどん話が逸れるだろ
そういう時に話を分ける意味で他の分野の知識などは付録にまとめておく
0160132人目の素数さん
垢版 |
2019/09/22(日) 15:53:26.82ID:2TbS0DPZ
あ、

>>154

で書いたのと川平さんが書いたことは少し違ってました。

以下に川平さんの解答から引用します:

H(z) := h(z) / (z - α)

とおく。これはある十分に小さな穴あき円板 D := D(α, r) - {α} 上で正則である。

h(α) = 0 より、 z → α のとき

H(z) = (h(z) - h(α)) / (z - α) → h'(α) ≠ 0

が成り立つから、 D 上の正則関数 F(z) := g(z) / H(z) は有界である。
0161132人目の素数さん
垢版 |
2019/09/22(日) 15:58:19.04ID:2TbS0DPZ
>>160

要するに、

lim_{z → α} F(z) = lim_{z → α} g(z) / H(z) ∈ C

だから、

十分小さな r に対して、 D(α, r) - {α} 上で F(z) は有界である

ということですね。

でも、

>>160

のような書き方だと最初にとった r をもっと小さな r に取り直さないといけない可能性がありますよね。

川平さんは、


D 上の正則関数 F(z) := g(z) / H(z) は有界である。


と書いていますが、最大最小値の定理を使わないのであれば、この書き方は不適切ですね。
0162132人目の素数さん
垢版 |
2019/09/22(日) 15:58:49.22ID:xM7D1Xt7
>>154
Hの有界性を示して何になるのかはわからんが
limitが存在してりゃそりゃ有界だよ
1/Hの有界性なら
正則関数の零点が孤立点である事と連続性から明らか
0163132人目の素数さん
垢版 |
2019/09/22(日) 15:59:37.42ID:R1QLTtDK
>>146
k番目の面の面積ヴェクトルをp_k としましょう。
この多面体を川の水に漬けます。
水の流速vは一定とします。
単位時間にこの多面体に
流れ込む水の体積−出ていく水の体積は
  (v・Σp_k) = 0,
これはvがどちらを向いてても成り立ちますから
  Σp_k = o
(流体運動学による証明)
0164132人目の素数さん
垢版 |
2019/09/22(日) 16:02:45.25ID:2TbS0DPZ
>>160


H(z) := h(z) / (z - α)

とおく。これはある十分に小さな穴あき円板 D := D(α, r) - {α} 上で正則である。

h(α) = 0 より、 z → α のとき

H(z) = (h(z) - h(α)) / (z - α) → h'(α) ≠ 0

が成り立つから、 D 上の正則関数 F(z) := g(z) / H(z) は有界である。


これを以下のように訂正すれば問題ないですね。


H(z) := h(z) / (z - α)

とおく。これはある十分に小さな穴あき円板 D := D(α, r) - {α} 上で正則である。

h(α) = 0 より、 z → α のとき

H(z) = (h(z) - h(α)) / (z - α) → h'(α) ≠ 0

が成り立つから、 必要ならば、上の r をもっと小さくとりなおせば、 D 上の正則関数 F(z) := g(z) / H(z) は有界である。
0165132人目の素数さん
垢版 |
2019/09/22(日) 18:39:36.40ID:2TbS0DPZ
以下のリーマンの定理が演習問題にあります。

f(z) = … + a_{-2} / (z - α)^2 + a_{-1} / (z - α) + a_0 + a_{1} * (z - α) + a_{2} * (z - α)^2 + …

と書いてみれば、

lim_{z → α} |a_{-n} / (z - α)^n| = +∞

なので明らかであるようにも見えます。

この線で、リーマンの定理を証明できませんか?


ちなみに、川平さんの解答では、 ML不等式を使って分かりやすく証明しています。



リーマンの定理:

関数 f(z) が穴あき円板 D = {z ∈ C | 0 < |z - α| < R} 上で正則かつ有界であるとき、
α は f(z) の除去可能な特異点であることを示せ。
0166132人目の素数さん
垢版 |
2019/09/22(日) 23:19:49.04ID:2TbS0DPZ
川平友規著『入門複素関数』を読んでいます。

以下の演習問題があります。


関数 f(z) は穴あき円板 D = {z ∈C | 0 < |z - α| < R} 上で正則であり、 α は f(z) の除去可能特異点であるとする。
このとき、ある D(α, r) 上の正則関数 g(z) で、 D 上 g(z) = f(z) をみたすようなものが存在することを示せ。


これは非常に簡単な問題ですが、べき級数の理論を使わない川平さんの解答は恐ろしく長いです。

以下のように、ほぼ自明な問題であるにもかかわらずです。

z ∈ D とする。

f(z) = a_0 + a_1 * (z - α) + a_2 * (z - α)^2 + …

とローラン展開できる。

g(z) := f(z) if z ∈ D
g(z) := a_0 if z = α

で定義される D(α, R) 上の関数 g(z) は D(α, R) 上の正則関数である。

8 自分返信:132人目の素数さん[] 投稿日:2019/09/22(日) 23:09:19.26 ID:2TbS0DPZ [2/2]
>>7

あと、


このとき、ある D(α, r) 上の正則関数 g(z) で


と書いてありますが、明らかに、


このとき、 D(α, R) 上の正則関数 g(z) で


としたほうがいいですよね?
0167132人目の素数さん
垢版 |
2019/09/23(月) 16:52:38.99ID:2PqEJji0
>>127
(b/a)tan(x) = t とおいて
(b/a)dx/cos(x)^2 = dt,
によりtで積分する。 被積分函数は
(a/b) cos(x)^2 /{aa・cos(x)^2 + bb・sin(x)^2}^3
 = {1/(ab)^5} (aatt+bb)^2 /(1+tt)^3
 = p/(1+tt) + (q/ab)(1-tt)/(1+tt)^2 + (r/aaab)(1-3tt)/(1+tt)^3,
∴ F(x) = p・arctan(t) + (q/ab)t/(1+tt) + (r/aaab)t/(1+tt)^2
 = p・arctan{(b/a)tan(x)} + q・cos(x)・sin(x)/{aa・cos(x)^2 + bb・sin(x)^2}
  + r・cos(x)^3・sin(x)/{aa・cos(x)^2 + bb・sin(x)^2}^2,
ここに
 p = (1/8)(3a^4 +2aabb +3b^4)/(ab)^5,
 q = (1/8)(5aa+3bb)(bb-aa)/(ab)^4,
 r = (1/4)(bb-aa)^2/(aa・b^4).
を思いつかないといけないですよね。
0169132人目の素数さん
垢版 |
2019/09/23(月) 18:59:51.46ID:3W6wuIwm
留数について質問です。

f(z) は {z ∈ C | 0 < |z - α| < R} を含む領域上で正則であるとする。

という仮定をしますが、

なぜ、

f(z) は {z ∈ C | R_1 < |z - α| < R_2} を含む領域上で正則であるとする。

という仮定はしないのでしょうか?
0170132人目の素数さん
垢版 |
2019/09/24(火) 00:50:42.80ID:CUDTSBu2
>>146
四面体OABCでは
 2p = OA×OB,
 2q = OB×OC,
 2r = OC×OA,
 2s = AC×AB = (OC-OA)×(OB-OA)
   = -OA×OB -OB×OC -OC×OA
より成立。
 多面体は四面体を何個か貼り合わせたもの。
 貼り合わせ面では相殺してoになる。
0172132人目の素数さん
垢版 |
2019/09/24(火) 20:31:15.81ID:qOGR6zKw
川平友規著『入門複素関数』を読んでいます。

∫_{-∞}^{∞} 1 / (1 + x^4) dx = π / sqrt(2)

という等式を示す例題があります。

その例題では、

lim_{R → ∞} ∫_{-R}^{R} 1 / (1 + x^4) dx = π / sqrt(2)

を示しています。

本来示すべきは、

lim_{S → -∞, T → ∞} ∫_{S}^{T} 1 / (1 + x^4) dx = π / sqrt(2)

ですよね。


2 * ∫_{0}^{R} 1 / (1 + x^4) dx = ∫_{-R}^{R} 1 / (1 + x^4) dx = π / sqrt(2)

∫_{0}^{R} 1 / (1 + x^4) dx = (1/2) * ∫_{-R}^{R} 1 / (1 + x^4) dx = π / (2 * sqrt(2))

なので、

lim_{S → -∞, T → ∞} ∫_{S}^{T} 1 / (1 + x^4) dx

=

π / sqrt(2)ですけど。
0173132人目の素数さん
垢版 |
2019/09/25(水) 03:33:41.20ID:HNtypll1
複素関数を使って解こうという趣旨だけど、実数の範囲内でも解けなくはない。
1 + x^4 = (1+xx)^2 - 2xx
= (1 +x√2 +xx)(1 -x√2 +xx)
= {1 + (1+x√2)^2}{1 + (1-x√2)^2}/4,

1/(1+x^4) = 1/(4√2)・{(√2 +2x)/(1 +x√2 +xx) - (-√2 +2x)/(1 -x√2 +xx)}
 + (1/2)/{1 +(1 +x√2)^2} + (1/2)/{1 +(1 -x√2)^2},
と部分分数に分けて
∫1/(1+x^4) dx = 1/(4√2)・{log(1+x√2 +xx) - log(1 -x√2 +xx)}
 + 1/(2√2)・{arctan(1+x√2) - arctan(1-x√2)}
 = 1/(4√2)・log[(1+x√2 +xx)/(1 -x√2 +xx)]
  + 1/(2√2)・arctan{(√2)x/(1-xx)},
0174132人目の素数さん
垢版 |
2019/09/25(水) 23:38:42.00ID:iXHFXXRG
川平友規著『入門複素関数』を読んでいます。

第4章「留数定理」の章末問題に以下の問題があります。

a > 0 とする。

∫_{-∞}^{∞} x^4 / (x^2 + a^2)^4 dx

の値を求めよ。

定石通りに計算すれば、答えが求まりますが、

g(z) := z^4 / (z + a*i)^4

の3次導関数を計算しなければなりません。

g(z) を 1 / (z + a*i) についての4次多項式で表して、なんとか3次導関数を計算しましたが、
かなり苦労しました。

簡単に計算する方法はありますか?
0175132人目の素数さん
垢版 |
2019/09/25(水) 23:39:43.33ID:iXHFXXRG
>>173

確かに複素関数論を知らなくても解けますね。
0176132人目の素数さん
垢版 |
2019/09/26(木) 02:00:06.61ID:SDysta5y
直接ローラン展開が出来たり…しないか中々
0177132人目の素数さん
垢版 |
2019/09/26(木) 10:51:47.22ID:GlcVFFf+
川平友規著『入門複素関数』を読んでいます。

第4章「留数定理」の章末問題に以下の問題があります。


∫_{0}^{∞} exp(-x^2) dx = sqrt(π) / 2 を用いて、

∫_{0}^{∞} sin(x^2) dx = ∫_{0}^{∞} cos(x^2) dx = sqrt(π) / (2 * sqrt(2))

を示せ。


この問題を自力で解けました。

結構すごいですか?


第4章に出てくる積分の積分路は決まって半円だったので、最初は戸惑いました。

が、↓が閃きました。


f(z) := exp(z^2)

とおくと、

f(i*t) = exp(-t^2)

f(sqrt(i) * t) = exp(i * t^2) = cos(t^2) + i * sin(t^2)


なかなか冴えていますか?


この問題が第4章の章末問題のラストを飾る問題です。

しかも、☆印つきの問題です。

「はじめに」には、


とくに発展的な問題には*をつけ区別してある。


などと書かれています。

気持ちよく、最終章第5章へと進むことができそうです。
0178132人目の素数さん
垢版 |
2019/09/26(木) 16:33:22.80ID:GlcVFFf+
>>177

の問題を自力で解けたということは、もう既に、「玲瓏なる境地」に達していると考えていいですか?
0179132人目の素数さん
垢版 |
2019/09/26(木) 18:00:52.69ID:GlcVFFf+
川平友規著『入門複素関数』を読んでいます。

∫_{-∞}^{∞} cos(x) / (1 + x^2)^2 dx

を計算せよ。

という問題を解きました。

怪しいなと思いつつ、まず以下の積分を考えました:

∫_{C} cos(z) / (1 + z^2)^2 dz

cos(z) = (exp(i * z) + exp(-i * z)) / 2

です。

|exp(i * z)| = exp(-y)
|exp(-i * z)| = exp(y)

ですので、普通に積分路を考えると 0 と評価したい積分が 0 と評価できません。

そこで、

∫_{C} exp(i * z) / (1 + z^2)^2 dz

を考えれば、

|exp(i * z)| = exp(-y)

ですから、 z の虚部が大きくなるような場所を通る積分路を考えれば、 0 と評価したい
積分を 0 と評価できそうです。

このような推理の結果、正解を得ることができました。
0180132人目の素数さん
垢版 |
2019/09/26(木) 18:03:30.48ID:GlcVFFf+
あ、というか、 |exp(-y)| ≦ 1 for y ≧ 0 ですね。
0181132人目の素数さん
垢版 |
2019/09/27(金) 18:09:38.57ID:ncViLEfF
>>177
(上) は数学者にとっては 2x2=4 と同じくらい明らかです。(ケルビン)

>>178
ちょこっと計算して数が合ってると「玲瓏なる境地」に達しているとうぬぼれながら、解析接続すら理解できていないバカですからね。
それにこの人間性じゃ受け入れ先も全くないでしょう。
[脇スレ.460]
0182132人目の素数さん
垢版 |
2019/09/27(金) 21:12:45.61ID:N/cfTNg/
川平友規著『入門複素関数』を読んでいます。

E を 複素平面内のコンパクト集合とする。
E_r を E から r 以下の距離にある点全体の集合とする。

このとき、 E_r がコンパクト集合であることの証明を以下のように書いています。


E はコンパクト集合(すなわち、有界な閉集合)なので、十分に大きな R > 0 を選んで
E ⊂ D(0, R) とできる。任意の正の数 r > 0 に対し E_r ⊂ D(0, R + r) であるから、
E_r は有界である。また、 E_r の補集合は E 上の各点からの距離が r より真に大きな点
からなる集合であり、開集合となる。すなわち、 E_r は閉集合。よって、コンパクト集合
である。



「E_r の補集合は E 上の各点からの距離が r より真に大きな点からなる集合であり、開集合となる。」

↑これは自明じゃないですよね?
0183132人目の素数さん
垢版 |
2019/09/27(金) 21:13:50.22ID:N/cfTNg/
a ∈ C とする。
関数 f : C ∋ x → |x - a| ∈ R は、連続関数である。

証明:

x_0 ∈ C とする。

f(x) - f(x_0) = |x - a| - |x_0 - a| ≦ |x - x_0|
f(x_0) - f(x) = |x_0 - a| - |x - a| ≦ |x_0 - x|

∴ |f(x) - f(x_0)| ≦ |x - x_0|

任意の正の実数 ε に対して、 δ = ε とすれば、

|x - x_0| < δ ⇒ |f(x) - f(x_0)| ≦ |x - x_0| < δ = ε

が成り立つから、 f は連続関数である。

a ∈ C とする。
関数 g : E ∋ x → |x - a| ∈ R は、コンパクト集合 E 上の連続関数である。

よって、 g は E 上で最大値・最小値をとる。

110 自分:132人目の素数さん[] 投稿日:2019/09/27(金) 19:48:42.53 ID:N/cfTNg/ [4/7]
x ∈ C とする。

dist(x, E) := min {|x - y| | y ∈ E}

と定義する。
0184132人目の素数さん
垢版 |
2019/09/27(金) 21:14:18.40ID:N/cfTNg/
C ∋ x → dist(x, E) ∈ R は連続関数である。

証明:

x, x_0 を任意の複素数とする。

任意の y ∈ E に対して、

dist(x, E) ≦ |x - y| ≦ |x - x_0| + |x_0 - y|

が成り立つ。

y_0 を

dist(x_0, E) = |x_0 - y_0|

を成り立させる E の元とする。

↑の不等式から、

dist(x, E) ≦ |x - y| ≦ |x - x_0| + |x_0 - y_0| = |x - x_0| + dist(x_0, E)

∴ dist(x, E) - dist(x_0, E) ≦ |x - x_0|

x と x_0 は任意だったから、

dist(x_0, E) - dist(x, E) ≦ |x - x_0|

も成り立つ。

∴ |dist(x, E) - dist(x_0, E)| ≦ |x - x_0|

任意の正の実数 ε に対して、 δ = ε とすれば、

|x - x_0| < δ ⇒|dist(x, E) - dist(x_0, E)| ≦ |x - x_0| < δ = ε

が成り立つから、 C ∋ x → dist(x, E) ∈ R は連続関数である。
0185132人目の素数さん
垢版 |
2019/09/27(金) 21:14:34.45ID:N/cfTNg/
E_r^C ∋ x_0 とする。

dist(x_0, E) > r

である。

C ∋ x → dist(x, E) ∈ R は連続関数であるから、

ε := dist(x_0, E) - r とおくと、

|x - x_0 | < δ ⇒ |dist(x, E) - dist(x_0, E)| < ε

を成り立たせるような正の実数 δ が存在する。

したがって、

|x - x_0 | < δ ⇒ dist(x_0, E) - dist(x, E) ≦ |dist(x, E) - dist(x_0, E)| < ε = dist(x_0, E) - r

が成り立つ。

|x - x_0 | < δ ⇒ r < dist(x, E)

が成り立つ。

∴ |x - x_0 | < δ ⇒ x ∈ E_r^C

よって、 x_0 は E_r^C の内点である。

以上より、

「E_r の補集合は E 上の各点からの距離が r より真に大きな点からなる集合であり、開集合となる。」

が証明された。
0186132人目の素数さん
垢版 |
2019/09/27(金) 23:22:32.63ID:yct95A6e
いやあ自明だと思うけどね
補集合の1点取ってきたらeとの距離はrより真に大きいんだから
その値とrとのギャップで円の半径決めれば明らかに分離できるでしょ
0187132人目の素数さん
垢版 |
2019/09/27(金) 23:24:41.07ID:yct95A6e
真に大きい事も閉集合なんだから明らかだし
0188132人目の素数さん
垢版 |
2019/09/27(金) 23:32:03.93ID:yct95A6e
慣れてれば明らかだけどまあ慣れてないならしっかり行間は埋めて
られるようにしておいた方がいいね
形式的にしっかり書くならそうなるだろう
0189132人目の素数さん
垢版 |
2019/09/28(土) 00:25:56.47ID:AcpNtBWc
x[n]∈E_r、lim x[n]=xとする。
y[n]∈Eをd(x[n],y[n])≦rととれる。
Eはコンパクトだから(必要なら部分列をとって)lim y[n]=y∈Eとしてよい。
この時d(x,y)≦r。
0190132人目の素数さん
垢版 |
2019/09/29(日) 20:58:00.20ID:rVYV+GdK
5800
かずきち@dy_dt_dt_dx 9月29日
京大オープン経済190/550しか取ってないやつにマウント取られて草
お前より90点高いんだよ黙って勉強しろ
https://twitter.com/dy_dt_dt_dx
https://twitter.com/5chan_nel (5ch newer account)
0191132人目の素数さん
垢版 |
2019/09/29(日) 23:34:55.48ID:nh4sklf7
マラソンについて質問です。

マラソンをテレビで見ていると、明らかに駆け引きが存在することが分かります。

解説者も駆け引きについて説明したりします。

サッカーのような競技とは違い、普通に考えれば、他の選手のことなど一切考えずに、
ゴールするまでのタイムが最小になるようにするにはどういうペースで走ればいいかのみ
を考えて走るのが最適な戦術であるように思います。

ところが、実際には相手の走り方に影響を受けて、自分の走り方を決めているように見えます。

これについて何か合理的な説明は可能でしょうか?
0192132人目の素数さん
垢版 |
2019/09/30(月) 08:29:37.54ID:75JdTEOX
終盤で優勝争いになればしかたないけど・・・
マイペースで走って無駄なスパートとかしないのが得策ぢゃね?
0193132人目の素数さん
垢版 |
2019/09/30(月) 11:53:40.41ID:blV8Xzc1
fがU⊂R^n上で一様連続であるとき単位法線ベクトルnに対して
∫[U] f_(x_i)(x) dx = ∫[∂U] f(x)n_i dS
を示して下さい
0195132人目の素数さん
垢版 |
2019/09/30(月) 13:28:25.07ID:2r+f+ssO
>>191
ついて行くのは簡単→ピッチ合わせて呼吸合わせるだけ
ピッチを作るのは他の選手を上回るだけの体力が必要

車で例えるなら100キロの車を追い抜くのに最低時速110キロで車間もわからないが大体6秒必要。それが人間の場合体力となる。
0197132人目の素数さん
垢版 |
2019/10/03(木) 16:07:17.47ID:n6mb43El
https://imgur.com/B4peVPp.jpg

川平友規著『入門複素関数』を読んでいます。

問題:

領域 D 内に任意の円板 E をえらび、それを「割った卵」に見立てて、図の左側のように「黄身」と「白身」に塗り分ける。
このとき、 D 上の定数関数ではない正則関数 f(z) による E の像は、決して図の右側のようにならない。すなわち、「黄身」が「白身」よりも外側に飛び出すことはない。

その理由を説明せよ。

解答:

もしそのように「黄身」が飛び出したと仮定すると、適当な1次関数 g(z) = exp(i*θ) * z + B(回転と平行移動)を用いて、
g(f(z)) が「黄身」の部分で最大絶対値をとるようにできるが、 g(f(z)) は正則であり、「黄身」の部分に E の境界点はないので、定理5.11に矛盾。
0198132人目の素数さん
垢版 |
2019/10/03(木) 16:09:01.86ID:n6mb43El
g(z) = exp(i*θ) * z + B = exp(i*θ) * (z + exp(-i*θ) * B)

|g(f(z))| = |exp(i*θ) * (f(z) + exp(-i*θ) * B)| = |f(z) + exp(-i*θ) * B|

だから、証明に用いている g(z) は、よりシンプルな g(z) = z + B(平行移動) でOKですよね。
0199132人目の素数さん
垢版 |
2019/10/03(木) 16:10:11.74ID:n6mb43El
>>197

この問題の解答ですが、もっと大きな問題があります。

「黄身の像の部分で最大絶対値を取るように平行移動できるとは限らない」

という指摘がありました。

確かにそうだと思います。
0200132人目の素数さん
垢版 |
2019/10/03(木) 16:11:10.78ID:n6mb43El
そこで、質問ですが、実際に、「黄身は白身の外に出ない」というのは成り立ちますか?成り立ちませんか?
0201132人目の素数さん
垢版 |
2019/10/03(木) 16:11:54.52ID:n6mb43El
「黄身が白身の外に出る」の定義は何でしょうか?
0202132人目の素数さん
垢版 |
2019/10/03(木) 19:22:53.19ID:n6mb43El
川平友規著『入門複素関数』を読んでいます。


関数 f(z) が D 上で有理型もしくは D 上の有理型関数であるとは、

・ D 内の点の集合 P := {α_1, α_2, … } が存在して、 f(z) は D - P 上で正則、かつ

・ 各 α_k (k = 1, 2, …) はそれぞれ f(z) の極

であることをいう。


と書いてあります。その下の「注意!」として、


P 自体は無限個の点を含んでもよいが、 D 内には集積点をもたない(もし集積点があれば、それは ∂D に属する)。


と書いてあります。

D 内に P の集積点がない理由は、以下でOKですか?

・P は孤立点からなる集合だから、 P の元は、 P の集積点ではない。

・「D - P 上で正則」だから、当然、 D - P は開集合でなければならない。D - P が開集合であれば、明らかに、 D - P の元は P の集積点ではない。
0203132人目の素数さん
垢版 |
2019/10/03(木) 21:11:08.52ID:n6mb43El
>>202

川平友規著『入門複素関数』を読んでいます。

有理型関数について質問です。

f(z) = sin(z) / z

は C - {0} で正則です。

ですが、 z = 0 は f(z) の極ではありません。

川平さんの本の定義では、 D 上の正則関数も有理型関数になります。

f(z) は、 f(0) := 1 と定義すれば、 C 上の正則関数になります。

f(z) は C 上の有理型関数ですか?
0205132人目の素数さん
垢版 |
2019/10/04(金) 16:21:51.52ID:lT/07YIu
極でないなら定義から外れるだけでは
0207132人目の素数さん
垢版 |
2019/10/11(金) 21:36:03.00ID:woYId+3K
松坂和夫著『解析入門中』を読んでいます。


A が距離空間 X の開集合であるとき、

A の各点 a に対して B(a ; r(a)) ⊂ A となる正の実数 r(a) が存在し、明らかに

A = ∪_{a ∈ A} B(a ; r(a))

となる。


という記述があります。

これって選出公理を使っていますよね。

それにもかかわらず、選出公理を使っていることを書いていません。

これはOKなんですか?
0210132人目の素数さん
垢版 |
2019/10/12(土) 12:41:05.57ID:hX/F/mRq
選出公理を使っているかいないかというのはどういう風に考えればいいんですか?


A ∋ a に対して、有界で連結な実数の集合 S_a が対応するとき、

a → (1/2) * (sup S_a + inf S_a) ∈ S_a

という写像が存在します。

A から ∪_{a ∈ A} S_a への写像 r で r(a) ∈ S_a となるものが存在することをいうのに選出公理は不要です。


A ∋ a に対して、有界な実数の集合 S_a が対応するとき、

A から ∪_{a ∈ A} S_a への写像 r で r(a) ∈ S_a となるものが存在することをいうためには、
選出公理は必要ですよね?


S_a に対する条件が空でないというだけの場合には選出公理を使わなくてはならない。

S_a に対する条件が強くなると選出公理を使わなくてもいいケースが増えてくる。

みたいなイメージですか?
0211132人目の素数さん
垢版 |
2019/10/12(土) 13:23:58.61ID:hX/F/mRq
(1)
S が空でない集合であるとき、 r ∈ S を取ることができる。

(2)
任意の A ∋ a に対して、 S_a ≠ φ とする。
任意の A ∋ a に対して、 r(a) ∈ S_a を取ることができる。


(2)ではなぜ選出公理が必要なのでしょうか?
0212132人目の素数さん
垢版 |
2019/10/12(土) 13:28:44.48ID:LBq3GV/u
写像の存在が言えないからだよ
∀a∃rが真でも∃r∀aが真とは限らない
0213132人目の素数さん
垢版 |
2019/10/12(土) 13:31:18.29ID:ECN1Py89
Aが有限なら明らかですよね
無限でも明らかだと思うんですけどぶっちゃけ

選択公理ってバナッハタルスキーのパラドックスとかそういうパラドックスを認めますよっていう意思表示だと思ってました
0214132人目の素数さん
垢版 |
2019/10/12(土) 13:45:19.24ID:jRtzBOcx
k×1000円払うとk%の確率で当たるクジがある
このクジを最も少ない金額で当てたい場合、どのようにクジを購入するのがよいだろうか
(100000円払えば確実に当たるが、それが答えではない)

色々考えたけど分からないのでお願いします
0215132人目の素数さん
垢版 |
2019/10/12(土) 14:00:19.40ID:v2xrsQvh
明らかだと大半の人間が思うがしかし他の公理からどうやっても数学的操作で示せないから公理なんだよ
0216132人目の素数さん
垢版 |
2019/10/12(土) 14:01:27.08ID:ECN1Py89
有限の場合はちゃんと示せるんですか?
0217132人目の素数さん
垢版 |
2019/10/12(土) 14:24:26.81ID:nQB1Tltw
選択公理がホントに必要である理由の説明は専門に勉強した人でないと難しいよ。
オレも数学科卒だけどできない。(お前がバカなだけと言われるかもしれないが)
自分にはこの命題選択公理なしで証明できないというのはその命題示すのに選択公理が必要である事の証明にはならない。
証明するためには選択公理が成立しないモデルでその命題が成立しないものを実際に作ってみるしかない。
しかし、その手の事が実際できるのは基礎論専攻した人間でないと無理だと思う。
0218132人目の素数さん
垢版 |
2019/10/12(土) 14:26:45.77ID:ECN1Py89
有限の場合にどうやって選択写像を具体的に構成するかがわかれば理由もわかると思います
教えてください
0219132人目の素数さん
垢版 |
2019/10/12(土) 14:47:49.89ID:Ty9mG3gK
空集合を含まない有限個の集合の列X[i] (1≦i≦n)がある時
ΠX[i]={s={<x,i>|1≦i≦n, x[i]∈X[i]} | ∀i∃!x <x,i>∈s}
は空集合ではない。
∵) nについての帰納法。n=1のとき略。
n=kのとき正しいとしてn=k+1のときを考える。
空集合を含まない有限個の集合の列X[i] (1≦i≦n)がある時
Π[1≦i≦k]X[i]は空集合でないからその元tが取れる。
一方で仮定よりx∈X[k+1]が取れる。そこで
s=t ∪ {<x,k+1>}とおけばs∈ Π[1≦i≦k+1]X[i]である。
0220132人目の素数さん
垢版 |
2019/10/12(土) 15:00:49.05ID:ECN1Py89
つまり∪で足していく操作が有限回しかできないというのが本質な訳ですね

閉集合の∪を有限回とると閉集合だけど、無限回とったら開集合になるかもしれないみたいな感じでしょうかね
0222132人目の素数さん
垢版 |
2019/10/13(日) 18:22:02.64ID:O7sZwhdv
松坂和夫著『解析入門中』を読んでいます。

以下の命題の証明で、「選出公理」を使っていますよね?


A を距離空間 X の部分集合、 a を X の点とする。 a が A の触点であるためには、 lim_{n → ∞} x_n = a となる
ような A の点列 (x_n) の存在することが必要かつ十分である。

証明

a ∈ 「A の閉包」ならば、任意の n = 1, 2, 3, … に対して

B(a ; 1/n) ∩ A ≠ φ

である。そこで B(a ; 1/n) ∩ A から点 x_n をとれば、 (x_n) は A の点列で、
d(x_n, a) < 1/n であるから、 x_n → a となる。
0224132人目の素数さん
垢版 |
2019/10/14(月) 01:29:44.90ID:JQb+gLUh
それでは卵の話で・・・・
混ざらない金属をいっしょに溶融して粉末化すると、卵型コア構造になるらしい・・・・

電気製鋼, 第74巻, 4号, p.221-226 (2003/Oct)
「論説 > 液相2相分離型合金粉末に形成される卵型コア構造組織」
 "Egg-type core structure formed in atomized powderes of immiscible ally systems"
 王、劉、大沼、貝沼、石田(東北大・工)
http://www.jstage.jst.go.jp/article/denkiseiko/74/4/74_4_221/_pdf

53rd 日本金属学会 金属組織写真賞 (2003/Mar)
佳作賞 B部門
「液相2相分離型合金で形成される卵型コア構造粒子の組織」
 受賞者: 王、劉、大沼、貝沼、石田(東北大・工)、大河内、小川、清水(大同特殊鋼)

ASM(アメリカ金属協会)国際写真コンテスト・貢献賞 (2003/Oct)
 "Egg-type core microstructure of immiscible alloy powders"
 受賞者: 王、劉、大沼、貝沼、石田(Tohoku Univ.)

銅と銅合金, 41(1), p.176 (2002)
 王、蒋、劉、大沼、貝沼、石田(東北大・工)
0225132人目の素数さん
垢版 |
2019/10/14(月) 10:57:02.78ID:7m/m3h4y
松坂和夫著『解析入門(中)』を読んでいます。

以下の定義ですが、 ε は任意の正の実数ですが、ある正の実数ではなぜいけないのでしょうか?
ある正の実数 ε に対して、半径 ε の有限個の開球から成る被覆をもてば、任意の ε に対しても
半径 ε の有限個の開球から成る被覆をもつように思います。


定義:

距離空間 X の部分集合 A は、任意の ε > 0 に対し、半径 ε の有限個の開球から成る被覆をもつとき、
全有界またはプレコンパクト(precompact)とよばれる。
0227132人目の素数さん
垢版 |
2019/10/14(月) 11:01:13.67ID:7m/m3h4y
あ、一般の距離空間では成り立ちませんね。

ユークリッド空間の場合にはどうですか?
0229132人目の素数さん
垢版 |
2019/10/15(火) 10:22:17.83ID:esVivUyK
松坂和夫著『解析入門(中)』を読んでいます。

距離空間 X が完備かつ全有界 ⇒ X はコンパクト

の証明ですが、おかしなことを書いています。

背理法で証明しているのですが、

「(U_λ) λ∈ Λ を X の任意の被覆とする。 X が U_λ のうちの有限個では決して被覆されないと仮定して矛盾を導こう。」

などと書かれています。

これはまずいですよね。

「X の任意の被覆 (U_λ) λ∈ Λ に対して、 X が U_λ のうちの有限個では決して被覆されないと仮定して矛盾を導こう。」

とも解釈できますよね。

「(U_λ) λ∈ Λ を X のある被覆とする。 X が U_λ のうちの有限個では決して被覆されないと仮定して矛盾を導こう。」

と書くべきですよね。
0231132人目の素数さん
垢版 |
2019/10/15(火) 11:55:05.52ID:imnYaC8C
どう読んでも間違った命題になるならともかく
普通に背理法の設定としてごくごく自然に正しい読み方ができるんだから問題ない
0232132人目の素数さん
垢版 |
2019/10/25(金) 17:28:43.75ID:X8B2Tg+D
松坂和夫著『解析入門(中)』を読んでいます。

以下の事実が証明抜きで使われています。

D_2 Φ および D_3 Φ が連続であることは分かります。
D_1 Φ が連続であることはどうやって証明するのでしょうか?


I を R の区間とする。
f : [a, b] × I → R とする。
D_2 f が [a, b] × I で存在し、連続であるとする。

Φ : I × [a, b] × [a, b] → R を Φ(y, u, v) := ∫_{u}^{v} f(x, y) dx で定義する。

Φ は C^1 級関数である。
0233132人目の素数さん
垢版 |
2019/10/25(金) 19:08:25.51ID:X8B2Tg+D
他の本(英語の教科書)やWikipediaも見てみたのですが、 Φ が C^1 であることには触れずに、

d/dy Φ(y, u(y), v(y)) を計算するのに、チェインルールを使っています。
0235132人目の素数さん
垢版 |
2019/10/29(火) 03:52:05.02ID:YMQU3aoz
問題
関数y=3^xのグラフをx軸方向にa,y軸方向にbだけ平行移動したところ、関数y=5*3^x+4のグラフになりました。このとき、定数a,bの値を求めなさい。

解答
平行移動後のグラフは
y=3^{x-a}+bとなり、
両辺に15を掛けて
15*3^{x-a}+15b=75*3^x+60
3^{x+1}*5*3^{-a}+15b=3^{x+1}*25+60
となり、両辺を比較して
5*3^{-a}=25,15b=60
3^{-a}=5,b=4
より、a=-log_3 5,b=4
としました。

合ってますか?
0236132人目の素数さん
垢版 |
2019/10/29(火) 06:34:53.81ID:wYFR2GdZ
合ってる。

y = f(x) のグラフを (a,b) だけ平行移動すると、 y = f(x-a) + b.

題意より 3^(x-a) + b = 5・3^x + 4.

x → -∞ を考えると、b=4
0237132人目の素数さん
垢版 |
2019/10/29(火) 08:03:02.86ID:+MmhDokY
>>235
マルチするな
しかも両辺に15掛けるとか意味不明
0239132人目の素数さん
垢版 |
2019/10/31(木) 20:17:57.32ID:IEA0zJ1O
断面が辺の長さ1の正三角形の円錐がある
正三角形の底辺側の1頂点から対辺に垂線を下ろし、
それを軸として円錐を回転させたときの体積を求めよ
円錐の中身が詰まった場合と
円錐の中身がなく側面だけで構成される場合の
2通り答えよ
0240132人目の素数さん
垢版 |
2019/11/01(金) 00:03:19.66ID:3SLkMpha
>>239
>断面
どんな断面よ
0241132人目の素数さん
垢版 |
2019/11/01(金) 00:03:33.74ID:3SLkMpha
>>239
>円錐
直円錐?
0242132人目の素数さん
垢版 |
2019/11/01(金) 00:04:24.83ID:3SLkMpha
>>239
>正三角形の底辺
正三角形の底辺とは?
0243132人目の素数さん
垢版 |
2019/11/01(金) 00:09:58.62ID:/vlfuSOi
頂点と底面の中心通る平面で切るんだろな。
解く気はサラサラないけど。
0244132人目の素数さん
垢版 |
2019/11/01(金) 01:36:03.35ID:bZF8kUUR
>>239
円錐
 xx + yy - (1/2 - z/√3)^2 = 0,
 (xx+yy≦1/4, 0≦z/√3≦1/2)
垂線
 (1/2, 0, 0) と (-1/4, 0, (√3)/4) をとおる直線だから
 z = (1/2 - x)/√3, y = 0,
0246132人目の素数さん
垢版 |
2019/11/01(金) 02:24:52.27ID:oJwbvalk
まぁ回転軸に垂直な平面はすべて母線に平行なので出てくる曲線は全部放物線だから解けるだろうけど、面白くも何ともないからなぁ。
0247132人目の素数さん
垢版 |
2019/11/01(金) 03:15:59.63ID:5ViU/3gF
ちょっと訂正

頂点を含む断面が辺の長さ1の正三角形の円錐がある
正三角形の底辺側の1頂点から対辺に垂線を下ろし、
それを軸として円錐を回転させたときの体積を求めよ
円錐の中身が詰まった場合と
円錐の中身がなく側面だけで構成される場合の
2通り答えよ
0250132人目の素数さん
垢版 |
2019/11/01(金) 04:00:30.71ID:5ViU/3gF
俺は円錐をこう設定したよ
ParametricPlot3D[{r*Sin[t]/2, -r*Cos[t]/4 + 3 r/4 - 1/2,
Sqrt[3]*r*Cos[t]/4 + Sqrt[3]*r/4}, {r, 0, 1}, {t, 0, 2*Pi},
ViewPoint -> All, PlotRange -> All, AxesLabel -> {"x", "y", "z"},
Mesh -> False, Axes -> None, Boxed -> False]
0251132人目の素数さん
垢版 |
2019/11/01(金) 04:03:30.19ID:5ViU/3gF
軸も含めるとこんな感じかな
ParametricPlot3D[{r*Sin[t]/2, -r*Cos[t]/4 + 3 r/4 - 1/2,
Sqrt[3]*r*Cos[t]/4 + Sqrt[3]*r/4}, {r, 0, 1}, {t, 0, 2*Pi},
ViewPoint -> All, PlotRange -> All, AxesOrigin -> {0, 0, 0},
Mesh -> False, Boxed -> False]
0254132人目の素数さん
垢版 |
2019/11/01(金) 04:19:08.37ID:5ViU/3gF
>>251のように座標をとれば
断面の縁は
y=√3/(2h) x^2 + h/√3 -1/2,
z=h,
0 \le h \le √3/2
と決まる
0255132人目の素数さん
垢版 |
2019/11/01(金) 04:21:07.65ID:5ViU/3gF
>>253
解けもせずに寝言だけほざくな低脳
えっらそうにほざくな見本解答を見せてみろゴミ野郎
0258132人目の素数さん
垢版 |
2019/11/01(金) 04:31:16.73ID:7ku5RLsx
自分で解けないから投稿したんだろ
何でバカが威張ってるんだ
0261132人目の素数さん
垢版 |
2019/11/01(金) 04:51:50.64ID:XPxKlycT
>>243
底面の中心とは限らんよ
そんな指定はないからな
0262132人目の素数さん
垢版 |
2019/11/01(金) 05:09:35.82ID:5ViU/3gF
頂点と母線を含む断面が辺の長さ1の正三角形の円錐がある
正三角形の底辺側の1頂点から対辺に垂線を下ろし、
それを軸として円錐を回転させたときの体積を求めよ
円錐の中身が詰まった場合と
円錐の中身がなく側面だけで構成される場合の
2通り答えよ
0263132人目の素数さん
垢版 |
2019/11/01(金) 06:58:44.07ID:bZF8kUUR
>>249 >>252

x = (1/2)(X -1/2) + (√3)/2・Z,
y = Y,
z/√3 = 1/2 + (1/2)(X -1/2) - Z/(2√3),
とおく。
円錐面から
 0 ≧ xx + yy - (z/√3 -1/2)^2
  = (2/√3)Z(X-1/2) + (2/3)ZZ + YY,  (放物線)
底面から
 0 ≦ z = (√3)/2・(X +1/2 -Z/√3),  (直線)
よって Z断面は
 Z/√3 - 1/2 ≦ X ≦ 1/2 - Z/√3 - (√3)/(2Z)・YY,
0264132人目の素数さん
垢版 |
2019/11/01(金) 07:16:06.31ID:3SLkMpha
>>262
>頂点と母線を含む断面
だから頂点から斜めに切ってもそれなんだって
分かってないな
母線2つ任意に取ってそれを2辺とする2等辺三角形が正三角形である円錐が無数に考えられるだろ?
0265132人目の素数さん
垢版 |
2019/11/01(金) 07:17:36.54ID:3SLkMpha
>>262
>正三角形の底辺
正三角形には底辺はない
というか
どんな三角形でもどの辺を底辺と考えてもいい
0266132人目の素数さん
垢版 |
2019/11/01(金) 07:21:50.77ID:3SLkMpha
>>262
>円錐の中身がなく側面だけで構成
側面に底面が含まれるかどうか紛らわしい
底面の円は含まれないあるいは含まれると明確に書くべき
0267132人目の素数さん
垢版 |
2019/11/01(金) 08:01:40.50ID:6+kfq8hz
どういう文章にすれば数学の問題になりますかという質問だったかww
0268132人目の素数さん
垢版 |
2019/11/01(金) 08:54:26.01ID:7ku5RLsx
ID:5ViU/3gFは只のガイジ

前にもこんな奴いたな
塾の講師になるしかないって言ってたバカが
同一人物か?
0269132人目の素数さん
垢版 |
2019/11/01(金) 12:04:03.79ID:bZF8kUUR
>>263 と Z≧0 より
 | X - Z/√3 | ≦ 1/2,

中実の場合は、Z軸からの距離の最大値Rを求めればよい。
XX + YY ≦ XX + (2/√3)Z(1/2 -X) -(2/3)ZZ  (円錐面)
 = (X -Z/√3)^2 +Z/√3 -ZZ
 ≦ 1/4 +Z/√3 -ZZ      (← z=0)
 = RR
等号成立は
 (X, Y) = ( Z/√3 - 1/2, 2√{(1/3)Z[(√3)/2 -Z]} )
 すなわち底円上の点 xx+yy = 1/4, z=0,

中実の場合の体積は
V = π∫[0,(√3)/2] RR dZ
 = π∫[0,(√3)/2] (1/4 + Z/√3 - ZZ) dZ
 = π[ Z/4 + ZZ/(2√3) - (Z^3)/3 ](0→(√3)/2)
 = π(√3)/8
 = 0.68017476
これは円錐の体積 π/(8√3) の3倍である。 
0270132人目の素数さん
垢版 |
2019/11/01(金) 12:19:44.21ID:bZF8kUUR
中空の場合は、Z軸からの距離の最小値rも求めねば・・・・
XX + YY ≧ ・・・・・ = rr,

中空の場合の体積は
V = π∫[0,(√3)/2] (RR-rr) dZ
  = ・・・・・
0272132人目の素数さん
垢版 |
2019/11/01(金) 12:44:09.20ID:bZF8kUUR
>>264

例1
頂点が (0, 0, √(2/3)) にあり 底円が xx+yy = 1/3 である円錐。
辺の長さ1の正四面体に外接する。

例2
頂点が (0, 0, √(1/2)) にあり 底円が xx+yy = 1/2 である円錐。
辺の長さ1の正八面体に外接する。


正20面体に外接する円錐も可能と思われ・・・
0273132人目の素数さん
垢版 |
2019/11/01(金) 14:05:05.43ID:bZF8kUUR
例3
頂点が (0, 0, (2/√5)sin(π/5)) = (0, 0, √{(√5 -1)/(2√5)} ) = (0, 0, 0.52573111)
にあり、底円が xx+yy = (5+√5)/10 = 0.72360680 の円錐。
辺の長さ1の正20面体に外接する。
0274132人目の素数さん
垢版 |
2019/11/02(土) 02:59:17.56ID:3PnzmJS5
>>263 より
 - (1/2 - Z/√3) ≦ X ≦ 1/2 - Z/√3,

中空の場合は
 XX + YY = XX + (2/√3)(1/2 -X) -(2/3)ZZ  (円錐面)
  = (X - Z/√3)^2 +Z/√3 -ZZ
・0 ≦ Z < (√3)/4 のとき
 XX + YY ≧ Z/√3 - ZZ = rr
  等号は (X,Y) = (Z/√3, √{(Z/√3)(1-4Z/√3)} )

・(√3)/4 ≦ Z ≦ (√3)/2 のとき
 XX + YY ≧ (1/2 -Z/√3)^2 = rr
  等号は (X,Y) = (±(1/2 -Z/√3), 0) のとき

中空の場合の体積は
V = π∫[0, (√3)/2] (RR-rr) dZ
 = π∫[0, (√3)/4] (1/4) dZ
 + π∫[(√3)/4, (√3)/2] {(2/√3)Z - (4/3)ZZ} dZ
 = π{(√3)/16 + 1/(8√3)}
 = π{5/(16√3)}
 = 0.56681230
これは円錐の体積の 2.5倍
0275132人目の素数さん
垢版 |
2019/11/02(土) 17:26:03.01ID:SzTOIpTt
答えを知りたいというより解き方の質問なんだが、

ある映画の試写会を行い、満足度のアンケート調査を行った。試写会に参加したのは300人でそのうち女性が180人であり、満足したと回答したのは男性の50%、女性の75%であった。この映画を見て満足しなかったと答えた人が女性である確率はいくらか。

この問題、回答例ではベイズの定理使って説いてたんだけど、普通に条件付き確率の式でも解けるよね?
わざわざベイズの定理を使って解かなきゃいけない理由とか、ベイズの定理を使うポジティブな理由があるなら教えてほしい
ちなみに答えは42.9
0276132人目の素数さん
垢版 |
2019/11/02(土) 17:44:58.54ID:x3xfX9R9
満足しなかった男性の数 120×50÷100=60
満足しなかった女性の数 180×25÷100=45

満足しなかったと答えた人が女性である確率
45÷(60+45)=3÷7×100=300/7≒42.9
0277132人目の素数さん
垢版 |
2019/11/02(土) 17:58:31.29ID:SzTOIpTt
>>276
そうそれで解けるよねって話
この問題の解説にベイズの定理が使われてたんだけど、その方が都合いいことってあるのかなって
0278132人目の素数さん
垢版 |
2019/11/02(土) 18:16:55.51ID:fOKcFE1D
>>275
高校数学について解説してあるサイト
mathtrain.jp

の記事から引用
↓↓↓
「ちなみに以下の問題をベイズの定理と応用例として紹介しているサイトが複数ありましたが,単純に条件付き確率の問題です。わざわざベイズの定理を持ち出す必要はありません。」

と書いてあり、早稲田大学の過去問が紹介されていた。条件付き確率で解けるならそれで構わないという考えらしい
0281132人目の素数さん
垢版 |
2019/11/03(日) 09:27:22.45ID:cGhpq8uA
>>269
球の一部
 XX + YY + {Z -1/(2√3)}^2 ≦ 1/3, Z≧0

>>274
半径比が 1:2 の同心球 の一部
 1/12 ≦ XX + YY + {Z -1/(2√3)}^2 ≦ 1/3, Z≧0
から円錐 (元の円錐の半分のサイズ)
 XX + YY - (1/2 - Z/√3)^2 = 0, (√3)/4≦Z≦(√3)/2,
をくり抜いたもの。
0282132人目の素数さん
垢版 |
2019/11/03(日) 14:00:29.55ID:Dun8b0tZ
(中略)(y^2+p/2+u)^2=(y^2+p/2)^2+ 2uy^2+pu +u^2=−qy −r+(p/2)^2+ 2uy^2+pu +u^2に なる
これから 右式か yに よる 完全平方式に なるよに uを きめる
このため uわ
q^2−8u(−r+(p/2)^2+pu +u^2) = 0に
満足させるべき
( 質問)
1.としたら uを きめますか?
2. そして なぜ
"uわ
q^2−8u(−r+(p/2)^2+pu +u^2) = 0に
満足させるべき"なんですか?
0283132人目の素数さん
垢版 |
2019/11/04(月) 15:37:21.28ID:pYz20bio
空間の点P(2,4,4)を頂点とする円錐V_rは、底円の中心Oの座標が(2,4,8)、底円の半径はrである。
3点(0,0,0),(2,4,3),(2,5,5)を通る平面をHとするとき、HとV_rの共通部分の面積が1となるrの値を求めよ。
0284132人目の素数さん
垢版 |
2019/11/04(月) 15:48:05.22ID:NsUMpY8R
a,b,cを自然数として

a^a+b^b=c^cを満たすabcの組についての研究ってどの程度進んでいますか?
0285132人目の素数さん
垢版 |
2019/11/04(月) 18:07:28.69ID:YPC7KhfL
Hを集合とします。
πを全単射写像X→Yとします。
{π(π^(-1)(h))|h∈H}= Hが成立しそうに思えるのですが、例外はありますか?
{π^(-1)(π(h))|h∈H}についても同様でしょうか。
0286132人目の素数さん
垢版 |
2019/11/04(月) 18:11:33.95ID:YPC7KhfL
>>285
全射性から前半がいえて、単射性から後半が言えましたね…お目汚し失礼しました。
0287132人目の素数さん
垢版 |
2019/11/04(月) 19:46:53.41ID:+E5iDXKl
>>271
円をある斜軸の周りに回転したときの軌跡は、
それらを小円とする球面(の一部) >>281

>>284
(n,・・・・,n,2n+1) の(n+1)個で AM-GM して
  n個
 (n+1)^(n+1) > (2n+1)n^n ≧ 3n^n,
∴ Max(a,b) < c のときは a^a + b^b ≦ (2/3)c^c < c^c,
 Max(a,b) ≧ c のときは a^a + b-b >c^c
∴ 題意を満足する自然数 (a,b,c) は存在しない。
0288132人目の素数さん
垢版 |
2019/11/04(月) 21:39:55.03ID:+E5iDXKl
(n+1)^(n+1) / (n^n) = √{n(n+1)} (1+1/n)^(n+1/2)
 ≧ e√{n(n+1)}
 ≧ e√2,

* nが大きいとき  (1+1/n)^(n+1/2) = e を使った。
0289132人目の素数さん
垢版 |
2019/11/05(火) 10:28:06.66ID:Dswj27rO
〔問題2〕
 a_n = (1 +1/n)^n
 b_n = (1 +1/n)^(n+1)
 c_n = (1 +1/n)^(n+1/2)
とおくとき、nが増加すると a_n は増加し、
b_n と c_n は減少することを証明せよ。
 (数学検定 1級 2次[2]改、2011年・秋)

採点者「微分法を使うのは・・・・・本末転倒の感がある。」
0290132人目の素数さん
垢版 |
2019/11/05(火) 10:34:54.36ID:Dswj27rO
(略解)
(a)
 {1,・・・・,1,(1-1/n)} のn個でAM-GMすると
  n-1個
 (1 -1/nn)^n > 1 -1/n,
∴ a_n / a_{n-1} = (1 +1/n)^n (1 -1/n)^(n-1) > 1,

(b)
 {1,・・・・,1,n/(n-1)} のn+1個で AM-GMすると
  n個
 {nn/(nn-1)}^(n+1) > n/(n-1),
∴ b_n / b_{n-1} = (1 +1/n)^(n+1) (1 -1/n)^n < 1,

(c) 二項公式を使う。
 (1 -1/nn)^(n+1/2)
  = 1 - (n+1/2)/nn + (n+1/2)(n-1/2)/(2n^4) - ・・・・
  = 1 - 1/n - 1/(2nn) + (nn-1/4)/(2n^4) - ・・・・
  < 1 - 1/n - 1/(2nn) + 1/(2nn)
  = 1 - 1/n,
∴ c_n / c_{n-1} = (1 +1/n)^(n+1/2) (1 -1/n)^(n-1/2) < 1,
0291132人目の素数さん
垢版 |
2019/11/05(火) 19:12:10.42ID:mMkIqLn2
A可換環、S⊂Aを積閉集合、M,Mを加群とする。またSによるMの局所化をM_Sなどと書く
このときExt_(A_S) (M_S,N_S)=(Ext_A (M,N))_Sがすべての次数で成り立つ(「=」は同型)ことを示せ、という問題がわかりません

雪江の代数学3の演習問題なのですが、このTor版は証明が載っていてそれと同じように考えると
自由加群Fについて以下が示せればよいことまでは分かったのですがそこから進めず困ってます
Hom_A (F,N_S)=(Hom_A (F,N))_S
よろしくお願いします
0293132人目の素数さん
垢版 |
2019/11/05(火) 20:24:10.45ID:vZ1cdcgz
>>289
anに上界があるのは加乗律でどう示すの?
0295132人目の素数さん
垢版 |
2019/11/06(水) 00:14:01.47ID:RueoJxgJ
>>201
それMか有限生性とかなんとか条件ないと成り立たないと思う。
たとえばR=Z、MがZの可算無限直和、SがZ\{0}のとき
Ext^0(M,R)_S = {{(ai),q)| ai∈Z,q∈Q}
Ext^0(M_S,R_S) = {(bi) | bi∈Q}
で自然射は((ai),q)を(ai×q)にマップするけど、この像にはたとえばbi=1/2^iは含まれないので全射にならない。
0297132人目の素数さん
垢版 |
2019/11/06(水) 01:45:58.41ID:7g9M9/dU
>>295
ありがとうございます
その例を見ると確かに同型にならなさそうですね
条件を足して証明考えてみます
0298132人目の素数さん
垢版 |
2019/11/06(水) 08:11:50.64ID:zGZD1ohT
>>297
いや、そうじゃなくて、その問題がそもそも成立しない事を証明せよと言ってる希ガス。
答えあるならもう答えみた方がいい。
答えないなら無視していいと思う。
数学者も神様じゃないので解答不能の問題作ってしまう事なんていくらでもありうる。
0301イナ ◆/7jUdUKiSM
垢版 |
2019/11/06(水) 19:31:03.02ID:o8CuOqKa
>>119たぶん俺。
>>283
平面Hの法線ベクトルを、
(a,b,c)とおくと、
各辺のベクトル、
(2,4,3),(2,5,5),(0,1,2)との内積が0だから、
2a+4b+3c=0
2a+5b+5c=0
b+2c=0
∴b=-2c,a=5c/2
(5c/2,-2c,c)
簡単にすると、
(5,-4,2)
平面Hの方程式は、
5x-4y+2z=0
(つづく)
0302132人目の素数さん
垢版 |
2019/11/06(水) 21:09:07.58ID:FmhIIeu3
イナの安定感は異常
0303132人目の素数さん
垢版 |
2019/11/07(木) 00:05:10.95ID:VhlwZ5Jt
自然数nについての数列a[n]を
a[n]=(1+1/n)^(n+a)
により定めるとき、下記(1)(2)が実定数aによりどのようになるかを調べよ。

(1)lim[n→∞] a[n]
(2)a[n]の増減
0304132人目の素数さん
垢版 |
2019/11/07(木) 06:48:46.33ID:4wihb3iI
>>303
(1) e
(2)
a <= (log 9/8)/(log 4/3) で増加
a >= 1/2 で減少
(log 9/8)/(log 4/3) < a < 1/2 で減少のち増加
自信ない
ちなみに微分法使った汚ねぇ解き方でした
0305132人目の素数さん
垢版 |
2019/11/07(木) 08:59:02.97ID:E16c0AKx
三辺の長さがa, b, c (a,b,c は整数)で、面積が 0.5a^2 になる三角形の例はありますか。
0307132人目の素数さん
垢版 |
2019/11/07(木) 22:45:10.02ID:ShBrNZga
>>298
答えにはTorの時と同様と一行書かれてるだけなので、著者の勘違いっぽいですね
スルーしときます
他の本とか色々調べてみたら、n=0の時はMが有限表示なら成り立つ的なことが書いてありました
0308132人目の素数さん
垢版 |
2019/11/07(木) 23:19:05.41ID:VhlwZ5Jt
AB=CD=4、BC=DA=3、∠ABC=60°の平行四辺形Hを、以下のように順々に座標平面上に置いていく。
以下、平面上にk番目に置いたHをH_kと書く。

・H_1を置くとき、H_1の頂点Bが座標平面の原点Oと一致するようにする
・H_(k+1)を置くとき、H_kとH_(k+1)とで少なくとも1つの等長辺が重なるように置く。

H_nを置いたとき、H_nの4頂点のうち原点Oから最も距離が離れているものをTとする。
|OT|が最大になるようなH_1,H_2,...,H_nの置き方を述べ、それが正しいことを説明せよ。
0310132人目の素数さん
垢版 |
2019/11/08(金) 00:12:52.17ID:gbqWfBwm
>>307
>n=0の時
Homと書け
0311132人目の素数さん
垢版 |
2019/11/08(金) 00:14:49.00ID:gbqWfBwm
>>307
>Mが有限表示なら成り立つ
有限生成だけでは良くないの?
>著者の勘違いっぽいで
どうかな?そもそも有限生成とか有限表示とかだけに限定していたりしない?
0312132人目の素数さん
垢版 |
2019/11/08(金) 01:49:57.47ID:/mMatjA+
>>311
もう一回確認したけど限定はされてませんでした

自分が見たのは有限表示なら十分であるという証明なので、有限生成だけでは言えないかどうかは不明です
まず有限表示でない有限生成加群の具体例が思い浮かばない…
(ちなみに証明見たのは桂「代数学2」のp.69です)
0313132人目の素数さん
垢版 |
2019/11/08(金) 02:15:14.77ID:RtlcYsMo
有限生成でもいけます。
Hom(X,Y)_S → Hom(X_S,Y_S)
はX=Rのとき同型で、Xが自由加群の時単射です。
よってXが有限個のRの直和の時同型。
一般の有限生成のときには右完全列
F1 → F2 → X → 0
をF2がRの有限個の直和、F1が自由加群にとれます。
ここにHom(-,Y)_S→Hom(-,Y_S)をヒットしてできる可換図式をよく見れば分かります。
0314132人目の素数さん
垢版 |
2019/11/08(金) 04:16:40.31ID:j9T478d/
https://w.atwiki.jp/omoshiro2ch/pages/49.html
ここの445をだれか解説お願いします…。
二列目の削減もやってみましたが証明できません。
あと、457がいうように「さらに小さな領域」でも無理とわかるようですが、「さらに小さな領域」のマス数の最小値はいくつなのでしょうか。
0315132人目の素数さん
垢版 |
2019/11/08(金) 19:51:47.26ID:hkWkuifK
三山ゲームの必勝法に出てくる排他的論理和で
 (a,b,c) を a,b,c は自然数で二進数で表示したときに各桁の1の個数の合計がすべての桁で偶数になる点とする。
格子点を空間に表示したらどんな感じに分布しているか見てみたい
0316132人目の素数さん
垢版 |
2019/11/08(金) 19:54:21.69ID:jTeJy7O9
座標平面上の3点O(0,0),A(2,0),B(1,√3)を頂点とする正三角形ABCを考える。
△ABCを直線OA,AB,BCの周りにそれぞれ一回転させて出来る立体をK_1,K_2,K_3とする。
このとき、(K_1∩K_2∩K_3)の体積を求めよ。
0318132人目の素数さん
垢版 |
2019/11/09(土) 00:12:38.00ID:yLFraa3Z
>>314
6っぽくね?4ではないし色々試したら5もなさそうだわ
証明は知らん
0319132人目の素数さん
垢版 |
2019/11/09(土) 02:03:20.54ID:3bqIbTDC
無限小数
1/7=0.142857142857...
を小数点以下第k桁目で打ち切った有限小数をa[k]とする。
例えばa[1]=0.1、a[2]=0.14、a[7]=0.1428571、である。

(1)kが6で割り切れるとする。a[k]を既約分数q/pの形で表すとき、pとqの最大公約数をkで表せ。答えのみでよい。
なおkによらない値になる場合は、その値を求めよ。

(2)一般のkに対して(1)の最大公約数をkで表し、その理由を述べよ。なおkによらない値になる場合は、その値を求めよ。
0320132人目の素数さん
垢版 |
2019/11/09(土) 02:16:57.25ID:WNgcr7q7
前なにかの本で読んだんですが
a^bc+b^ca=c^abは結構大きい自然数解を持つ というのを聞いた気がするのです
なにか知っている方はいませんか?
0323132人目の素数さん
垢版 |
2019/11/09(土) 08:48:21.61ID:Vf5Ox1c2
>>319
q/pが既約→最大公約数は1
0324132人目の素数さん
垢版 |
2019/11/09(土) 10:24:16.87ID:YvHvPTYX
どっかで見かけた定理なんですけどコンパクト凸領域Aと正の数tに対して
f(t)=vol({p | d(p,A)<t})
とおくときコレがtの多項式になって係数もわかるっていう定理なんでしたっけ?
0325132人目の素数さん
垢版 |
2019/11/09(土) 10:52:09.54ID:kBjuF2/7
x=tcosθ , y=1-t²+tsinθ
で曲線Cを定義する(tは実数全体を動く)
Cがx軸のx≥0の部分と交わる点をQとする。
θを変化させた時、Qのx座標が最大となるθについてtanθの値を求めよ。

sinθ=s、cosθ=cと書いて、
二次方程式を解いて√の入った形で表示すると
Qのx座標q=(1/2)*c*{s+√(s²+4)}
(s>0かつc>0の時を考えれば十分)
これをθで微分してみたのですが計算が下手なのかうまく行きません
どなたか解答をお願いしますm(_ _)m
0326イナ ◆/7jUdUKiSM
垢版 |
2019/11/09(土) 12:06:10.36ID:ptT1lX7e
>>301
>>283三角錘が平面で斬られて断面が欠円ていうのかな、蒲鉾の断面みたいになる気がするんだよね。
| ∩∩ ∩∩  /\
|((-_-)-_-)) / 「
|(`っΔU⌒U、//|
| ‖υυ~UU~‖ |
| ‖ □ □ ‖ |
∠‖____‖/|
 ̄ ̄ ̄ ̄ ̄ ̄‖ |
□ □ □ ‖ |
______‖/|
 ̄ ̄ ̄ ̄ ̄ ̄‖ |
□ □ □ ‖ |
______‖/|
 ̄ ̄ ̄ ̄ ̄ ̄‖ |
□ □ □ ‖ |
______‖/|
 ̄ ̄ ̄ ̄ ̄ ̄‖ |
□ □ □,彡ミ、|
_____川`,`;,'
______U⌒U、;,
/_/_/_/;_~U U~_;
/_/_/_/_○_/_
/_/_/_/_/_/_/_/_/_/_/_/
0327132人目の素数さん
垢版 |
2019/11/09(土) 12:07:15.51ID:3bqIbTDC
p_nを小さい方から数えてn番目の素数とする。
有理数1/(p_n)の循環節の長さをf(n)とおく。
このときどのようなnに対しても、
Max( f(1), f(2), ... , f(n) ) < f(N)
を満たす自然数Nが存在することを示せ。
0329132人目の素数さん
垢版 |
2019/11/09(土) 13:28:19.22ID:3bqIbTDC
p[n]を小さい方から数えてn番目の素数とするとき、以下の極限の収束、発散を調べよ。
発散する場合は、無限大に発散するか振動するかも述べること。

lim[n→∞] (p[n]+p[n+3])/(p[n+1]+p[n+2])
0330132人目の素数さん
垢版 |
2019/11/09(土) 13:52:33.05ID:YvHvPTYX
limsup log (p(n+1)-p(n))/p(n) ≦ 21/40 により

lim[n→∞] (p[n]+p[n+3])/(p[n+1]+p[n+2]) = 1。
0331132人目の素数さん
垢版 |
2019/11/09(土) 15:32:55.58ID:czrA/bm0
q[n] = p[n+1]/p[n] とおくと q[n] >1,
(与式) = (1/q[n] + q[n+1]q[n+2])/(1+q[n+1])
q[n] が上に有界とする。
 1 < q[n] < α
∴ (1/α)(1+αq[n+1])/(1+q[n+1]) < (与式) < (1+ αq[n+1])/(1+q[n+1])
∴  (1+α)/(2α) < (与式) < (1+αα)/(1+α)
0332イナ ◆/7jUdUKiSM
垢版 |
2019/11/09(土) 15:58:31.69ID:ptT1lX7e
>>326
>>283
z=4上の円(x-2)^2+(y-4)^2=r^2の平面5x-4y+8=0による切り口は線分で、
端点は円周上にあり、
y=5x/4+2を代入すると、
(x-2)^2+(5x/4-2)^2=r^2
x^2-4x+4+25x^2/16-5x+16=r^2
41x^2/16-9x+20=r^2
41x^2-144x+320-16r^2=0
x=[72±√{72^2-41(320-16r^2)}]/41
={72±√(5184-13120+656r^2)}/41
={72±√(656r^2-8064)}/41
={72±4√(41r^2-504)}/41
切り口の端の2点が確定するから、線分の長さも決まる。
z=t上の切り口の線分の長さを、
t=4から8まで足しあつめると面積が1になる式からrを決める。
0333132人目の素数さん
垢版 |
2019/11/09(土) 16:09:13.52ID:YvHvPTYX
>>329
はp(n+1)-p(n)=o(p(n))を知らないと無理だろ?
チェビシェフの評価
p(n+1)/p(n)<9/8
とかではちょっと無理。
0334132人目の素数さん
垢版 |
2019/11/09(土) 23:46:29.30ID:bT8+Krws
>>313
わかったと思ったのですが、一箇所よく分からないことに気づいたのでまた質問いいでしょうか

>Xが自由加群の時単射
この部分はどのようにしたら言えるのでしょう。X=直和Rをばらして計算すると
(ΠY)テンソルA_S → Π(YテンソルA_S)
の単射性が言えればいいですが、これが言えずに困ってます

例えばΠYの元(yi)で各yiに対してあるsi∈S⊂Aがあってsi*yi=0となるものを考えたとき
(ΠY)テンソルA_Sの元 (yi)テンソル1 は0になるのでしょうか
0336132人目の素数さん
垢版 |
2019/11/10(日) 00:23:00.77ID:NCLiZrEd
>>324
嘘定理
0337132人目の素数さん
垢版 |
2019/11/10(日) 00:58:44.33ID:KFIy1Hep
>>334
失礼しました。
反例ありますね。
R=Z、S=Z\{0}、X=Rの可算無限個の直和、Y=Q/Zのとき
Hom(X,Y) ⨂ R_S = (⨅ Q/Z )) ⨂ Q
の方は消えませんが ((1/2,1/3,1:4,‥) ⨂ 1 が0でない)
Hom(X ⨂ R_S,Y ⨂ R_S) = ⨅ (Q/Z ⨂ Q)
の方は死んじゃうので単射にならないですね。
忘れてください。
よってXが自由加群の場合ですら単射にすらならないので例の練習問題は解けないですね。
0339132人目の素数さん
垢版 |
2019/11/10(日) 01:15:10.56ID:v9KtaDvq
>>325

q(θ) = f(sinθ)・cosθの形だから、
(dq/dθ) = f(s)・(-s) + f '(s)・cc
  = f(s){(-s) + cc/√(ss+4)}
  = f(s){(-s)√(ss+4) + 1-ss}/√(ss+4)

∴ (dq/dθ) = 0 とおくと、 f(s) >0 より
 (-s)√(ss+4) + 1-ss = 0, 
 s = sinθ = 1/√6,
 tanθ = 1/√5,
 qの最大値は (1/2)√5,
0340132人目の素数さん
垢版 |
2019/11/10(日) 01:53:00.84ID:D4YmcvR8
>>337
ああなるほど、Q/Zで反例作れればよかったんですね
いろいろいじっていい訓練になりました
0341132人目の素数さん
垢版 |
2019/11/10(日) 02:24:42.46ID:r+RoVOy/
座標平面の4点O(0,0),A(1,0),B(1,1),C(0,1)を4頂点とする正方形の内部の領域をSとする。

(1)以下のような直線Lと点Hの例を1つ挙げよ。

『Sの内部を通る直線を1つ引き、それをLとする。その直線にOから垂線を下ろし、Lとの交点をHとする。
SはLと線分OHにより3つの領域に分けられるが、3つの領域それぞれの面積は全て等しい。』

(2)(1)の条件を満たす点Hの存在しうる領域を求めよ。
0342イナ ◆/7jUdUKiSM
垢版 |
2019/11/10(日) 04:56:38.06ID:R+XmyG3o
>>332
>>283
平面Hで切りとられるz=tにおける円錐内の線分yの幅はxの幅の5/4
切りとられるz=4における円錐内の線分の長さは、
(√41/4)・8√(41r^2-504)}/41=(√41)・2√(41r^2-504)}/41
=2√(r^2-504/41)
z=tにおける円錐内の線分の長さをt=4から8まで足しあつめて=1にしたい。
0344132人目の素数さん
垢版 |
2019/11/10(日) 07:58:05.44ID:SXNDd19m
>>263
>x = (1/2)(X -1/2) + (√3)/2・Z,
>y = Y,
>z/√3 = 1/2 + (1/2)(X -1/2) - Z/(2√3),
>とおく。

断面が放物面であると予め知ってるので回転させるのか?w

>中実の場合の体積は
>V = π(√3)/8

同じく

>中空の場合の体積は
>V = π{5/(16√3)}

(5π√3) /48
0345イナ ◆/7jUdUKiSM
垢版 |
2019/11/10(日) 08:38:30.13ID:R+XmyG3o
>>342
>>283
平面Hで切りとられるz=tにおける円錐内の線分の長さはxの幅の√41/4だから、
切り口の面積
=∫[t=4→8](√41/4)(8-t)r/4
=[t=4→8](√41/16)(8t-t^2/2)r
=(√41/16)(8・4-64/2+16/2)r
=(√41/16)8r
=r√41/2
=1
∴r=2/√41
=2√41/41
0346132人目の素数さん
垢版 |
2019/11/10(日) 09:24:35.47ID:v9KtaDvq
>>325 >>339
少し拡張して
 y = y。 + (sinθ)t - tt,
としてみると、
 q(θ) = f(sinθ)・cosθ,
 f(s) = (1/2){s + √(ss+4y。)},
(dq/dθ) = 0 から
 (-s)√(ss+4y。) + 1-ss = 0,
 s = sinθ = 1/√(2+4y。),
 tanθ = 1/√(1+4y。),
 qの最大値は (1/2)√(1+4y。),

なお、y。=0 なら θ=π/4 のとき qが最大 (1/2) となります。 
0347132人目の素数さん
垢版 |
2019/11/10(日) 10:41:16.18ID:v9KtaDvq
>>339 >>346

t=0,  (x, y) = (0, y。)

t=s/2, (x, y) = (sc/2, y。 + ss/4), y:最大

t=s,  (x, y) = (sc, y。)

t=f(s), (x, y) = (f(s)c, 0)
0350132人目の素数さん
垢版 |
2019/11/10(日) 17:15:58.06ID:v9KtaDvq
連立方程式
 a + b = c,
 a^a + b^c = c^b,
は自然数解をもつでしょうか?
0353132人目の素数さん
垢版 |
2019/11/10(日) 21:35:15.99ID:alonVGiM
帰ったら計算機回して解いてみるわ
ちなみに解は有限個かな?これ
0355132人目の素数さん
垢版 |
2019/11/10(日) 23:00:16.29ID:v9KtaDvq
>>316
点Cはどこにある?
K_1 は∠OBA内にあり、
K_2 は∠AOB内にあり、
K_3 は∠OAB内にある。
∴ △OAB内で考えてよい。
まづ (√3)y≦x≦1, 0≦y≦1/√3, z≧0 の部分(1/12)に注目する。
点(x,y)での高さは ±√(2y)・√{(√3)(2-x) + y}

∫[(√3)y,1] √{(√3)(2-x) + y} dx
 = [ -(2/3√3){(√3)(2-x) + y}^(3/2) ]
 = {(4√2)/(3√3)}(√3 -y)^(3/2) - {2/(3√3)}(√3 +y)^(3/2),

Vo = ∫[0,1/√3] √(2y)∫[(√3)y,1] √{(√3)(2-x) + y} dx dy
 = -(7/(9√2)) + (1/2)arccos(1/3) +log(3)/(4√2)
 = 0.2597168175505403503
全体の体積はこれの12倍だから
12Vo = 3.1166018106064842
0356132人目の素数さん
垢版 |
2019/11/10(日) 23:31:21.85ID:knnXg3Yg
>>354
計算機回したけど(a,b;c)=(1,1;2), (1,2;3)だけっぽいね
0360132人目の素数さん
垢版 |
2019/11/10(日) 23:53:24.43ID:v9KtaDvq
>>355

∫[0,1/√3] √(2y)・(√3−y)^(3/2) dy
 = 7/(36√3) + ((3√6)/16)arccos(1/3)
 = 0.677616764822162

∫[0,1/√3] √(2y)・(√3+y)^(3/2) dy
 = 77/(18√6) - ((3√6)/16)log(3)
 = 1.24182555243931
を使う。
0361132人目の素数さん
垢版 |
2019/11/11(月) 01:51:09.32ID:lEidjnT0
平面が一辺の長さ1の正方形のタイルで敷き詰められている。そのうち1つのタイルには「0」と書かれている。
これらのタイルに以下の(操作)を行う。

(操作)
・「0」または「1」が書かれたタイルに隣接するタイルで、何も書かれていないものを無作為に1つ選ぶ。
(どのタイルを選ぶかは同様に確からしい)。
・選んだタイルに1/2の確率で「0」を、1/2の確率で「1」を、それぞれ書く。

この操作を繰り返し行い、以下の(状態)になった時点で操作を終了する。

(状態)
・正方形をなすある4つのタイル(タイルが縦2つ、横2つの形で並んでいる)が存在して、左上と右下のタイルには「1」が、右上と左下のタイルには「0」が書かれている。

操作をちょうどn回繰り返したとき、操作が終了する確率を求めよ。
0362132人目の素数さん
垢版 |
2019/11/11(月) 07:39:59.36ID:IFAKo+jz
>>361
隣全部埋まってたら?
0363a4 ◆L1L.Ef50zuAv
垢版 |
2019/11/11(月) 09:43:06.99ID:bnED1m3P
P=NPの夢を見ました。数学者の神経構造を持った人工知能が巡回セールスマン問題の
配置にあって、お互いに長い手紙を書いて郵送でやり取りするんです。これなら、
トリビアルなグラフならすぐ解けるし、多項式時間になるかもしれません。
SUBSET SUMの問題に多項式時間で変換できれば、答えのチェックができるんじゃない
かと。神経細胞って何?ってことですが、海馬と前頭葉の新皮質に関しては、僕が
ソフトウェアを創りました。
http://www.01ken.com/art1.html
これで解けるんでしょうか?反証があるんでしょうか?新しいスレを立てたほうが
いいのでしょうか?よろしくお願い致します。
0364a4 ◆L1L.Ef50zuAv
垢版 |
2019/11/11(月) 09:53:46.78ID:bnED1m3P
個人的には、クイックソートが平均でO(n*log(n))というのは証明されてないのでは?
最大は、O(n^2)になるのに。
0367132人目の素数さん
垢版 |
2019/11/11(月) 13:51:00.95ID:dtL00467
xyz空間の円筒C:x^2+y^2=1,-∞<z<∞を考える。
原点Oを端点とする半直線を考え、半直線と円筒の交点をSとし、OS・SP=1なる点Pをとる。
ただしPは円筒の外側にあるものとする。
半直線が色々動くとき、点P全体がなす曲面をDとする。

(1)Dを平面x=0で切った切り口の曲線の方程式を求めよ。

(2)Dを平面x+y+z=0で切った切り口の曲線の方程式を求めよ。
0368132人目の素数さん
垢版 |
2019/11/11(月) 17:59:44.59ID:lEidjnT0
以下の3つの方程式が1つ以上の共通解を持つような実数aとbの条件を求めよ。
x^2-2ax+b=0
x^2-(2x)*x+a=0
x^2-2bx+x=0
0370132人目の素数さん
垢版 |
2019/11/11(月) 20:38:12.68ID:nKlbHIyi
当たり1本、はずれ4本の合計5本のくじが入った箱から、1本ずつ 3回くじを引く。
ただし1回引くごとに当たり外れ関係なくはずれを1本補充して、箱の中を常に5本に保った。
その結果、あたり1回、はずれ2回であった。
この時、2回目に引いたくじが当たりであった確率を求めよ。
誰かお願いします。
0371132人目の素数さん
垢版 |
2019/11/11(月) 20:42:47.78ID:IYKDpocT
すでに起きた事象なので0%か100%かどちらか
くじを引いた人に聞けばわかる
0372132人目の素数さん
垢版 |
2019/11/11(月) 20:43:29.80ID:FRx+BjCW
2回目に当たりだった確率/(1回目に当たりだった確率+2回目に当たりだった確率+3回目に当たりだった確率)
0373132人目の素数さん
垢版 |
2019/11/11(月) 21:03:05.26ID:nKlbHIyi
>>372
サンガツ、20/61やな
0374132人目の素数さん
垢版 |
2019/11/11(月) 21:52:40.67ID:uIUz6082
>>367

S (X,Y,Z) はC上の点だから XX+YY=1, OS = √(1+ZZ),
題意より SP = 1/OS = 1/√(1+ZZ),
点Pは 半直線OS の延長線上にあるとする。
 ↑OP = {1 + 1/(1+ZZ)}↑OS
P(x,y,z) とすると、
 x = {1+1/(1+ZZ)}X,
 y = {1+1/(1+ZZ)}Y,
 z = {1+1/(1+ZZ)}Z,

 r = √(xx+yy) = 1 + 1/(1+ZZ), とおくと
 ZZ = (2-r)/(r-1),

Dの方程式は
 zz = rr{(2-r)/(r-1)}, r=√(xx+yy),

(1) Dを平面 x=0 で切った切り口の曲線は
 zz = yy(2-|y|)/(|y|-1),

(2) Dを平面 z=-x-y で切った切り口の曲線は
 (-x-y)^2 = rr{(2-r)/(r-1)}, r=√(xx+yy),
0375132人目の素数さん
垢版 |
2019/11/11(月) 23:16:03.04ID:lEidjnT0
f(x)={e^(-x)}(x^2-2x*sinx+cos3x)
g(x)=(4+sinx)(1+e^(sinx))
に対し、
h(x)=f(x)/g(x)
とおく。

(1)h(x)の増減を調べよ。

(2)n=1,2,...に対して、数列h(nπ/2)の増減を調べよ。
0377132人目の素数さん
垢版 |
2019/11/12(火) 11:39:13.05ID:lwaVe1Mn
f を R^n から R への微分可能な関数とする。

超曲面 f(x_1, …, x_n) = 0 上の曲線 γ : R ⊃ I → R^n が連続であるとき、 γ は微分可能になりますか?
0379132人目の素数さん
垢版 |
2019/11/12(火) 11:59:28.31ID:lwaVe1Mn
たしかにそうですね。
0380132人目の素数さん
垢版 |
2019/11/12(火) 12:05:55.11ID:lwaVe1Mn
U : R^n の開集合
f : R^n から R への微分可能な関数
S_c := {x ∈ U | f(x) = c}

命題

ベクトル grad f(x_0) は、曲面 S_c 上にあって点 x_0 を通る任意の微分可能な曲線の点 x_0 における接ベクトルと直交する。
0381132人目の素数さん
垢版 |
2019/11/12(火) 12:09:17.03ID:tM6kTGwK
これ結局曲線 γ : R ⊃ I → R^n が連続であるとき、 γ は微分可能になるかしか聞いてないな?
0382132人目の素数さん
垢版 |
2019/11/12(火) 12:09:31.45ID:lwaVe1Mn
>>380

γ : R ⊃ I → R^n を曲線とする。
{γ(t) | t ∈ I} ∋ x_0 とする。

grad f(x_0) が {γ(t) | t ∈ I} と直交することの定義はどのように定義するのでしょうか?
0383132人目の素数さん
垢版 |
2019/11/12(火) 12:11:52.23ID:lwaVe1Mn
>>382

なんか γ が微分可能でなくても連続でありさえすれば、

grad f(x_0) が {γ(t) | t ∈ I} と直交する

ということが定義できそうな気がしたので質問しました。
0384132人目の素数さん
垢版 |
2019/11/12(火) 14:08:31.11ID:eEp/461s
曲線じゃなく集合との直交を定義すりゃいいのさ
微小近傍の任意2点を結ぶ直線と直交だね
0387132人目の素数さん
垢版 |
2019/11/12(火) 16:32:25.63ID:/fKqxMRL
nは3以上の整数とする。
x^n+2(y^n)=3(z^n)
を満たす自然数(x,y,z)の組は存在しないことを示せ。
0388132人目の素数さん
垢版 |
2019/11/12(火) 17:56:45.53ID:60GMZ1mJ
>>376
 知らねぇ。

 b(n) = log{a(n)}
とおく。 nが大きいときは
 b(n+2) ≒ b(n+1) + b(n),
から
 b(n) ≒ 0.453730157860675φ^n − 0.1273423052(-1/φ)^n,
だが・・・・
 φ = (1+√5)/2 = 1.618034   黄金比
0390132人目の素数さん
垢版 |
2019/11/12(火) 19:02:19.37ID:6/kpexpf
nを自然数とする。
xについての方程式
ax^(n+1)+bx^n+cx^(n-1)=0
が0でない実数解を持つために、実数a,b,cが満たすべき条件を述べよ。
0391442
垢版 |
2019/11/13(水) 00:24:13.35ID:06oQbgPS
>>390
x≠0 の時、与式の両辺を x^(n-1) で割ると
ax^2+bx+c=0
これが実数解を持てばよいので、
b^2-4ac>0 の時、与式は 0 でない実数解を持つ。

えーと、あとは a=0, b≠0, c≠0の時も満たすけど a=b=c=0 の時を解に含めるかどうかはよくわかんね。
0393a4 ◆L1L.Ef50zuAv
垢版 |
2019/11/13(水) 04:14:05.85ID:AfKpBTAU
>>376
量子コンピュータで忌み名を計算したら、

「nガ3ジャナイ」
=(nに3などを次々に代入していく問題ではない。|
(逆)否、野次(馬)さんがno.)
0394132人目の素数さん
垢版 |
2019/11/13(水) 16:28:21.68ID:s6ZQZRbo
四面体OABCにおいて,辺ABを2:1に内分する点をPとし,△OPCの重心をG,直線AGと
△OBCの交点をQとするとき,OQ↑を,OB↑,OC↑を用いて表せ。

OA〜AP〜PG〜GQで求めるのではないでしょうか。先生、しかし、AG:AQがわかりません。
図がないのが大変申し訳ないのですが、この点Qというものは何者なのか教えていただけないでしょうか?求め方を教えていただけないでしょうか?
0396132人目の素数さん
垢版 |
2019/11/13(水) 19:07:35.30ID:EAVJoV2Y
ある△ABCについて、その外心Oを中心とするある円Kが存在し、Kは以下の2条件を満たすという。

・Kは△ABCの各辺とそれぞれ異なる2つの交点を持つ(したがって、Kと△ABCは6つの異なる交点を持つ)

・ABとKの交点をP,Q、BCとKの交点をR,S、CAとKの交点をT,Uとおくと、
∠POQ、∠ROS、∠TOU
のいずれもπの有理数倍である

△ABCの形状を全て決定せよ。
相似形は考慮しなくてよい。
0397132人目の素数さん
垢版 |
2019/11/13(水) 23:47:53.95ID:s6ZQZRbo
>>395
そのメネラウスの定理の辺と比の式を書いてくれよ。
空間ベクトルだぞ?
0400132人目の素数さん
垢版 |
2019/11/14(木) 00:44:40.32ID:FsuFRGjN
x>0で定義された関数f(x)=ln(x)/xについて、以下の問に答えよ。

(1)f(x)の増減を調べ、f(x)の最大値を求めよ。

(2)m,nをm<nの相異なる自然数とするとき、等式f(m)=f(n)を成立させる(m,n)の組を全て求めよ。

(3)p,qを有理数とするとき、等式f(p)=f(q)を成立させる(p,q)の組が有限個であるか、理由をつけて述べよ。
0402132人目の素数さん
垢版 |
2019/11/14(木) 06:36:21.81ID:7OF7+yl7
4-エックスジジョウを ビブンするとき ゼンタイビブン なかみビブンで
-エックスニジョウを ビブンするとき -エクスジジョウでのく +エクスジジョウにして
かぞえあげを 誤り(とちり)てました。どうもありがたくそうろいました。
0403イナ ◆/7jUdUKiSM
垢版 |
2019/11/14(木) 10:59:21.33ID:c8D1dxnq
>>395訂正。
>>394
>>397メネラウスの定理の辺の比は、OCの中点をMとして、Bを起点に、
(BA/AP)(PG/GM)(MQ/QB)=1
(3/2)(2/1)(MQ/QB)=1
MQ/QB=1/3
Aを起点に、
(AG/GQ)(QM/MB)(BP/PA)=1
(AG/GQ)(1/4)(1/2)=1
AG/GQ=8
→AQ=(9/8)→AG――@
→OG=(1/3)→OP+(1/3)→OC――A
→OP=(1/3)→OA+(2/3)→OB――B
AをBに代入し、
→OG=(1/3){(1/3)→OA+(2/3)→OB}+(1/3)→OC
=(1/9)→OA+(2/9)→OB+(1/3)→OC――C
→OQ=→OA+→AQ
@より、
→OQ=→OA+(9/8)→AG
=→OA+(9/8)(→OG-→OA)
=(-1/8)→OA+(9/8)→OG
Cを代入し、
→OQ=(-1/8)→OA+(9/8){(1/9)→OA+(2/9)→OB+(1/3)→OC}
=(-1/8)→OA+(1/8)→OA+(1/4)→OB+(3/8)→OC
=(1/4)→OB+(3/8)→OC
0404132人目の素数さん
垢版 |
2019/11/14(木) 12:02:33.85ID:L5m7M805
文系馬鹿の質問ですみません
今度ホンダからCBR1000RR-Rという長ったらしいバイクが出るのですが
これを数学的にシンプルな式に変換できませんか?
0406132人目の素数さん
垢版 |
2019/11/14(木) 14:40:54.29ID:CYKGROTS
>>400

(1) f '(x) = {1-ln(x)}/x^2,
 x<e で単調増加、x>e で単調減少
x=e で最大値 f(e) = 1/e,
なお f "(x) = {2ln(x)-3}/x^3, f "(e) = -1/e^3 < 0,

(2) 題意より
 m < e < n
 f(1) = 0 > f(n) ・・・・ 不適
 f(2) = ln(2)/2 = ln(4)/4 = f(4),
0407a4 ◆L1L.Ef50zuAv
垢版 |
2019/11/14(木) 18:28:51.84ID:UfXDYHgL
>>404
文字列の画像をbmpからjpegに変換するなら短くはなるんじゃないかと。
0408132人目の素数さん
垢版 |
2019/11/14(木) 20:36:16.27ID:+IsNbCMn
次の差分方程式で表されるデジタル・システムのインパルス応答を求めよ。
y(k)=−y(k−1)−y(k−2)+0.3x(k−1)

紛らわしくてすみませんがk,k-1,k-2はx,yの右下の文字です。
Y(z)=-z^-1Y(z)-z^-2Y(z)+0.3z^-1X(z)と変換して解くのだと思います。
答えはh(k)=0.2√3sin((2πk/3)+f)ですが、
このfの数値が分かりません。誰か教えて下さい。
0409132人目の素数さん
垢版 |
2019/11/14(木) 20:48:46.52ID:lUItUZBi
でんきさんすーは物理板か電気板辺りできいた方が良いんじゃない?
0410132人目の素数さん
垢版 |
2019/11/14(木) 21:13:06.75ID:CYKGROTS
>>401

 ===== 問 題 =====

6 次の関数の最大値と最小値を求めよ。
  (1) y = x√(4-x^2),
  (2) y = sin(x)^3 + cos(x)^3  (0≦x≦π/2)

7 0<x<π のとき,不等式 sin(x) > x・cos(x) を証明せよ。

8 aを定数とするとき,xについての方程式 2x^3 -ax^2 +1 = 0 の異なる
  実数解の個数が3個となるようなaの値の範囲を求めよ。


2節 ===== 解 答 ===== ・・・・ p.173

6 (1) x=√2 のとき 最大値 2,
    x=-√2 のとき 最小値 -2,
  (2) x=0,π/2 のとき 最大値 1,
    x=π/4 のとき 最小値 (√2)/2,

7 f(x) = sin(x) - x・cos(x) とおき、f(x) が区間 0≦x≦π で増加することを示す。

8 a > 3,
0412132人目の素数さん
垢版 |
2019/11/14(木) 22:38:10.55ID:CYKGROTS

 (1)
 (2+y)(2-y) = 4-yy = 4 - xx(4-xx) = (2-xx)^2 ≧ 0,
∴ -2≦y≦2
 等号は 2-xx=0, x=±√2 のとき。

 (2)
 y = {sin(x) + cos(x)}{sin(x)^2 -sin(x)cos(x) + cos(x)^2}
  = {sin(x) + cos(x)}{1 - sin(x)cos(x)}
  = {sin(x) + cos(x)}{3 - [sin(x)+cos(x)]^2}/2
  = X(3-XX)/2
ここに X = sin(x) + cos(x) = (√2)sin(x+π/4) とおいた。
 0≦x≦π/2 より 1≦X≦√2,

 1 - y = (1/2)(X+2)(X-1)^2 ≧ 0,
 y - (√2)/2 = (√2 -X){XX +(√2)X -1} ≧ 0,
∴ (√2)/2 ≦ y ≦ 1,
等号は (右) X=1, x=0,π/2 のとき 1
    (左) X=√2, x=π/4 のとき (√2)/2,


 0<x<π のとき sin(x) >0,
 x/tan(x) < 1 より
  x・cos(x) = {x/tan(x)}sin(x) < sin(x),

あ、微分するの忘れてたわ。
0413132人目の素数さん
垢版 |
2019/11/14(木) 23:08:26.33ID:CYKGROTS
>>394
 OP↑ = (OA↑ + 2 OB↑)/3,
 OG↑ = (OO↑ + OP↑ + OC↑)/3 = (3 OO↑ + OA↑+ 2 OB↑ + 3 OC↑)/9, >>399
 AG↑ = OG↑ - OA↑ = (3 OO↑ -8 OA↑+ 2 OB↑ + 3 OC↑)/9,
 AQ↑ = (9/8)AG↑ = -OA↑ + (3 OO↑ + 2 OB↑ + 3 OC↑)/8,
   (OA↑の係数を-1にする)
 OQ↑ = OA↑ + AQ↑ = (3 OO↑ + 2 OB↑ + 3 OC↑)/8,  >>403
にて一見落着。
0414132人目の素数さん
垢版 |
2019/11/14(木) 23:33:43.50ID:FsuFRGjN
rを正の実数とする。
xyz空間の半球B:x^2+y^2+z^2=r^2(z≥0)について以下の問に答えよ。

(1)nを2以上の自然数とする。
平面H_kをz=kr/n(k=0,1,...,n-1)と定め、H_kとBの交線である円をC_kとする。
C_kを底円とし高さがr/nである円筒の側面積をS_kとするとき、それらの和
T_n = Σ[k=0,1,...,n-1] S_k
を求めよ。

(2)lim[n→∞] T_n とBの側面積は一致しないことを示せ。
0415132人目の素数さん
垢版 |
2019/11/15(金) 06:54:41.29ID:X6o/YTJ4
Ldi/dt+Ri=0の計算過程を教えてください。
答えはi=e^(-Rt/L+C) (Cは積分定数)になります。
0417132人目の素数さん
垢版 |
2019/11/16(土) 17:55:12.93ID:c8Gst240
〔応用問題〕 
次を示せ。
(a) n! > n^n / e^(n-1),
(b) n! < n^(n+1) / e^(n-1),
(c) n! < n^(n+1/2) / e^(n-1),
0418132人目の素数さん
垢版 |
2019/11/16(土) 17:56:08.96ID:c8Gst240
(略証)
(a) >>289 より
 (1+1) < (1+1/2)^2 < (1+1/3)^3 < ・・・・ < {1+1/(n-1)}^(n-1) < e,
すなわち
 2 < (3/2)^2 < (4/3)^3 < ・・・・ < {n/(n-1)}^(n-1) < e,
右のn-1項を掛け合わせて
 n^n / n! < e^(n-1),

(b) >>289 より
 (1+1)^2 > (1+1/2)^3 > (1+1/3)^4 > ・・・・ > {1+1/(n-1)}^n > e,
すなわち
 2^2 > (3/2)^3 > (4/3)^4 > ・・・・ > {n/(n-1)}^n > e,
右のn-1項を掛け合わせて
 n^(n+1) / n! > e^(n-1),

(c) >>289 より
 (1+1)^(3/2) > (1+1/2)^(5/2) > (1+1/3)^(7/2) > ・・・・ > {1+1/(n-1)}^(n-1/2) > e,
すなわち
 2^(3/2) > (3/2)^(5/2) > (4/3)^(7/2) > ・・・・ > {n/(n-1)}^(n-1/2) > e,
右のn-1項を掛け合わせて
 n^(n+1/2) / n! > e^(n-1),
0419132人目の素数さん
垢版 |
2019/11/17(日) 03:20:16.37ID:bVYW1FCH
〔応用問題〕 
次を示せ。
(a) (2n)! / n! > (4n/e)^n,
(b) (2n)! / n! < 2(4n/e)^n,
(c) (2n)! / n! < (√2)(4n/e)^n,

(略証)
 (1+1/n)^(n+a), {1+1/(n+1)}^(n+1+a), ・・・・・, {1+1/(2n-1)}^(2n-1+a)
すなわち
 {(n+1)/n}^(n+a), {(n+2)/(n+1)}^(n+1+a), ・・・・・, {2n/(2n-1)}^(2n-1+a)
のn個を掛け合わせると
 (2^a)(4n)^n・n!/(2n)!,
これと e^n と比べる。 >>289
0420132人目の素数さん
垢版 |
2019/11/17(日) 10:57:41.91ID:/4QkpOpq
連結なリーマン多様体は距離空間になりますが、逆に可微分多様体に距離が定まっているときリーマン計量が誘導されますか?
0423132人目の素数さん
垢版 |
2019/11/17(日) 12:32:02.69ID:OuUZuRko
滑らかな1の分割1=Σfiとsupp fi上の滑らかな計量giを用意してg=Σfigiで行けるのでは?
0426132人目の素数さん
垢版 |
2019/11/17(日) 15:17:34.46ID:bVYW1FCH
スターリングの公式
 n! ≒ n^(n+1/2) e^(-n + 1/(12n)) √(2π)
と比べてみると・・・・

>>417(c) は真値の約 1.08444 倍。

>>419(c) は真値の約 exp(1/(24n))倍。n→∞ では1に近づく。
0427132人目の素数さん
垢版 |
2019/11/17(日) 22:49:00.29ID:9ZLYQCr+
(-∞,∞)で微分可能な関数f(x)、実定数h、および実定数aに対し、g(x)を以下のように定める。
・x≠hのとき、g(x)={f(x)-f(h)}/(x-h)
・x=hのとき、g(x)=a
このとき、g(x)が(-∞,∞)で微分可能であるための必要十分条件を述べよ。
0432132人目の素数さん
垢版 |
2019/11/18(月) 14:31:17.05ID:+ecGhDD/
>>430
R上でd(x,y)=|x^4-y^4|とかで原点での計量テンソルはそのままでは死ぬ。
dが滑らかとか制限かけても死ぬ例あると思う。
そもそもgがC^∞級の計量テンソルだったとしても、そこから作った距離がC^∞とは限らない(eg. Rの通常の距離でg=1はC^∞だけどd(x,y)=|x-y|はC^∞ではない)のでdにC^∞の制限かける事自体、適当とは思えないし。
0434132人目の素数さん
垢版 |
2019/11/18(月) 18:15:20.51ID:RYFckUIJ
A∪B∪C∪Dみたいなのを、A∩B∩Cみたいなのの交代和にできる定理の名前ってなんでしたっけ
0435132人目の素数さん
垢版 |
2019/11/18(月) 18:17:23.99ID:ZoexRUOh
n=1,2,...について、
a[n]=(1+1/n)^n
で定められる数列{a[n]}と、
b[n]=p*a[n+1]+q*a[n]+r
により定められる数列{b[n]}を考える。
このとき以下の条件をすべて満たすように、pq≠0なる実数p,q,rをとることは可能か。

・lim[n→∞] b[n] = lim[n→∞] a[n] (=e)
・ある自然数Nが存在して、Nより大きい任意の自然数kに対し|b[k]-e|<|a[k]-e|
0437132人目の素数さん
垢版 |
2019/11/18(月) 18:26:33.37ID:phk4LRaO
0 ≦ θ0 ≦ π とする。

∫_{θ0}^{π} sqrt(1 - cos(θ)) / sqrt(cos(θ0) - cos(θ)) dθ

を求めよ。
0439132人目の素数さん
垢版 |
2019/11/18(月) 18:35:47.69ID:0TcS9s3A
>>437
マルチするな
0440132人目の素数さん
垢版 |
2019/11/18(月) 19:27:52.73ID:RYFckUIJ
(1)お願いします
Pを(x,p)と置いてd=d(x)をxで微分するという考えで解いたのですがどこが間違いでしょうか?
x→±∞でd→∞なので、
d'(x)=0となるかd'(x)が計算できない(d'=±∞)のxのうちのどれかが最小のdを与えるはずと思ったんですが、
本来必要な場合分けを考慮しない解答が得られてしまいました
2≦p≦4の時に対しては正しい解なので計算ミスが原因ではない?と思うのですが
これは何が悪かったのでしょうか?

https://i.imgur.com/yR3Ejcf.jpg
https://i.imgur.com/ozST013.jpg
0441132人目の素数さん
垢版 |
2019/11/18(月) 21:42:04.27ID:+sob8BsZ
z2を2で割った余りの群とする
準同型写像z2xz2→z2はいくつあるかという問題がわかりません
0442132人目の素数さん
垢版 |
2019/11/18(月) 21:48:36.20ID:TSILQFfn
写像全体を見ても精々2^4=16個しかないし、(0,0)を0に写すものだけ考えれば2^3=8個しかないんだから一つずつ準同型になるか確認してみたら?準同型に関するいい演習問題だと思って
0443132人目の素数さん
垢版 |
2019/11/18(月) 22:00:12.31ID:phk4LRaO
>>437

ヒント: θ0に依存しません。
0444132人目の素数さん
垢版 |
2019/11/18(月) 22:02:42.98ID:+sob8BsZ
解答の一部にf(z2xz2)=z2のときkerfの元の個数が2よって全準同型写像とz2xz2のいすう2の部分群の個数が一致するって書いてるんですがよくわかりません
0445132人目の素数さん
垢版 |
2019/11/18(月) 22:05:10.18ID:39+/YJbu
よくわかりませんってことは、まあまあわかるんだからガンバ><
0446132人目の素数さん
垢版 |
2019/11/18(月) 22:08:55.27ID:TSILQFfn
>>444
>解答の一部にf(z2xz2)=z2のときkerfの元の個数が2
準同型定理、Z2=(Z2×Z2)/Kerfの位数を考える

>よって全準同型写像とz2xz2のいすう2の部分群の個数が一致する
これも準同型定理、位数2の部分群を核とする準同型写像fを考えれば自動的に全射
0447132人目の素数さん
垢版 |
2019/11/18(月) 22:36:23.56ID:+sob8BsZ
なぜ|kerf|=2だとf:z2 x z2 →z2からなる全準同型写像と
z2 x z2の位数2の部分群の個数が一致するのですか?
定理か何かあるのですか?
0448132人目の素数さん
垢版 |
2019/11/18(月) 22:36:34.33ID:v+leOqMQ
ひどい解答だな。一般にkerf=kerg⇒f=gなんて言えるハズないんだから、本問ではそれが言えるのも言わんといかん。
しかしそんなルート通ろうと思うかね?
0449132人目の素数さん
垢版 |
2019/11/18(月) 23:04:49.17ID:TSILQFfn
>>447
あ、わからないのは個々の主張じゃなくて「よって」の部分か
[任意の全射準同型に対して|Ker|=2]⇒#{全射準同型}=#{位数2の部分群}
を示せってことね
これを示すには>>448を示せばいい

位数2の部分群Hを核にもつ準同型f,gがあったとする
もしa∈Z2×Z2でf(a)≠g(a)となったとすると、f(a)=0またはg(a)=0よりa∈Hとなるのでf(a)=g(a)=0でなければならない
ゆえにf=g
0450132人目の素数さん
垢版 |
2019/11/18(月) 23:51:44.20ID:szggTHgJ
>>441
Hom(Z2Z2,Z2)=Z2Z2で4つだけど
自己同形で区別なら2つ
0451132人目の素数さん
垢版 |
2019/11/19(火) 01:15:45.83ID:MvZ1phLK
>>434
 ド・モルガンの法則?

>>435
a[n] → e  (n→∞) より
 (p+q)e + r = e,
ここで
 0<q<1, 0<p+q<1, r=(1-p-q)e,
とおいてみよう。
 a[k] < a[k+1] < e,
から
 e - b[k] = (p+q)(e-a[k+1]) + q(a[k+1]-a[k]) > 0,
 b[k] - a[k] = (1-p-q)(e-a[k+1]) + (1-q)(a[k+1]-a[k]) > 0,
∴ a[k] < b[k] < e,
∴ 0 < e-b[k] < e-a[k] が成立

>>437
 π
 (θ≒θ。 で発散・・・・ 広義積分?)
0454132人目の素数さん
垢版 |
2019/11/19(火) 13:32:46.06ID:8zdmomcm
>>451
ご存知でしたら数列をより早く収束させる一般論が存在するかご教示願えませんか。
0455132人目の素数さん
垢版 |
2019/11/19(火) 17:31:26.13ID:MvZ1phLK
たとえば
 c[n] = (1+1/n)^(n+1/2)
はどうでしょうか。(単調減少ですが)

(a[n] と a[n+1] を組合せても nが1ずれる程度で、早くはなりません。)
0456132人目の素数さん
垢版 |
2019/11/19(火) 17:47:45.96ID:MvZ1phLK
あるいは
 a[n] ≒ e/√(1+1/n) ≒ e{1 - 1/2(n+1)}
から
 b[n] = (n+2)a[n+1] - (n+1)a[n],
としては?
0457132人目の素数さん
垢版 |
2019/11/19(火) 20:52:07.44ID:MvZ1phLK
>>456 を修正。。。

 a[n] = (1+1/n)^n ≒ e^{1 -1/(12nn)}/√(1+1/n) ≒ e{1 - 1/(2(n+5/6))}
から
 b[n] = (n +11/6)a[n+1] - (n +5/6)a[n],
としては?
0458132人目の素数さん
垢版 |
2019/11/19(火) 21:45:38.40ID:oytmbi3J
商群の像ってなんですか
0459132人目の素数さん
垢版 |
2019/11/19(火) 21:48:22.46ID:oytmbi3J
a*は元aの商群<a>/<a^2>での像を表しってあるのですが商群の像ってなんですか
0461132人目の素数さん
垢版 |
2019/11/19(火) 21:57:47.37ID:MvZ1phLK
>>457
 修正幅が倍になってますた。再修正を.....orz
 a[n] = (1+1/n)^n ≒ e^{1 -1/(12nn)}/√(1+1/n) ≒ e{1 - 1/(2(n+11/12))}
から
 b[n] = (n +23/12)a[n+1] - (n +11/12)a[n],
0462132人目の素数さん
垢版 |
2019/11/19(火) 22:03:22.66ID:oytmbi3J
標準射影ってなんですか?
その像は2乗すると単位元になりますか?
0463132人目の素数さん
垢版 |
2019/11/19(火) 22:14:49.28ID:NjIHoz8s
像を二乗?w
0464132人目の素数さん
垢版 |
2019/11/19(火) 22:19:55.60ID:wMI7t6A9
>>462
どういう本を読んだか知らないけど、商写像もしくは標準全射、自然な全射、商射影と言えばわかる?
単位元になるかどうかは実際にやってみればいい
0465132人目の素数さん
垢版 |
2019/11/19(火) 22:26:47.32ID:oytmbi3J
Dn(nは偶数)は二面体群でaを回転するやつbをひっくり返すやつとするときDn/<a^2>~={e,b}x{e, a*}~=c^2xc^2
(c2は位数2の巡回群)で真ん中の同型がわからないa*は元aの商群<a>/<a^2>での像を表しって書いてるけど意味がわからないから教えてください
0466132人目の素数さん
垢版 |
2019/11/19(火) 22:44:26.71ID:Re3h5cg0
群G、その正規部分群K、GのKによる商群G/K なんて言葉で表わされる数学的な概念をちゃんと掴むことが必要なのかも。
0467132人目の素数さん
垢版 |
2019/11/19(火) 23:07:49.84ID:oytmbi3J
自然な射影ってことかもしれなかったでした
0468132人目の素数さん
垢版 |
2019/11/20(水) 17:05:51.63ID:0VJwxXDk
>>454
数列の加速に関する基本定理:
「極限を持つ任意の数列の収束を加速するアルゴリズムは存在しない。」

ただし任意でない規則性を持つ数列の加速法ならば存在し、エイトケン加速が有名です。
数列{(1+1/n)^n}の場合はエイトケン加速を繰り返し使うことで収束がずっと速くなります。
0469132人目の素数さん
垢版 |
2019/11/20(水) 22:40:01.70ID:3buwO4hW
以下の広義積分の値が(1/2)√(π/e)となる実数aの値を求めよ。
必要であれば
∫[0→∞] sin(x)/x dx = (1/2)√(π/2)
を用いて良い。

∫[0→∞] sin(ax)/x dx
0471132人目の素数さん
垢版 |
2019/11/21(木) 00:22:05.17ID:csjb64+h
∫[0→∞] sin(x)/x dx = π/2,
∫[0→∞] sin(ax)/x dx = sign(a)π/2,

高木:「解析概論」改訂第三版, 岩波書店 (1961)
 第4章 §48 [例4]  p.168-169 実数、技巧的
 第5章 §62 [例1]  p.223-224 複素変数
0472132人目の素数さん
垢版 |
2019/11/21(木) 00:46:43.27ID:jINudw3y
初項から第n項までの和Snが次の式で表される数列{an}の一般項を求めよ。
(2)Sn=3^n+2n

階差 8 20 56の規則性がわかりません。3の累乗で無理やり一般項を作ろうとしても無理でした。
2時間以上格闘しました。先生お教えいただけないでしょうか?
0475132人目の素数さん
垢版 |
2019/11/21(木) 01:34:43.09ID:csjb64+h
a_1 = S_1 = 5,
n>1 のとき
a_n = S_n - S_{n-1}
 = (3^n + 2n) - {3^(n-1) - 2(n-1)}
 = 2・3^(n-1) + 2,
これより一般項は
a_n = 2・3^(n-1) + 2 + 0^(n-1).
0477132人目の素数さん
垢版 |
2019/11/21(木) 02:10:27.11ID:E3Ov7Zu8
予備校模試の改題らしいのですが解けないのでお願いします
座標平面上でC:y=x^5-kx(kは実数)と置く
原点を中心としC上に4頂点を持つ正方形が存在するkの範囲を求めよ
0478132人目の素数さん
垢版 |
2019/11/21(木) 02:39:27.57ID:jINudw3y
>>475
ありがとうございました。
n>1はおかしいですよね?符号も1つ間違っていますよね?0の累乗もちょっと意味がわかりませんでしたね。
0479132人目の素数さん
垢版 |
2019/11/21(木) 09:33:58.06ID:RxtSd1r+
>>477
y=x^5-kxを極方程式にして
r^4=(sinθ+kcosθ)/(cosθ)^5。
π/2回転して
r^4=(-cosθ+ksinθ)/(sinθ)^5。
条件は
(sinθ+kcosθ)/(cosθ)^5
=(-cosθ+ksinθ)/(sinθ)^5
が0≦θ≦π/2で解を持つ事。
すなわち
k=(sin^6θ+cos^6θ)/(cos^5θ-sin^5θ)の値域。
wolfram先生にグラフ書いてもらったらθ=π/4の時の値をk0として正の値域はk≧k0。
‥‥
0481132人目の素数さん
垢版 |
2019/11/21(木) 13:52:58.80ID:eJmyUNPB
a - b = cが成り立つ時
cとaが互いに素な事を利用してaとbが互いに素な事を証明しろって問題あるんですけど
今日学校で習ったら対偶命題を明らかにしろって書かれてますがコレって

a=kA b=kBとおいてk(A-B)=c、この時kが1以外は有り得ない!って証明じゃダメなんですか?
0482132人目の素数さん
垢版 |
2019/11/21(木) 14:03:08.20ID:lRL7YkNF
気になったんで今やってみた。
kについて解いた式間違ってた。
k=(4-3sin2θ)/(2sin4θ)。
これの値域高校数学の範囲内で解けるかな?
0485132人目の素数さん
垢版 |
2019/11/21(木) 15:19:52.55ID:hSO9LIQF
>>449
てことは準同型写像は1個ってことですか?
0486132人目の素数さん
垢版 |
2019/11/21(木) 16:05:18.98ID:hSO9LIQF
すいません4つでした
0487132人目の素数さん
垢版 |
2019/11/21(木) 19:52:52.91ID:csjb64+h
>>457 >>461
まだおかしいな....orz
a[n] = (1 +1/n)^n ≒ e^{1 + 1/[12n(n+1)]} /√(1+1/n)
 ≒ e{1 - 1/(2n) + 11/(24nn) - 7/(16n^3)}
 ≒ e{1 - 1/[2(n + 11/12 -5/(144n))]},
から
 b[n] = (n +23/12 -5/(144n)) a[n+1] - (n +11/12 -5/(144n)) a[n],
だな。
0488132人目の素数さん
垢版 |
2019/11/21(木) 21:02:46.41ID:csjb64+h
>>477
C: y = x^5 - kx,
を90゚回すと
C': -x = y^5 - ky,
これらが (0,0) 以外の交点もつ条件を求める。
k≦0 ならば (0,0) のみだから
 k > 0   ・・・・ (1)
 y/x = tanθ = t,
とおく。
 t^5 = (-1+kt)/(t+k),
 (t^3 + 1/t^3) + k(t^2 - 1/t^2) = 0,
t+1/t≠0 で割ると  (← |t+1/t|≧2)
 (tt-1+1/tt) + k(t-1/t) = 0,
 (t-1/t)^2 + k(t-1/t) + 1 = 0,
これが実根をもつから
 |k| ≧ 2   ・・・・ (2)
(1),(2) より k≧2,
0489132人目の素数さん
垢版 |
2019/11/21(木) 21:09:15.49ID:csjb64+h
>>488
C': y^5 = -x + ky,
C: x^5 = y + kx,
を辺々割って
 t^5 = (-1+kt)/(t+k),
 t^5 (t+k) -kt +1 = 0,
これを t^3 で割ると
 (t^3 + 1/t^3) + k(t^2 - 1/t^2) = 0,
これをさらに t+1/t ≠0 で割る。
0490132人目の素数さん
垢版 |
2019/11/21(木) 22:28:41.13ID:jINudw3y
次の数列の初項から第n項までの和Snを求めよ。

(1)1/(1・4),1/(4・7),1/(7・10),1/(10・13),・・・・・・

1=4-3,1=2-1?
先生教えて下さい。
0492132人目の素数さん
垢版 |
2019/11/21(木) 22:51:23.72ID:w2xrCldR
これあれだろ
前に対数の問題で暴れてたやつ
0493132人目の素数さん
垢版 |
2019/11/21(木) 23:21:32.79ID:csjb64+h
そうだよ。

>>479 >>482
 (sinθ)^2 + (cosθ)^2 = 1, より

 k = {(sinθ)^6 + (cosθ)^6}/{sinθcosθ[(cosθ)^4 - (sinθ)^4]}
  = {(sinθ)^4 -(sinθcosθ)^2 +(cosθ)^4}/{sinθcosθ[(cosθ)^2 - (sinθ)^2]}
  = {1 - 3(sinθcosθ)^2}/{sinθcosθ[(cosθ)^2 - (sinθ)^2]}
  = {4 - 3sin(2θ)^2}/{2sin(2θ)cos(2θ)}
  = {4 - (3/2)[1-cos(4θ)]}/{sin(4θ)}
  = {5 + 3cos(4θ)}/{2sin(4θ)},
ここで、
 {5 + 3cos(x)}^2 = {3 + 5cos(x)}^2 + (5^2-3^2){1-cos(x)^2}
  = {3 + 5cos(x)}^2 + (4^2)sin(x)^2
  ≧ 4{2sin(x)}^2,
に注意すれば、kの値域は
 |k| ≧ 2 = k。

>>488
k=k。=2 のとき (t -1/t) = -1,
 0 = tt +t -1 = (t+φ)(t-1/φ),
 t = -φ, 1/φ
C と C' は次の4点で接する。
 (√φ, 1/√φ), (-1/√φ, √φ), (-√φ, -1/√φ), (1/√φ, -√φ)

 φ = (1+√5)/2 = 1.618034  黄金比

>>490
(1)
 a_k = 1/{(3k-2)(3k+1)} = (1/3){1/(3k-2) - 1/(3k+1)},
 S_n = Σ[k=1,n] a_k = (1/3){1 - 1/(3n+1)} = n/(3n+1),
0494132人目の素数さん
垢版 |
2019/11/22(金) 04:56:03.33ID:3r/t/szP
>>492
もしかして態度めちゃくちゃ悪かった奴?
こっちは塾講師になるしかないって喚いたカスか
0495132人目の素数さん
垢版 |
2019/11/22(金) 17:18:30.84ID:iULrXsIp
>>493
{(3k-2)(3k+1)} 、これ初見で絶対気づけないだろでした。
先生助かりました。大変ありがとうございました。
0496132人目の素数さん
垢版 |
2019/11/22(金) 18:04:42.95ID:GDfT7FjC
以下、正三角形と言うときは一辺の長さが1の正三角形を指す。

平面Hは正三角形によって隙間なく埋め尽くされている。
ある時刻0において、それら正三角形のうち1つを選んで黒く塗りつぶし、それをT、またTの重心をOと定める。

さらに時刻n(n=1,2,3...)ごとに、
「現在黒く塗りつぶされている正三角形に隣接するすべての正三角形を、黒く塗りつぶす」
という操作を行う。

時刻Nにおいて、黒く塗りつぶされている正三角形全体からなる領域D[N]上で、点Oから最も距離が離れている点を1つとり、それをP[N]とする。

線分長OP[N]をd[N]、D[N]の面積をS[N]とおくとき、
lim[N→∞] (d[N]^2)/D[N]
を求めよ。
0497132人目の素数さん
垢版 |
2019/11/22(金) 23:24:31.10ID:zZAbrMZn
巡回群Cnの共役類の個数ってアーベル群だからn個ですか?あってますか?
0499132人目の素数さん
垢版 |
2019/11/22(金) 23:35:59.28ID:zZAbrMZn
ありがとうございます
0500132人目の素数さん
垢版 |
2019/11/23(土) 12:20:10.92ID:m/FSqs0y
誘導表現の表現行列はなぜあんな形になるのですか?
0501132人目の素数さん
垢版 |
2019/11/23(土) 12:39:40.67ID:u+xN0AGh
どの面も鏡面である正四面体Vを考える。Vの1頂点をAとし、その対面上に点Pをとる。

AからPに向かって光線を放つと、光線はVの内部を通り、各面で次々と反射していく。
点Pを対面上のどの場所にとるかにより、光線がVの4面すべてに接触するか否かが変わる。

光線がVの4面すべてに接触するようなPのとり方を述べ、そのようなP全体の集合を図示せよ。
なお光線がVの頂点または辺に接触した場合、それは面に接触したとはみなさないこととする。
0503132人目の素数さん
垢版 |
2019/11/23(土) 13:02:21.55ID:lHHxttJQ
光線が辺に当たって消えてしまうなら鏡に辺が写ってる部分の全体。
図示できるわけがない。
0506132人目の素数さん
垢版 |
2019/11/23(土) 15:35:55.65ID:BK1DSCKR
>>496

D[N] は 3N(N+1)/2 +1 個の「正三角形」からなる。
∴ S[N] = {3N(N+1)/2 +1}(√3)/4,

Tの頂点を (±1/2, -1/(2√3)) (0, 1/√3) とおく。

P[N] = (±(N+1)/2, -1/(2√3)) (Nが偶数)
  = (±(N+1)/2, 1/√3)   (Nが奇数)

∴ d[N]^2 = (OP[N])^2
  = {(N+1)/2}^2 + (1/12),  (Nが偶数)
  = {(N+1)/2}^2 + (1/3),   (Nが奇数)

∴ lim[N→∞] (d[N])^2 / S[N] = 2/(3√3),

N=0 で 4/(3√3) だったのが減少して 2/(3√3) に近づく。

N=0 の「正三角形」が徐々に崩れて「正六角形」に近づくことを表わす。
0507132人目の素数さん
垢版 |
2019/11/23(土) 16:30:57.29ID:i25fBy3X
>>497
あってるというか
その質問は出るはずがない
0509132人目の素数さん
垢版 |
2019/11/23(土) 21:53:57.64ID:m/FSqs0y
G を有限群, D(G) をその交換子群とする. このとき, G のすべての 1 次指標は, アー ベル群 G/D(G) の既約指標 χ0 と標準的な準同型写像 π : G → G/D(G) との合成 χ0 ◦ π として得 られる.
証明がわかりません教えてください
0511132人目の素数さん
垢版 |
2019/11/24(日) 23:34:41.19ID:FgHXk+oJ
次の数列の初項から第n項までの和Snを求めよ。

(2) 1/(1・4・7),1/(4・7・10),1/(7・10・13),1/(10・13・16),・・・・・・

(3) 1/(1・4・7・10),1/(4・7・10・13),1/(7・10・13・16),1/(10・13・16・19),・・・・・・
0512132人目の素数さん
垢版 |
2019/11/25(月) 01:23:57.46ID:+7EInZ9j
4*x^3 -4*x^2 -40*x -79 mod 11
≡ 4*(x -5)^2 * (x -2) mod 11
らしいんだが、計算法が分かりません。
0514132人目の素数さん
垢版 |
2019/11/25(月) 14:49:49.45ID:WFy/GgNc
単に計算法なら
f(x):=4*x^3 -4*x^2 -40*x -79 とおくと, f(2)≡0
割り算を実行して, f(x)=(x-2)*g(x)
g(5)≡0 なので…なだけでね
多少工夫のしようはあるが、計算法すらわからないなら上一択
0515132人目の素数さん
垢版 |
2019/11/25(月) 16:56:41.75ID:JoXp5/Or
3次方程式
x^3-19x+n=0
が、実部虚部ともに整数である解を持つ。
このような自然数nを全て求めよ。
0517132人目の素数さん
垢版 |
2019/11/25(月) 17:45:12.04ID:LiiNnBRe
実部虚部ともに整数である解を1つ持てばいいのだから、
x=0という解を持つn=0
x=1という解を持つn=18
等の解があるだろう
0520132人目の素数さん
垢版 |
2019/11/26(火) 02:42:06.35ID:USg5aONF
3解がa+bi, a-bi, -2a (a,b:整数)の形の解は解ないよ。
この形の解はない愚問。
0521132人目の素数さん
垢版 |
2019/11/26(火) 06:46:09.26ID:/rJkC9KX
>>511

(2)
b_k = 1/{(3k-2)(3k+1)(3k+4)}
 = 1/{6(3k-2)(3k+1)} - 1/{6(3k+1)(3k+4)}

和 1/24

(3)
c_k = 1/{(3k-2)(3k+1)(3k+4)(3k+7)}
  = 1/{9(3k-2)(3k+1)(3k+4)} - 1/{9(3k+1)(3k+4)(3k+7)}

和 1/252
0522132人目の素数さん
垢版 |
2019/11/26(火) 17:54:56.59ID:KOr1Vr3j
1<x<9とする。
AB=4,BC=5,CA=xの△ABCがある。
辺AB上にn個の点P_1,P_2,...,P_nを、
AP_1=P_1P_2=...=P_(n-1)P_n=P_nB
となるようにとる。 

f(x) = (1/n)*{ Σ[k=1,2,...,n] CP_k}
の取りうる値の範囲を求めよ。
0525132人目の素数さん
垢版 |
2019/11/27(水) 01:17:06.99ID:YEgb5q8J
>>522
題意より
 BC = 5, CA = x,
第二余弦定理より
 2↑CA・↑CB = CA^2 + BC^2 - AB^2 = xx +25 -16 = xx +9,
また
 ↑CP_k = (1-t)↑CA + t↑CB,
ここに t = k/(n+1),
よって
CP_k = √(↑CP_k・↑CP_k)
 = √{(1-t)^2・xx + (1-t)t(xx+9) + (5t)^2}
 = √{(1-t)xx + (9+16t)t},
tについて下に凸 (双曲線) だから、上限は
 f_n(x) = (1/n)Σ[k=1,n] CP_k
 < ∫[0,1] √{(1-t)xx + (9+16t)t} dt
 = (1/64)(x+5)(xx-10x+41) + (1/512)(81-xx)(xx-1)log{(x+9)/(x+1)},
下限は
 f_n(x) > f_1(x) = CP_1 = √{(xx+17)/2},  (1<x<9)

f_n(1+0) = 3,  (nによらず)
f_n(9-0) = 7,  (nによらず)
0526132人目の素数さん
垢版 |
2019/11/27(水) 01:22:32.02ID:hGn3pwUt
f(x)のとりうる範囲っていだてるんだから変数はxでnは定数扱いじゃね?
0527132人目の素数さん
垢版 |
2019/11/27(水) 19:58:25.66ID:5iA7g8JS
交代群A4の交換子群ってどうやって求めることができますか
0528132人目の素数さん
垢版 |
2019/11/27(水) 20:11:16.07ID:auW39EF9
次の式で定義される数列{a[n]}の一般項を求めよ。

(1) a[1]=2,a[n+1]=a[n]+1/{n(n+1)}

b[n]の和に求めようとしているa[n]が残ってしまってわかりません。
そして、a[1]が2である情報がa[1]が消えてなくなるのに必要なのがわかりません。
生意気に書いてしまって申し訳ないのですが解き方がまったくわかりません。αを使うのでしょうか?
先生お教え願います。
0530132人目の素数さん
垢版 |
2019/11/27(水) 20:20:47.74ID:Cg2pNeKR
a[n+1]=a[n]+1/{n(n+1)}
1/{n(n+1)}=1/n-1/(n+1) を使って書き直せば
a[n+1]+1/(n+1)=a[n]+1/n
あとは分かるでしょう
0531132人目の素数さん
垢版 |
2019/11/27(水) 20:51:27.86ID:cK03Akv3
平行四辺形ABCDの辺BCを4等分した点のうちCに近い点をE、CDを3等分した点のうちCに近い方の点をFとする。DとE、AとFをそれぞれ結んだときの交点をGとする。

四角形GFCEの面積は平行四辺形ABCDの面積の何倍か求めよ。

という問題の解法が分かりません。お教え願います。
0532132人目の素数さん
垢版 |
2019/11/27(水) 21:25:03.52ID:YVbOdBz7
>>531
AFの延長とBCの延長との交点をHとする
△EGHがABCDの何倍か、△CFHがABCDの何倍かを考える
0533132人目の素数さん
垢版 |
2019/11/27(水) 22:20:34.57ID:4qB6cJ+R
凸四角形ABCDの各辺を3等分する点をとる。
すなわち、ABを3等分する点をAに近い方からP,Qとし、同様にBCをR,S、CDをT,U、DAをV,W、によってそれぞれ3等分する。
また□ABCDを4直線PU、QT、RW、SVによって9つの小四角形に分割する。

以下の問(a)(b)の結論を述べよ。

(a)□ABCDがどのような形でも、9つの小四角形の中に、面積が□ABCDの1/9であるものが少なくとも1つ存在する。

(b)9つのどの小四角形も、その面積が□ABCDの1/9である。
0534132人目の素数さん
垢版 |
2019/11/27(水) 22:22:29.99ID:auW39EF9
>>529
階差数列です。知らないんですか〜?
>>530
そこまでは当然行きました。bnにanが含まれていればanを求めるのは不可能です。
0535132人目の素数さん
垢版 |
2019/11/27(水) 22:25:24.35ID:okJaOkWT
>>534
一刻も早く私文専願に転向をすることをお勧めします
0536132人目の素数さん
垢版 |
2019/11/27(水) 22:40:26.13ID:YVbOdBz7
階差をbnとしているならbnにanが含まれているってのが意味不明だな
そして階差を考えるより>>530のほうが断然簡単
0538132人目の素数さん
垢版 |
2019/11/27(水) 23:03:04.29ID:W3qDnCai
>>527
|A4|=24ぐらいだから全部書けよ
0539132人目の素数さん
垢版 |
2019/11/28(木) 01:07:36.67ID:ghZZAPQ9
>>527

A_4 = S_4 / Z_2 より
#(A_4) = #(S_4) / #(Z_2) = 4! / 2 = 12,

V_4 = {e, (12)(34), (13)(24), (14)(23)} ・・・・ クラインの4元群
#(V_4) = 4

さて、(中略) A_4 の交換子は V_4 の元
∴ D(A_4) ⊃ V_4,
また、V_4 は S_4, A_4 の正規部分群であり、剰余群 A_4 / V_4 = H とおくと
 #H = #(A_4) / #(V_4) = 12 / 4 = 3,
∴ H はアーベル群。
∴ D(A_4) ⊂ V_4
∴ D(A_4) = V_4
0540132人目の素数さん
垢版 |
2019/11/28(木) 01:20:45.49ID:ghZZAPQ9
>>538
An は偶置換だけからなるので、 #An = (1/2)n!

A_4 = {e, (123), (132), (124), (142), (134), (143), (234), (243),
    (2143), (3412), (4321)}
0541132人目の素数さん
垢版 |
2019/11/28(木) 01:34:00.27ID:B5rRG7SA
>>540
Thx following. (12)(34) (13)(24) (14)(23)
0542132人目の素数さん
垢版 |
2019/11/28(木) 01:34:34.04ID:ghZZAPQ9
>>525

〔補題〕
g(t) が 0<t<1 で下に凸ならば
 f_n = (1/n)Σ[k=1,n] g(k/(n+1))
は単調増加。

(略証)
Jensenより
 g(k/(n+1)) < {(n+1-k)g(k/(n+2)) + k・g((k+1)/(n+2))}/(n+1),
これを右辺に入れて
 f_n < f_{n+1},
0543132人目の素数さん
垢版 |
2019/11/28(木) 02:03:44.56ID:vSevY0pT
6面体群Gをその対角線への作用を考えることによりG→S4が構成される。
全射性も容易で位数が等しいので同型。
Gは八面体群とも同型であるから八面体の対角線への作用を考えればG→S3が誘導される。
G→S3→S3/[S3,S3]
は全射だからその核がA4。
よってA4→[S3,S3]=C3が誘導される。
この核をKとすれば[A4,A4]⊂K。
一方でK={(12)(34),(13)(24),(14)(23),e}の単位元でない元は内部同型で移り合うから真の非自明正規部分群を持たない。
∴[A4,A4]=K。
0544132人目の素数さん
垢版 |
2019/11/28(木) 02:40:04.99ID:YpAebch2
>>539
ありがとうございます
0545132人目の素数さん
垢版 |
2019/11/28(木) 02:42:05.03ID:YpAebch2
構成とか誘導って一体なんですか
0546132人目の素数さん
垢版 |
2019/11/28(木) 02:44:48.97ID:YpAebch2
>>539
でもクラインの四元群がA4の交換子群だと予想しないといけないんですか
0547132人目の素数さん
垢版 |
2019/11/28(木) 12:46:21.24ID:B5rRG7SA
>>546
だから|A4|=12 ぐらいなんだから全部書けよ
11C2=55通りぐらいだから大したことない
0548132人目の素数さん
垢版 |
2019/11/28(木) 13:15:19.13ID:uwYY2BgZ
方程式に自信が無いので教えて下さい。

(0.037+x+0.0165)/85%- (0.037+x+0.0165) = 0.085

x=0.0848で合っていますでしょうか?
0549132人目の素数さん
垢版 |
2019/11/28(木) 14:27:35.68ID:AcVyz5kK
凸四角形ABCDの各辺を3等分する点をとる。
すなわち、ABを3等分する点をAに近い方からP,Qとし、同様にBCをR,S、CDをT,U、DAをV,W、によってそれぞれ3等分する。
また□ABCDを4直線PU、QT、RW、SVによって9つの小四角形に分割する。

以下の命題(a)(b)の真偽を述べよ。

(a)□ABCDがどのような形でも、9つの小四角形の中に、面積が□ABCDの1/9であるものが少なくとも1つ存在する。

(b)9つのどの小四角形も、その面積が□ABCDの1/9である。
0550132人目の素数さん
垢版 |
2019/11/28(木) 16:20:36.08ID:tXP6mf4y
線形代数のジョルダン標準形について

大学から配られた具体例を読んでいるのですが、変換行列P=(p1,p2,p3)のp1,p2,p3はどのように並べられているのですか?

λ=-1に対する固有ベクトルをp3=()
λ=2に対する固有ベクトルをp1=()
とする

と唐突に書かれていて、なぜλ=-1に対する固有ベクトルがp3になるのかが分かりません…
0551132人目の素数さん
垢版 |
2019/11/28(木) 16:25:17.58ID:vSevY0pT
固有値の順番はなんでもいい。
固有値の順番と固有ベクトルの順番を合わせればいいだけ。
固有値a,b,c に対する固有ベクトルがu,v,wなら
(u,v,w)diag(a,b,c)=A(u,v,w)。
0553550
垢版 |
2019/11/28(木) 22:48:17.16ID:uT32Lilr
本当にすみません。問題をつけるのでもう一度お願いします。
https://i.imgur.com/T6CaiWa.jpg
この後、P=(p1,p2,p3)とおいてP^(-1)APでジョルダンの標準形を求めます。

p3やp1をどのように決めているのかが分かりません。
入れ替えてしまうとジョルダンの標準形にならないです。
0554132人目の素数さん
垢版 |
2019/11/28(木) 23:08:29.97ID:vSevY0pT
固有方程式が重解もつならそのときはその重解aと対応する固有ベクトルuに対して方程式
Av=av+u
を解く。
解が見つかれば
A(uv)=(u v)[[a 1], [0 a]]
となる。
別の固有値bに対する固有方程式
Aw=bw
の解と合わせて
A(u v w)=(u v w)[[a 1 0],[0 a 0],[0 0 b]]
となる。
0555132人目の素数さん
垢版 |
2019/11/28(木) 23:09:16.69ID:B5rRG7SA
>>553
>入れ替えてしまうとジョルダンの標準形にならないです。
(p3,p1,p2)でもいいでしょ?p1,p2でひとまとめよ
0556550
垢版 |
2019/11/28(木) 23:24:37.65ID:uT32Lilr
>>554
>>555
ありがとうございます。理解できました。
0557132人目の素数さん
垢版 |
2019/11/29(金) 00:17:49.47ID:PbMEuWah
(a^2-5)/b と (b^2-5)/a がともに自然数 になるような、互いに素な自然数a,b の組( a,b ) をすべて
求めるのは難しいでしょうか。
0558132人目の素数さん
垢版 |
2019/11/29(金) 00:53:36.59ID:HxBJUCIV
>>557
ちゃんと誰かが考えて答えが出ると確定していない問題は難しい。
答えでない可能性のある問題に取り組むまんぞいくら5chの暇人でもやらない。
0560132人目の素数さん
垢版 |
2019/11/29(金) 04:53:35.58ID:ZbNFv4Wb
>>557
もし
 a_{n+1}a_{n-1} = (a_n)^2 - 5,   ・・・・ (*)
を満たすような {a_n} があったら
 {(a_n)^2 - 5} /a_{n+1} = a_{n-1} = 自然数
 {(a_n)^2 - 5} /a_{n-1} = a_{n+1} = 自然数
だから
 (a,b) = (a_n, a_{n+1})
とすればいい。
ぢゃあ、そんな旨い {a_n} はあるのか?
cosh の和積公式と似てるから
 a_n = 2cosh((2n+1)α)
はどうか?
(*) から
 a_0 = 1, a_1 = 4,
 a_{n+1} = 3a_n - a_{n-1},
これを解くと
 a_n = φ^(2n+1) + (1/φ)^(2n+1)
  = F_{2n+2} + F_{2n}
  = F_{2n+3} - F_{2n-1}  ・・・・・ フィボナッチ数
ここに φ = (1+√5)/2 = 1.618034
0561132人目の素数さん
垢版 |
2019/11/29(金) 05:05:09.31ID:ZbNFv4Wb
>>557
それより、5の倍数の方はどうなってるかな?
 (5,5) (5,10) (5,20) (10,95) (20,395) (110,205) など
0563132人目の素数さん
垢版 |
2019/11/29(金) 06:50:46.90ID:ZbNFv4Wb
>>560 訂正
 a_n = 2sinh((2n+1)α)
   = φ^(2n+1) − (1/φ)^(2n+1),
でござった....orz

a_2 = 11, a_3 = 29, a_4 = 76, ・・・・
0564132人目の素数さん
垢版 |
2019/11/29(金) 07:40:43.66ID:ZbNFv4Wb
>>546
V_4 は S_4 において共役類をなすから
x∈S_4、y∈V_4 のとき
 xyx^(-1) ∈ V_4
つまり
 xyx^(-1)y^(-1) ∈ V_4
x は (12) や (123) でもよい。 

参考文献
http://www.isc.meiji.ac.jp/~kurano/soturon/ronbun/10kurano.pdf
(明治大・理工・数)
の定理2.5 (iv) n=4 (p.8)
0565132人目の素数さん
垢版 |
2019/11/29(金) 07:58:01.88ID:ZbNFv4Wb
>>548
 (0.037+x+0.0165)/0.85 - (0.037+x+0.0165) = 0.085

 (0.037+x+0.0165)(1/0.85- 1) = 0.085

 (0.037+x+0.0165) = 0.085/(1/0.85 - 1) = 0.481667

 x = 0.481667 - (0.037+0.0165) = 0.428167
ぢゃね?
0566132人目の素数さん
垢版 |
2019/11/29(金) 08:07:24.57ID:ZbNFv4Wb
>>548
 (0.037+x+0.0165)/{0.85 - (0.037+x+0.0165)} = 0.085
とすると
 (0.037+x+0.0165) = 0.085 {0.85 - (0.037+x+0.0165)}

 (0.037+x+0.0165)(1+0.085) = 0.085×0.85 = 0.07225

 (0.037+x+0.0165) = 0.07225/(1+0.085) = 0.0666

 x = 0.0666 - (0.037+0.0165) = 0.0131
ぢゃね?
0567132人目の素数さん
垢版 |
2019/11/29(金) 12:25:09.55ID:vF6B+RWB
曲線C上を2点P,Qが動く。
P,Qの中点をM、Mの軌跡をDとするとき、P,Qの位置がどのようであっても、D上に有理点は存在しないという。
このとき、命題「C上に有利点は存在しない」の真偽を述べよ。
0570132人目の素数さん
垢版 |
2019/11/29(金) 15:59:51.51ID:MRKD0xG4
B=(√2-s)(√2-c)
ただしs,cは0以上でss+cc=1
Bの最大最小を求めよ

これってどうときますか?
そのまま微分、c=√1-ssと置いて二乗してなんとかルート消すとか色々やってみましたが自分の実力では泥沼になってしまいました
0573132人目の素数さん
垢版 |
2019/11/29(金) 16:46:27.95ID:MRKD0xG4
>>571
あのね、そんなの見りゃわかりますよ
解けないなら話し掛けなくていいスよ
結局解けたので、撤回します
0574132人目の素数さん
垢版 |
2019/11/29(金) 16:57:41.47ID:VFCQZuB+
>>536>>537
b1=a2-a1+(1/1)-(1/2)
b2=a3-a2+(1/2)-(1/3)-(1/1)+(1/2)
b3=a4-a3+(1/3)-(1/4)-(1/2)+(1/3)
bn=a[n+1]+(1/n)-(1/n+1)
bn=an+(1/n)-(1/n+1)+(1/n)-(1/n+1)←bnにanが含まれている

次の式で定義される数列{a[n]}の一般項を求めよ。

(1) a[1]=2,a[n+1]=a[n]+1/{n(n+1)}
0577132人目の素数さん
垢版 |
2019/11/29(金) 17:27:21.33ID:y7aCYuR+
>>574
何をやっているのかまるでわからない
君はbnを階差数列だって言っていなかったか?
0578132人目の素数さん
垢版 |
2019/11/29(金) 17:45:23.70ID:DcV00EMt
>>574
ごめん、何を言いたいか分からないんだけど
b1=a2-a1+(1/1)-(1/2)
って
b1=a2-a1=(1/1)-(1/2)
の誤植だったりする?
0579132人目の素数さん
垢版 |
2019/11/29(金) 19:15:12.72ID:VFCQZuB+
誤植の間違いで合っています。実際は階差というものが何なのか理解していませんでした。
0580132人目の素数さん
垢版 |
2019/11/29(金) 19:36:14.10ID:VFCQZuB+
なんと!答えまでたどり着きました!

a1+b1の残り+b[n-1]の残り=2+(1/1)+{-(1/n)}=(答)

先生、気付けました。お騒がせしました。大変ありがとうございました。
0581132人目の素数さん
垢版 |
2019/12/01(日) 00:06:00.48ID:4s3BBnlt
因数分解で
Ax^2+Bx+C=(ax+b)(cx+d) と考えるのが正解で
Ax^2+Bx+C=(a'x+b')(c'x+d')+e のような答えを考えないは何で?
0582132人目の素数さん
垢版 |
2019/12/01(日) 00:14:44.54ID:lbe/ELFr
因数分解とは、より小さい物の積で表すこと
整数の因数分解で20=4*5と考え、3*6+2としないことと同じ
0584132人目の素数さん
垢版 |
2019/12/01(日) 10:53:27.67ID:kXI40RLJ
xyz空間の平面z=0上に、点A_i(i=1,2,...,n)を頂点とする正n角形がある。

これらの頂点から異なる2点を選ぶ。
それらの座標をP(a,b,0),Q(c,d,0)と表し、また2点X(a,b,1),Y(c,d,1)をとる。

A_i(i=1,2,...,n)からPともQとも異なる2つの頂点を無作為に選び、それらをA_kとA_mとする。
直線XA_kと直線YA_mが交点を持つ確率をX(n)、直線PA_kと直線QA_mが交点を持つ確率をP(n)とするとき、
X(n)とP(n)の大小を比較せよ。
0587132人目の素数さん
垢版 |
2019/12/01(日) 13:43:31.08ID:XYunrNrA
>>581
>Ax^2+Bx+C=(a'x+b')(c'x+d')+e のような答えを考えないは何で?
和じゃん
0588132人目の素数さん
垢版 |
2019/12/01(日) 14:30:14.91ID:slEA0Uds
定義を知る前に問題を解く技を鍛錬している人なのでしょう
0589132人目の素数さん
垢版 |
2019/12/01(日) 16:27:17.23ID:uBpTHkZU
a[n]が定積分で定義されていてa[1],a[2]などの値は計算できるのですが、
a[n+1]=e-(n+1)a[n] の関係式を導き、(ここまではよくある問題)
次に、a[n]を求めよという問題ですが、これはΣをつかって表すか、
・・・などの記号をつかって表すしかないですよね。
0590589
垢版 |
2019/12/01(日) 16:34:55.77ID:uBpTHkZU
a[n]=∫(1→e){(log)^n}dx となっています。
0591132人目の素数さん
垢版 |
2019/12/01(日) 17:25:16.92ID:7DO8BvIj
曲率円の方程式を求める

 曲線 y = f(x) の曲率円の半径を R とすると
  1/R = ( 1/(1+(dy/dx)^2)^(3/2) )(d^2y/dx^2)
なので y = x^2 の曲率は
  f'(x) = dy/dx = 2x
  d^2y/dx^2 = 2
  (dy/dx)^2 = 4x^2
より
  1/R = 2/(1+4x^2)^(3/2)
  R = (1+4x^2)^(3/2)/2
 したがって x = 1 のとき
  R = 5^(3/2)/2
  f'(1) = 2
 よって y = x^2 の (1,1) における接線の傾きは 2、法線の傾きは -1/2 なので曲率円の中心(x0,y0)は
  x0 = 1 - (5^(3/2)/2)(2/√5) = 1 - 5^(3/2)・5^(-1/2) = -4
  y0 = 1 + (5^(3/2)/2)(1/√5) = 1 + (5^(3/2)/2)・5^(-1/2) = 1 + 5/2 = 7/2
 また
  R^2 = 5^3/4 = 125/4
なので x = 1 における y = x^2 の曲率円の方程式は
  (x+4)^2 + (y-7/2)^2 = 125/4
 図を描いたらいい線いってると思ったのですが、正しくないようです。
 どこがおかしいのでしょうか。
0592589
垢版 |
2019/12/01(日) 20:48:04.38ID:uBpTHkZU
自己解決しました。
0594132人目の素数さん
垢版 |
2019/12/02(月) 01:44:21.53ID:Kr9ku+H8
f_n(x)=x/nとする。
不等式
f_n(x) < π < f_n+1(y)
を満たす自然数x,yのうち、|x-y|を最小にするものを(x,y)=(a[n],b[n])とおく。
|a[n]-b[n]|を最大にする自然数nを求めよ。
0595591
垢版 |
2019/12/02(月) 07:11:11.22ID:ljHVrsIS
自己解決しました。正しかったようです(笑)。
0596132人目の素数さん
垢版 |
2019/12/02(月) 19:55:45.85ID:Kr9ku+H8
各項が1または2のいずれかの値のみをとる無限数列のうち、周期を持たないもの全体からなる集合をSとする。

ただし数列{a[n]}が周期を持つとは、「ある自然数の定数Kとpが存在して、K以上のすべての自然数iについてa[i+p]=a[i]が成立する」
ことを指す。

(1)Sの要素のうち、「ある項から2020項連続して同じ数が続くことはない」ような数列の例を1つ挙げ、その一般項を述べよ。

(2)Sの要素のうち、「2020以下の任意の自然数jについて、ある項からちょうどj項連続して同じ数が続く部分がある。一方、2021以上の任意の自然数kについて、ある項からちょうどk項連続して同じ数が続くことは決してない」ような数列の例を1つ挙げ、その一般項を述べよ。
0597132人目の素数さん
垢版 |
2019/12/03(火) 17:43:40.23ID:LZHB9q0E
0<p<1/2なる有理数pを1つ自由に選び、そのpに対し以下の定積分を計算せよ。
∫[0→pπ] xln(sin(x)) dx
0598132人目の素数さん
垢版 |
2019/12/04(水) 17:41:25.12ID:Y6beWzZs
次の数列の初項から第n項までの和Snを求めよ。

(2)1/3,1/6,1/10,1/15,・・・・・・

先生、bn=1/(n+2)、あとわかりません。お教えください。誠によろしくお願い致します。
0599132人目の素数さん
垢版 |
2019/12/04(水) 18:44:07.05ID:KlQoN5fe
1+1/3+1/6+1/10+1/15=Σ[k=1,n]2/(k(k+1))
Σ[k=1,n]2/(k(k+1))=2Σ[k=1,n](1/k-1/(k+1))=2n/(n+1)
1/3+1/6+1/10+1/15=2n/(n+1)-1=(n-1)/(n+1)
nをずらして1/3+1/6+1/10+1/15=2n/(n+1)-1=n/(n+2)
0601132人目の素数さん
垢版 |
2019/12/05(木) 01:49:00.21ID:g/xUy6Rb
xyz空間の3点O(0,0,0),A(2,0,0),B(2cosθ,2sinθ,0)を頂点とする△ABCを底面とする三角柱Tがある。
Tは領域0≦z≦kの部分に含まれ、kは十分大きい。

T内に点P(cosα,sinα,p)をとり、∠OPA=π/3、∠APB=π/4となるようにする。

このような実数α、pをθの式で表せ。
0602132人目の素数さん
垢版 |
2019/12/05(木) 13:37:52.67ID:4zgD7nuM
へんな問題
0603132人目の素数さん
垢版 |
2019/12/05(木) 19:39:35.72ID:3Kjl+Ehp
xe^x=1の解αが
-(3+√17)/2<α<1/√3
を満たすのってどうやったら示せますか?
0604132人目の素数さん
垢版 |
2019/12/05(木) 19:40:13.59ID:3Kjl+Ehp
間違えました
(-3+√17)/2<α<1/√3 です
0605132人目の素数さん
垢版 |
2019/12/05(木) 20:05:00.34ID:3Kjl+Ehp
後、m,n∈ℕ
Im,n=∫[0→1]x^m・(logx)^ndx
=1/(m+1)∫[0,1](x^(m+1))'・(logx)^ndx
=1/(m+1)[x^(m+1)・(logx)^n][0→1]
-n/(m+1)∫[0,1]x^m・(logx)^(n-1)dx
=-n/(m+1)・Im,n-1
={-n/(m+1)}・{-(n-1)/(m+1)}・…・{-1/(m+1)}Im,0
={(-1)^n・n!}/(m+1)^(n+1)
∴1/(m+1)^(n+1)=(-1)^n/n!・Im,n
∴Σ[k=1,m]1/(k+1)^(n+1)
=(-1)^n/n!・Σ[k=1,m]Ik,n
=(-1)^n/n!・∫[0,1]x(1-x^m)(logx)^n/(1-x)dx

∴Σ[k=1,∞]1/(k+1)^(n+1)
=(-1)^n/n!∫[0,1]x(logx)^n/(1-x)dx

は合ってますか?
0607132人目の素数さん
垢版 |
2019/12/06(金) 01:14:43.96ID:sPB1n2Xg
>>601
定まるわけ無いやン
0608132人目の素数さん
垢版 |
2019/12/06(金) 01:17:42.67ID:sPB1n2Xg
>>601
>∠OPA=π/3
Pはとある球面上の点
>∠APB=π/4
Pはまた別のとある球面上の点
2球面の交線の円上の点は無数にある
0609132人目の素数さん
垢版 |
2019/12/06(金) 01:18:28.96ID:sPB1n2Xg
>>601
>ID:g/xUy6Rb
適当な出題をしてもしょうも無いだけだよ
0610132人目の素数さん
垢版 |
2019/12/06(金) 05:12:39.35ID:vGhbHWKK
>>603
e > 2.7 = (3/2) * 1.8 > (3√3)/2,
 e^(2/√3) > (2/√3)e > 3,
 e^(1/√3) > √3,
 f(1/√3) = (1/√3)e^(1/√3) > 1 = f(α),
f(x) = x・e^x は単調増加だから
 α < 1/√3,

>>606
(L,m,n) = (1,1,1) (4,2,2)
0613132人目の素数さん
垢版 |
2019/12/06(金) 12:29:20.42ID:fhqc4qJe
Xを位相空間として、f:X→C(R^n,R^m) x→f_xを連続写像とします。
ここでC(R^n,R^m)はR^nからR^mへの線形写像全体とします。
C(R^n,R^m)の位相はR^nとR^mの基底を適当にとってR^nmから定まるものとします。

このとき、g:X×R^n→R^mを(x,y)→f_x(y)で定めると連続写像になりますか?

なるなら証明を、ならないなら反例をお願いします。
0615132人目の素数さん
垢版 |
2019/12/06(金) 13:42:34.05ID:b7GC2q2H
ああ、意味わかった。
C(R^n,R^m)なんてただの行列の空間やん。
C(R^n,R^m)×R^n→R^nなんてただの行列の積の写像なだから連続なの当たり前。
0616132人目の素数さん
垢版 |
2019/12/06(金) 13:55:48.95ID:1Nlu71Qy
わかってない予感
0617132人目の素数さん
垢版 |
2019/12/06(金) 14:13:22.03ID:a81dp8ZB
xyz空間に4点
O(0,0,0), A(1,0,0), B(0,1,0), C(1,1,0)がある。

四角形OACBを直線ABを軸として一回転させてできる立体をV1, 四角形OACBを直線OCを軸として一回転させてできる立体をV2とするとき、V1とV2の共通部分の体積Tを求めよ。

円錐面の方程式出して、z=t平面でやろうとしたら上手くいかないです。
0622132人目の素数さん
垢版 |
2019/12/06(金) 16:03:11.48ID:4nXDP5CI
>>612
これの(1)って、有名な水汲みパズルの応用だね。
Aを出発して、川に着いたら川から水を汲んでBに向かうとき、
歩く距離を最短にするには川に向かってどう歩けばよいか、というやつ。

この問では川は y=pで表される直線で、川幅は0と見なす。

p≦2 或いは p≧4 のときは、A、B は同じ側の川岸の2点、
2<p<4のときは、A、Bは川の対岸同士にある2点。
ただこの問ではPHが加わるので、 p の正負による場合分けも生じる。
0623132人目の素数さん
垢版 |
2019/12/06(金) 17:00:25.29ID:fw9UG0Kd
>>622
ありがとうございます。
なぜ距離の和dをxで微分し、d'が定義できないか0になる点が最小値を与える、という考えでは正答が出なかったのでしょうか?
0626132人目の素数さん
垢版 |
2019/12/06(金) 18:16:43.58ID:fw9UG0Kd
ん?と思ったけど符号変化関係あるんですかね
x→±∞でd=∞はわかっているのだから
d'=0かd'が計算できない(±∞になる)xを全て調べればそこに必ず最小値が含まれている、と思ったのですが
0627132人目の素数さん
垢版 |
2019/12/06(金) 19:31:12.30ID:u4LjGQnh
>>618
4つだと思います。一応、対称性から楽しようとしたのですが、領域がなぜかうまく絞り込めません…
0628132人目の素数さん
垢版 |
2019/12/06(金) 20:28:00.54ID:/GA8bJMY
x+y=1との共通面をA、x=yとの共通面をBとする。
(0,0)とAの凸包X、
(1,1)とAの凸包Y、
(1,0)とBの凸包Z、
(0,1)とBの凸包W
とする。
XとZの共通域のうちy≦x、x≦1/2の部分を求める。
x=1/2,y≦1/2と領域の共通部分の面をCとして
(0,0)、CはX,Zの両方に含まれるからその凸包はX、Zの両方に含まれる。
逆に領域のy≦x、x≦1/2の部分はこの凸包に含まれる。
以上により該当部分は(0,0)とCの凸包。
0629132人目の素数さん
垢版 |
2019/12/06(金) 20:38:12.19ID:5yJQhmHx
>>440
最後にx=a(p-2)/2(p-3), (p-2)/2のどちらを取るかで不等式解き間違えてるだけでは?
冷静に考えれば0<(p-2)/(p-3)<2が有り得ない訳ないでしょ
0630132人目の素数さん
垢版 |
2019/12/06(金) 21:32:41.32ID:JL7GznQx
整式P(x)をx^(2)+1で割れば-5x-10余り、x-2で割れば-5余る。P(x)を(x^(2)+1)(x-2)で割った余りを求めよ。

P(x) = (x^(2)+1)*A(x)-5x-10 = (x^(2)+1)*A(x)-5(x-2)-20 = (x^(2)+1)(x-2)*B(x)-5(x-2)-5 = (x^(2)+1)(x-2)*B(x)-5x+5
ゆえに余りは-5x+5

これは間違いですが具体的にどこが間違いなのでしょうか?
0632132人目の素数さん
垢版 |
2019/12/07(土) 00:30:03.24ID:uyA7rt4S
>>612
(1)
0≦p≦4のときp+√(a^2-4a+20)
p≧4のときp+√{(2p-4)^2+(a-2^2)}
p≦0のとき-p+√{(2p-4)^2+(a-2^2)}
(2)
√(a^2-4a+20)

でいいのかな?
0634132人目の素数さん
垢版 |
2019/12/07(土) 03:43:49.83ID:YKmiJP8B
>>620
直感的には分かるのですが、積を与える写像が連続であることの証明がわからないです
0635132人目の素数さん
垢版 |
2019/12/07(土) 07:31:52.91ID:qIKSSiPZ
>>634
成分個別にR^mnで考えるって書いてるやン
それらの積と和で書かれるんだから連続になるの当たり前
それともR×R→Rの(a,b)→abと(a,b)→a+bの連続性から証明したいということ?
0636132人目の素数さん
垢版 |
2019/12/07(土) 12:10:52.99ID:/GHyGODE
一辺の長さが2の立方体ABCD-EFGHがある。
その面上または内部に含まれる長さ1の線分Lを考え、その両端点をP,Qとする。
Lが色々動くとき、以下の長さの和の最大値と最小値を求めよ。

(AP+PG)+(BQ+QH)
0638132人目の素数さん
垢版 |
2019/12/07(土) 15:47:00.24ID:8M7oXPTh
おっぱいに三角形を書くと内角の和が180度にならないと聞いたのですが本当ですか?
0639132人目の素数さん
垢版 |
2019/12/08(日) 01:06:04.30ID:nrFfxmSw
直積環R*Rに対して
(R*R)[T]の可逆元全体とR*R可逆元全体は一致するか
また一致しない場合(R*R)[T]の可逆元全体はどんな集合か
0641132人目の素数さん
垢版 |
2019/12/08(日) 15:17:06.47ID:nrFfxmSw
不定元ですのでXと書いた方が良かったですね
R[X]でR上の一変数多項式環です
0642132人目の素数さん
垢版 |
2019/12/08(日) 15:45:41.57ID:AKQ+qoBm
なんで直積で考えるのか
任意の環R上の多項式環R[X]の可逆元はRの可逆元でしょ
逆は明らかだし
0643132人目の素数さん
垢版 |
2019/12/08(日) 19:52:53.91ID:1RPIXi6U
それは言えないのでは
簡単な例だとa∈R,a^k=0を満たすとき
1=1-(aX)^kより1-aXはR[X]の可逆元だけどRの可逆元とは限らない
0644132人目の素数さん
垢版 |
2019/12/08(日) 20:00:10.35ID:1RPIXi6U
一致するのは整域のときだけでR×Rは聖域ではないから反例が出てくる
具体的な反例は知らん
0645132人目の素数さん
垢版 |
2019/12/08(日) 20:01:54.67ID:GND5Qdtj
とりあえず

R×R → R

(a,b) →  a+b

で考えようか
0646132人目の素数さん
垢版 |
2019/12/08(日) 23:34:04.66ID:IF2loAVY
一般に
R[X]上の元が可逆
⇔定数項が可逆かつ一次以上の係数が全てベキ零元
は言える。

←は明らか。
ある係数が冪令でないが、可逆なものが取れたとする。
その元と逆元の係数だけから生成される部分環に制限してよいからネーター環として良い。
n次の係数aが冪零でないとする。
R[1/a]の素イデアルの引き戻しPをとるとa+PはR/Pの非冪零元でR/P[X]の可逆元の係数となるからRは整域として良い。
しかし整域の場合はR[X]の可逆元は定数項が可逆元で一次以上の係数が全て0である場合に限られるから矛盾。

でもこれが答えといっていいのかは疑問。
こんなもん必要十分条件なんかいくらでもあるだろうし。
大学数学以上のテーマでこの手の問題は成立しないと思う。
本来受験数学でも必要十分条件求めよ系は危ないけど、それはこの手の問題の答えはこの形という暗黙の了解があるからギリギリ許されてるだけで、大学の数学以上ではそんなもん存在しないと思う。
0647132人目の素数さん
垢版 |
2019/12/08(日) 23:54:34.29ID:GND5Qdtj
R⊆R[X]より

Rで言えることはR[X]ですべて言える
というのもR[X]の元定数多項式をRの元と看做せばよいから
0649132人目の素数さん
垢版 |
2019/12/09(月) 01:17:25.23ID:jWKKnttG
[x]でxを超えない最大の整数を表す。
Nを自然数の定数とし、数列a(n)を以下のように定める。

a(0)=N
a(n+1)=a(n)+1-[a(n)/2]

(1)lim[n→∞] a(n) は収束することを示せ。

(2)Nの値により、n→∞でのa(n)の極限値が変化するかを判定せよ。
(変化する場合、各極限値に対応するNを具体的に記述しなくてよい)
0650132人目の素数さん
垢版 |
2019/12/09(月) 02:03:30.33ID:aToMvgjC
>>647
Sを実数体とするとき
・Sの0ではない元は単数である

これと
>Rで言えることはR[X]ですべて言える
という壮大な理論の系として、
・S[X]の0ではない元は単数である
という定理が得られる
0652132人目の素数さん
垢版 |
2019/12/09(月) 02:44:50.73ID:E2WZGv9A
そうですね
体上の多項式環の例として実数体の多項式です

R×R  → (同型)R^2[X]

 ↓       ↑

 R   → (包含写像)
0653132人目の素数さん
垢版 |
2019/12/09(月) 02:47:18.16ID:E2WZGv9A
>>652
間違えたw
0655132人目の素数さん
垢版 |
2019/12/09(月) 02:54:12.04ID:E2WZGv9A
R×R  → (同型写像)R^2 → (包含写像)R^2[X]

 ↓       ↑

 R   → (包含写像)

それなので
実数体Rで成り立つことは
R^2係数多項式全体R^2[X]
でも成立します
0656132人目の素数さん
垢版 |
2019/12/09(月) 07:00:52.62ID:JdEMhXhJ
Excelでsum ifって3d参照使えないんらしいんですが、3d参照っぽく使う方法って何かありますか?
0659132人目の素数さん
垢版 |
2019/12/09(月) 20:33:30.33ID:RtADCnwt
x,yが自然数で,(x^2+1)/y も (y^2+1)/x も自然数なら
(x^2+y^2+1)/(xy) も自然数といえますか?
0661132人目の素数さん
垢版 |
2019/12/09(月) 21:07:48.61ID:jWKKnttG
(ア)l≦mとする。
l!+m!=n!を満たす自然数l,m,nをすべて求めよ。

(イ)k≦lかつm≦nとする。
k!+l!=m!+n!が成り立つような自然数k,l,m,nで、k≠mかつl≠nを満たすものは存在しないことを証明せよ。
0662659
垢版 |
2019/12/09(月) 21:18:25.96ID:RtADCnwt
ええっ?
そんなに即答が返ってくるくらい簡単なことなのですか。
有名な事実なんでしょうか。
0664659
垢版 |
2019/12/09(月) 22:17:13.36ID:RtADCnwt
・・・あら、ほんまですわ。
いやお恥ずかしい (〃∀〃)ゞ
0665132人目の素数さん
垢版 |
2019/12/09(月) 23:51:56.46ID:qsm3npU8
定積分∫[-π/2,π/2]tan^3x/(cosx^sinx+1)dxを求めよ。
どうやってとけば良いですか?
0666132人目の素数さん
垢版 |
2019/12/09(月) 23:54:41.30ID:qsm3npU8
間違えました
cos^3x/(cosx^sinx+1)です
0667132人目の素数さん
垢版 |
2019/12/09(月) 23:57:00.39ID:WPDpIZNQ
>>666
おまえ、なんで便所行ったときに手を洗わないの?
0676132人目の素数さん
垢版 |
2019/12/10(火) 11:25:19.01ID:nNLGby/V
>>666
誰かお願いします
0677132人目の素数さん
垢版 |
2019/12/10(火) 11:31:40.57ID:/A9ufgZe
>>649
(1)
a(n) が自然数であれば(n≧0)、
a(n)が偶数のとき、a(n+1)={a(n)+2}/2 (2以上の自然数)
a(n)が奇数のとき、a(n+1)={a(n)+3}/2  (  〃   )

a(0)=1だと、a(1)=2だが、a(i)=2だとa(i+1)=2 なので a(n)は2に収束
a(0)=2だと、a(1)=2となり、上記よりやはりa(n)は2に収束
a(0)=3だと、a(1)=3だが、a(i)=3だとa(i+1)=3なので、a(n)は3に収束
Nが3以下であれば、a(n)は2か3に収束する。

Nが4以上の場合、
N=2k (k>1)の場合、a(1)=k+1 < 2k となり、a(1)はNより小となる。
N=2k+1(k>1)の場合、a(1)=k+2 < 2k +1 =N となり、やはりa(1)はNより小。
よって、仮にNがM以下のすべての値でa(n)が収束するのであれば、N=M+1
としても、a(1)≦Mとなり、これはN=a(1)とした場合と同じ値に収束する。
ゆえに、数学的帰納法により、すべての自然数Nに対してa(n)は収束する。

(2)N=1,2であれば極限値は2、N=3以上の場合は極限値は3となる。
 ∵極限値が2となるのは、あるi(≧1)でa(i)=2となる場合だが、
  a(i-1)=2a(i)-2またはa(i-1)=2a(i)-3 より、a(i-1)=2または1
  となるので、a(0)=N は1または2しかありえない。
0678132人目の素数さん
垢版 |
2019/12/10(火) 12:56:28.99ID:YF04UXDx
杉浦光夫著『解析入門I』を読んでいます。

ダルブーの定理の証明ですが、p.215に

「(3.9)により 0 ≦ n_k ≦ n である。」

と書いてあります。これって間違っていませんか?

「(3.9)により 0 ≦ n_k ≦ 1 である。」

が正しいと思いますが、どうですか?
0679132人目の素数さん
垢版 |
2019/12/10(火) 14:49:59.27ID:nDYrVseb
投げたときに表が出る確率がpのコインがある。表が出たら勝ち、裏が出たら負けとすると、n回連続でコインを投げたときk連敗以上する確率を求めよ。

確率に自信ニキいましたらよろしくお願いします。
0680132人目の素数さん
垢版 |
2019/12/10(火) 15:06:27.35ID:Gmq3JIZH
>>678
(3.9)は各辺につき高々1個と言っているので、
n次元区間だとその総数は高々n個という意味
0681132人目の素数さん
垢版 |
2019/12/10(火) 15:50:14.81ID:9+9M8wAb
>>657

1/x - √(1/x^2 - x^2) = {1 - √(1-x^4)} /x = x^3 /{1 + √(1-x^4)},

(与式) = ∫[0,1] x^3 dx /{1 + √(1-x^4)}
 = (1/2)∫[0,1] y/(1+y) dy   {← y=√(1-x^4) }
 = (1/2)∫[0,1] {1 - 1/(1+y)} dy
 = (1/2)[ y - log(1+y) ](y=0,1)
 = (1/2){1 - log(2)}
 = 0.15342640972
0683132人目の素数さん
垢版 |
2019/12/10(火) 20:57:08.17ID:NpnzYFbS
>>666
これ考えてくださった方いたら申し訳ないのですが自己解決しました
自己解決しました
一応、
∫[-π/2,π/2](cosx)^3/cosx^(sinx)+1dx…A
=∫[0,π/2](cosx)^3/cosx^(sinx)+1dx
+∫[-π/2,0](cosx)^3/cosx^(sinx)+1dx

ここで、x=-tとするとdx=-dt
このとき、
∫[-π/2,0](cosx)^3/cosx^(sinx)+1dx
=∫[t=π/2,t=0](cos(-t))^3/cos(-t)^(sin(-t))+1(-dt)
=∫[0,π/2](cost)^3/cost^(-sint)+1dt
=∫[0,π/2](cosx)^3/cosx^(-sinx)+1dx

∴A=∫[0,π/2](cosx)^3・(1/(cosx^(sinx)+1)+1/(cosx^(-sinx)+1))dx
=∫[0,π/2](cosx)^3dx
=∫[0,π/2]cosx(1-(sinx)^2)dx
=∫[0,π/2](cosx-(sinx)'(sinx)^2)dx
=sinπ/2-1/3・(sinπ/2)^3
=1-1/3=2/3

になりました(計算ミスしてたらすいません…)
0688132人目の素数さん
垢版 |
2019/12/11(水) 00:18:45.56ID:FCybO3Mj
途中でcos(x)をtan(x)と打ち間違いしたと気づいて訂正したとかいってるね。
どう間違ったらその誤写はできるものやら。
0691132人目の素数さん
垢版 |
2019/12/11(水) 04:27:33.67ID:NyJ5MAli
>>599
わからないのと難しいのを恐れいて遅くなりました。まず、初項が1/1というのを求めるのがみそだったんですね。
そして、第1+1項から第n+1項まで求める。
先生、本当に助かりました。ありがとうございました。
0692132人目の素数さん
垢版 |
2019/12/11(水) 06:17:13.00ID:NyJ5MAli
>>403
先生、素晴らしいです。よくこんな空間からメネラウスの定理が使える平面をイメージなされました。
まさかわかる方がいらっしゃるとは思いませんでした。横柄な態度を取って誠に申し訳ありませんでした。
0694132人目の素数さん
垢版 |
2019/12/11(水) 08:08:36.68ID:hDrsdm2G
>>689
こういう応用問題も計算可能になる。

(問) 表の出る確率がpの歪(いびつ)なコインを100回投げたときに表が連続した数の最大値が4であった場合のpの期待値・モード値と95%信頼区間を求めよ。

pの確率密度関数を図示すると
https://i.imgur.com/3JwdM8g.jpg
0695132人目の素数さん
垢版 |
2019/12/11(水) 08:17:54.50ID:RCvw2MiZ
>>694
確率の確率なのねw
0696132人目の素数さん
垢版 |
2019/12/11(水) 10:27:53.71ID:VLSxIs0+
>>666

1/(y+1) + 1/(1/y + 1) = 1/(y+1) + y/(1+y) = 1,

1/{cos(x)^(sin(x)) + 1} + 1/{cos(-x)^(-sin(x)) + 1} = 1,

∫cos(x)^3 dx = ∫{(3/4)cos(x) + (1/4)cos(3x)} dx
 = (3/4)sin(x) + (1/12)sin(3x)
 = sin(x) - (1/3)sin(x)^3
0697132人目の素数さん
垢版 |
2019/12/11(水) 11:43:09.17ID:X7w93S94
質問です。
証明がわからなんじゃなくて誰の定理だったか知りたいのです。

有限集合A,Bと写像F:A→2^Bが与えられていて任意のAの部分集合Xに対して
#{b∈B | ∃a∈X b∈F(a)}≧#X
が成り立つとき単射f:A→Bをf(x)∈F(x)となるように取れる。

これ誰の定理でしたっけ?
0699132人目の素数さん
垢版 |
2019/12/11(水) 13:17:33.72ID:hDrsdm2G
>>695
コインの裏表じゃ面白くないけど
こういう問題にしてみると臨場感があるね。

今だと、桜を見る会の反社勢力率とか、風俗嬢の梅毒感染率とかに置き換えると面白い。

私立医大の裏口入学調査団が100人を順次調査した。
裏口入学判明人数をそのまま公表はヤバすぎる結果であったため、
連続して裏口が入学がみつかった最大数は4人であったとだけ公表した。
公表結果が正しいとして裏口入学人数の期待値、最頻値、及び95%信頼区間を述べよ。
0700132人目の素数さん
垢版 |
2019/12/11(水) 14:09:16.35ID:VLSxIs0+
x,yが自然数で,(x^2+1)/y も (y^2+1)/x も自然数なら
(x^2+y^2+1)/(xy) = 3 といえますか?
0703132人目の素数さん
垢版 |
2019/12/11(水) 14:42:28.57ID:VLSxIs0+
フィボナッチ数を使って表わせば
 (F_{2n+1})^2 + 1 = F_{2n-1} F_{2n+3}
より
 (x, y) = (F_{2n-1}, F_{2n+1}),

 (x^2 +1)/y = F_{2n-3},
 (y^2 +1)/x = F_{2n+3},
よって
 (xx+yy+1)/xy = F_{2n-3} F_{2n+3} - F_{2n-1}F_{2n+1}
  = (3F_{2n-1} -F_{2n+1}) (3F_{2n+1} -F_{2n-1}) - F_{2n-1} F_{2n+1}
  = 3[ 3F_{2n-1} F_{2n+1} - F_{2n-1}^2 - F_{2n+1}^2 ]
  = 3,
0704132人目の素数さん
垢版 |
2019/12/11(水) 14:47:47.16ID:X7w93S94
>>703

> フィボナッチ数を使って表わせば
>  (F_{2n+1})^2 + 1 = F_{2n-1} F_{2n+3}
> より
>  (x, y) = (F_{2n-1}, F_{2n+1}),

なぜですか?
0705132人目の素数さん
垢版 |
2019/12/11(水) 16:16:35.60ID:hDrsdm2G
>>692
イナ氏の計算力と粘着力には感服しております(皮肉ではありません)。
つくづく東大に行かなくてよかったと思う(某二期校に逃げました)。
0706132人目の素数さん
垢版 |
2019/12/12(木) 07:36:42.40ID:7ocXyweL
>>700

(x^2+y^2+1)/(xy) = 2 + {(x-y)^2 +1}/(xy) > 2,

一方、一つ飛ばしのフィボナッチ数については
 1/φ^2 < y/x < φ^2,
よって
(x^2+y^2+1)/(xy) = x/y + y/x + 1/(xy)
 < φ^2 + 1/φ^2 + 1
 = 3 + (φ - 1/φ)^2
 = 3 + 1
 = 4,

左辺は自然数だから >>659-664
 (x^2+y^2+1)/(xy) = 3,
0708132人目の素数さん
垢版 |
2019/12/12(木) 10:33:38.91ID:5AHNk2TW
6辺の各々の長さが整数値で、
4つの面の各々の面積が整数値で、
体積が整数値である。

このような四面体が存在することを証明せよ。
0710132人目の素数さん
垢版 |
2019/12/12(木) 16:40:43.41ID:flOpkEvS
1回3.6%で激レアが出るガチャを10回回した確率って
36%なのでしょうか?
それとも0.964*0.964*0.964(略 0.964を10回電卓にかけた数なのでしょうか?
教えてください。
0711710
垢版 |
2019/12/12(木) 17:06:19.10ID:flOpkEvS
すみません、くだらねぇ質問板に書きます。
0714132人目の素数さん
垢版 |
2019/12/12(木) 19:04:33.45ID:7ocXyweL
>>707
 {(F_{2n-1})^2 + 1} / F_{2n+1} = F_{2n-3} = 自然数
 {(F_{2n+1})^2 + 1} / F_{2n-1} = F_{2n+3} = 自然数
は周知だから
 (x, y) = (F_{2n-1}, F_{2n+1}),
とおける。   >>703
0715132人目の素数さん
垢版 |
2019/12/12(木) 19:30:47.20ID:FIUHqke0
>>714
> (x, y) = (F_{2n-1}, F_{2n+1})

> x,yが自然数で,(x^2+1)/y も (y^2+1)/x も自然数
と、
> (x, y) = (F_{2n-1}, F_{2n+1})

> (x^2+y^2+1)/(xy) = 3
は示されているが、

> x,yが自然数で,(x^2+1)/y も (y^2+1)/x も自然数

> (x^2+y^2+1)/(xy) = 3

> x,yが自然数で,(x^2+1)/y も (y^2+1)/x も自然数

> (x, y) = (F_{2n-1}, F_{2n+1})
は示されていないんじゃないか?
0717132人目の素数さん
垢版 |
2019/12/12(木) 20:08:37.17ID:2DJln6HR
とりあえず>>700の解答としては

x^2+y^2-nxy+1=0
としてx=yであるときは
(n-2)x^2=1
となるのでn=3のとき意外解を持ち得ない。
x>yである解が存在したしてxが最小であるものをとるとき(y,(y^2+1)/x)も解になるので最小性に矛盾する。

で言える。
0718132人目の素数さん
垢版 |
2019/12/12(木) 22:20:16.29ID:7ocXyweL
nを固定して考えると・・・・
(x, y) が解ならば (y, (y^2+1)/x) も解。
x>y ならば、
 (y^2 +1)/x ≦ (y^2 +1)/(y+1) ≦ y
(x,y)≠(2,1) ならば不等号となって最小性に反する。
nに関して解があるなら、最小解は (x,y)=(1,1) に限る。

でござるか
0720132人目の素数さん
垢版 |
2019/12/13(金) 01:11:08.12ID:Tg/EfDMa
任意の実数xについてf"(x)>0のとき、
n∈ℕで
nΣ[k=0,n]f(2k)>(n+1)Σ[k=1,n]f(2k-1)
が成り立つことを示せ。
どうやって解けば良いのでしょうか…
0722132人目の素数さん
垢版 |
2019/12/13(金) 11:27:29.68ID:6gtv5jiW
>>720

f "(x) >0 から凸不等式
 {f(2k-2) + f(2k)}/2 > f(2k-1)  ・・・・ (1)
また
 {(n-k)f(0) + k・f(2n)}/n > f(2k),
 {k・f(0) + (n-k)f(2n)}/n > f(2n-2k),
辺々たすと
 f(0) + f(2n) > f(2k) + f(2n-2k)  ・・・・ (2)

これらより
 左辺 - 右辺 > nΣ[k=0,n] f(2k) - (n+1)Σ[k=1,n] {f(2k-2)+f(2k)}/2
 = (1/2)(n-1){f(0) + f(2n)} - Σ[k=1,n-1] f(2k)
 = (1/2)Σ[k=1,n-1] {f(0) + f(2n) - f(2k) - f(2n-2k)}
 > 0,
0723132人目の素数さん
垢版 |
2019/12/13(金) 16:34:33.71ID:jmwF4mt7
>>708
これどなたかお願いします
等面四面体で何とかならないかと計算中ですが難しいです
0725132人目の素数さん
垢版 |
2019/12/14(土) 09:03:04.67ID:xRSTix9N
a,b,cを正の実数とする。
空間の点A(a,0,0),B(0,b,0),C(0,0,c)を通る平面π上にあり、点Cを中心とする半径1の円Kを考える。
K上の点(x,y,z)で、xy+yz+zxを最大にするものの座標をa,b,cで表せ。
0728132人目の素数さん
垢版 |
2019/12/14(土) 17:15:58.20ID:+YP0IWDD
>>726
おるよ
0729132人目の素数さん
垢版 |
2019/12/14(土) 17:34:23.32ID:yPAFUFSH
C:y=1/x (x>0)上の異なる2点P, QがPQ=1をみたしながら自由に動くとき、線分PQが通過する領域のx=tにおける最大値をf(t)とする。
0<α<β, βは定数
S(α)=∫[α→β](f(x)-(1/x))dxとするとき
lim[α→0]S(α)は収束するのでしょうか?

また、収束するようなPQの値の範囲はどうなるのでしょうか。
ご教示ください
0730132人目の素数さん
垢版 |
2019/12/14(土) 18:30:04.14ID:Z4g9QIZJ
あるソシャゲの10連ガチャの排出率に関して、
10連ガチャを実行した場合に★4以上が1個確定と
表記されておりますが、単発での排出率が下記の割合となってます。

----------------------------------
★5 1.5%
★4 13.5%
★3 85%
----------------------------------

ここで過程として、
★4が不要となり★5のみ必要となった場合に
単発を実行するほうが当たりやすいのか
もしくは、10連を実行するほうがあたりやすいのか
どちらか判断ができない為、ご教授をお願いします。

また、この様な場合にどのように計算したらいいのか
記載してもらえると幸いです。ご回答宜しくお願い致します。
0732132人目の素数さん
垢版 |
2019/12/14(土) 18:42:38.88ID:UtwXwdkG
>>730
>10連ガチャを実行した場合に★4以上が1個確定と
の仕様がわからないと、計算のしようがないと思う。
0733132人目の素数さん
垢版 |
2019/12/14(土) 18:55:48.60ID:wdo3TLaX
|z|=1 とき, z^n→-1 となることはあるか?

ないと思うんですが,うまく示せなくて困っています。
お願いできたら幸いです。
0734132人目の素数さん
垢版 |
2019/12/14(土) 19:12:47.77ID:QpFD+FzG
重みによるデータの平滑化の方法を聞きたいです。
問題はx(t)、 t=1〜5

データ
x(1)=2,x(2)=7,x(3)=欠損値,x(4)=5,x(5)=3

重み
w(t-2)=0.5,w(t-1)=2,w(t)=4,w(t+1)=2,w(t+2)=0.5

これの欠損値を求めろという問題です。
0735132人目の素数さん
垢版 |
2019/12/14(土) 19:14:46.46ID:UtwXwdkG
>>730
>10連ガチャを実行した場合に★4以上が1個確定と表記されております
を次のように解釈する。
すなわち、
10連ガチャを選択すると1個確定する★4以上とは
★5と★4は1.5 : 13.5 の比率で出る。
つまり、10連ガチャでの★5は1.5/(1.5+13.5)=0.1の確率ででる。
単発を10回繰り返すときに1回でも★5がでる確率は1-(1-1.5/100)^10= 0.1402696
ゆえに、
単発を実行するほうが当たりやすい
0736132人目の素数さん
垢版 |
2019/12/14(土) 19:20:54.71ID:Z4g9QIZJ
>>735
すごい助かりましたありがとうございました!
0738132人目の素数さん
垢版 |
2019/12/14(土) 21:00:21.55ID:2yORFwv1
10連ガチャは10回やって★4以上が2個以上出ることはないという設定なの?
0739132人目の素数さん
垢版 |
2019/12/14(土) 21:10:09.72ID:+YP0IWDD
>>733
極限のつもりならz=1のときのみz^n→1
それ以外は収束しない
0740132人目の素数さん
垢版 |
2019/12/14(土) 23:28:08.44ID:gXafMNq+
>>739
極限です。ちなみにzは複素定数です。
一応ε−N論法で示せましたが、もっと簡単に示す方法はありますでしょうか。
0744132人目の素数さん
垢版 |
2019/12/15(日) 02:35:01.51ID:x9fURbad
>>738
でますよ。あくまで10個目が★4以上確定というだけです。
0745132人目の素数さん
垢版 |
2019/12/15(日) 07:11:18.83ID:7sTpxzy0
>>744
10連のとき最初の9個は★5,★4,★3から選ばれる
その比率は1.5:13.5:85
10個目は★5,★4から(1.5:13.5の比率で)選ばれるという仕様とすると

10連で★5が全く含まれない確率pは((100-1.5)/100)^9 * 13.5/(1.5+13.5)
少なくとも1個の★5が含まれる確率は1-p=0.214となり、10連の方が有利になる。
0746132人目の素数さん
垢版 |
2019/12/15(日) 07:31:30.06ID:7sTpxzy0
>>745

その仕様で100万回シミュレーション

> sim10 <- function() {
+ gacha10 = c(sample(c(3,4,5),9,replace=TRUE,prob=c(85,13.5,1.5)),sample(c(4,5),1,prob=c(13.5,1.5)))
+ any(gacha10==5)
+ }
> mean(replicate(1e6,sim10()))
[1] 0.214313
> sim1 <- function() {
+ gacha1 = c(sample(c(3,4,5),10,replace=TRUE,prob=c(85,13.5,1.5)))
+ any(gacha1==5)
+ }
> mean(replicate(1e6,sim1()))
[1] 0.140582
0747132人目の素数さん
垢版 |
2019/12/15(日) 07:44:18.05ID:W9tlAU4y
>>744
10個目は確定なの?
9個目までに★4以上が出なかったときだけ確定ってことではないの?
この場合でも確定のときの★5と★4の比率が1.5:13.5であるなら10連の方が期待値高くなるけど
0748132人目の素数さん
垢版 |
2019/12/15(日) 07:59:36.65ID:7sTpxzy0
>>747
9個目までに★4以上が出なかったときだけ10個目は★5,★4から比率1.5:13.5で選ぶという仕様でシミュレーション

> sim10a <- function(){
+ gacha9=c(sample(c(3,4,5),9,replace=TRUE,prob=c(85,13.5,1.5)))
+ if(max(gacha9)==3){
+ g10=sample(c(4,5),1,prob=c(13.5,1.5))
+ }else{
+ g10=sample(c(3,4,5),1,prob=c(85,13.5,1.5))
+ }
+ gacha10=c(gacha9,g10)
+ any(gacha10==5)
+ }
> mean(replicate(1e6,sim10a()))
[1] 0.159585
0749132人目の素数さん
垢版 |
2019/12/15(日) 12:58:12.74ID:H2WYBDRI
☆4確定なのに☆5の確率アップしたらおかしいだろ
1.5:98.5になるに決まってるだろ
0751132人目の素数さん
垢版 |
2019/12/15(日) 13:10:41.68ID:7sTpxzy0
>>748
数値計算すると

m3=0.85^9 # max(gacha9)==3
m4=(0.85+0.135)^9-0.85^9 # max(gacha9)==4
m5=1-(1-0.015)^9 #max(gach9)==5
m3*0.1+m4*0.015+m5

> m3*0.1+m4*0.015+m5
[1] 0.159957
0752132人目の素数さん
垢版 |
2019/12/15(日) 13:58:45.92ID:Nfo6ujPm
>>733
 lim[n→∞] x^n = -1 だったと仮定する。

 1 - x^(2n) = (1 + x^n) (1 - x^n)
で n→∞ としたとき
 1 - (-1) = {1+(-1)} {1-(-1)},
 2 = 0  ((矛盾)
0753132人目の素数さん
垢版 |
2019/12/15(日) 14:17:04.99ID:x9fURbad
なんかみなさんこんなくだらないことにありがとうございます。

ちなみに>>747の返答は
10個目が★4か★5
1個目から9個目までが★3〜★5です。
0754132人目の素数さん
垢版 |
2019/12/15(日) 14:50:31.91ID:a93Bvb/N
>>753
1〜9個目は★3 85%、★4 13.5%、★5 1.5%
1〜9個目で★4、★5が出ていても出ていなくても10個目は★4か★5(13.5:1.5)ってこと?
それなら★5の可能性は悩むまでもなく10連の方が高いに決まっとるじゃん
0755132人目の素数さん
垢版 |
2019/12/15(日) 15:11:19.16ID:LASktn7I
コインの表をH、コインの裏をTで表すとき、「コイン投げを無限回繰り返す」という試行の標本空間は{H,T}^∞となります。
この標本空間の濃度は連続体(つまり、Rや、R上の区間と同じ濃度を持つ)だったと記憶しているのですが、証明方法を思い出すことができません。
スケッチでもかまいませんので、証明を教えて頂けないでしょうか。
0756132人目の素数さん
垢版 |
2019/12/15(日) 15:48:35.80ID:Nfo6ujPm
区間 [0,1] に含まれる実数は2進法により無限小数として一意的に表わせる
かも。
0757755
垢版 |
2019/12/15(日) 15:57:45.05ID:LASktn7I
>>756
対角線論法ですね。
対角線論法でもう一度考えてみます。ありがとうございます。
0759132人目の素数さん
垢版 |
2019/12/15(日) 17:08:56.12ID:PRdnkv5o
>>756
>区間 [0,1] に含まれる実数は2進法により無限小数として一意的に表わせる

「一意的に」は嘘
連続性を保つ限り、一意性は有しない

0.1000…=0.0111…
0761132人目の素数さん
垢版 |
2019/12/15(日) 17:52:45.50ID:FFLFQcS/
>>729
どなたか、数学の得意な方、お願いします…
0762755
垢版 |
2019/12/15(日) 19:10:54.94ID:LASktn7I
>>756
対角線論法をヒントに色々探したのですが、
以下のURLにある資料の135ページに証明の詳細を見つけました。
math.aalto.fi/~kkytola/files_KK/ProbaTh2019/ProbaTh-2019.pdf

>>760
ベルンシュタインの定理でも大丈夫なんですね。こちらも考えてみます。

コメントをくださった皆様、ありがとうございました。
0763132人目の素数さん
垢版 |
2019/12/15(日) 19:51:24.86ID:1xZAPqJd
>>762
まぁ納得したならそれでいいけど対角線論法それ本体から言えるのは{0,1}^ωもRも可算無限ではないまでだけだけどね。
ぴったり同じっていうには不十分だよ。
0764132人目の素数さん
垢版 |
2019/12/16(月) 06:40:03.24ID:1OLu1m2e
aを0より大きく1より小さい実定数とする。

また、
(i)区間[0,a)に含まれ、2以上の自然数nを用いて1/nと表せる全ての有理数を要素とする無限集合をS
(ii)区間[a,1]に含まれ、同様に1/nと表せる全ての有理数を要素とする無限集合をT
と定める。

いま集合Sまたは集合Tに含まれる有理数を無作為に2つとり、それらの和をとる。
その和をxとおくとき、xが集合Xに含まれる確率P(X)について、
P(S):P(T) = a:(1-a)
が成り立つか。
結論を述べ、またその根拠を簡単に述べよ。
0765132人目の素数さん
垢版 |
2019/12/16(月) 11:31:51.08ID:csa12zwJ
>>752
それスマートですね。自分のε-Nによる1ページほどの答案が恥ずかしい…。
0766132人目の素数さん
垢版 |
2019/12/16(月) 12:01:16.05ID:/i4Cvb2a
>>725
a=b の場合は
 (x, y, z) = (a/√(2aa+4cc), a/√(2aa+4cc), c - 2c/√(2aa+4cc))
 最大値 2c/√(2aa+4cc) - a(4c-a)/(2aa+4cc),
0767132人目の素数さん
垢版 |
2019/12/16(月) 18:04:10.91ID:SF0nDj5O
Table[sum[C(2n-1+α,k-1),{n,1,a}],{k,1,b}]

a=n(n+1)/2-1 
b=n(n+1)

を満たす差分追尾数列αを見つけてくれ〜(・ω・)ノ
0768132人目の素数さん
垢版 |
2019/12/16(月) 18:13:46.48ID:SF0nDj5O
> sapply(1:20,function(k) treasure0(4,5,k))
[,1] [,2] [,3] [,4] [,5] [,6] [,7] [,8] [,9] [,10] [,11]
短軸有利 9 84 463 1776 5076 11249 19797 28057 32243 30095 22749
長軸有利 9 83 453 1753 5075 11353 20057 28400 32528 30250 22803
同等 2 23 224 1316 5353 16158 37666 69513 103189 124411 122408
[,12] [,13] [,14] [,15] [,16] [,17] [,18] [,19] [,20]
短軸有利 13820 6656 2486 695 137 17 1 0 0
長軸有利 13831 6657 2486 695 137 17 1 0 0
同等 98319 64207 33788 14114 4571 1106 188 20 1

4×5の場合
宝:1個 同等
宝:2〜5個 短軸有利
宝:6〜13個 長軸有利
宝:14〜20個 同等

□■■■■
□□■■■
□□□■■
□□□□■

短軸有利☆

Table[sum[C(2n-1+C(0,(21mod n)-1),k-1),{n,1,9}],{k,1,20}]

長軸有利☆

Table[sum[C(2n-1+C(0,6mod n)-C(0,C(3,n-2)-1),k-1),{n,1,9}],{k,1,20}]

同等☆

Table[C(19,k-1)+C(17,k-2)+C(15,k-2)+C(13,k-2)+C(8,k-2)+C(1,k),{k,1,20}]
0769132人目の素数さん
垢版 |
2019/12/16(月) 18:42:55.78ID:QUlMC4qk
0≤a<b≤1とする。
区間[a,b]に含まれる有理数のうち、1/n(nは自然数)の形で表されるものの個数をN(a,b)とする。
N(a,b)*a^pが0でない定数に収束するような有理数pを求めよ。
0770132人目の素数さん
垢版 |
2019/12/16(月) 18:53:31.50ID:QUlMC4qk
連続する二項係数の和(n,k)+(n,k+1)で以下の整数を表すとき、それぞれnの最小値を求めよ。

(1)2019
(2)2020

【注】
例えば6=(5,0)+(5,1)である。
しかし6はよりnの小さい形6=(3,1)+(3,2)で表され、これより小さいnでは表されないから、従って求めるnは3である。
0772132人目の素数さん
垢版 |
2019/12/16(月) 23:17:22.23ID:0NEM/kvz
https://i.imgur.com/2Zk3jKC.jpg

三角形の面積が底辺×高さ÷2になるのって、AやBの場合ってEやFの半分になるからというのはわかるんだけど、Cの場合って何でそうなるんだったっけ?

移動とかできる?
0774132人目の素数さん
垢版 |
2019/12/17(火) 00:13:54.11ID:gjJSfhwE
────────
   ── 

下を三角形の底辺として
上の直線上のどこに頂点を取っても面積は一定

だからCは頂点を平行移動して直角三角形の面積に帰着できる
0776132人目の素数さん
垢版 |
2019/12/17(火) 00:39:09.40ID:gjJSfhwE
>>775
できる
まず先に三角形を片側に足して直角を作ってから、
同じ形の三角形を反対側から切り取ればいい
手順が逆なだけ
0777132人目の素数さん
垢版 |
2019/12/17(火) 00:46:49.30ID:gjJSfhwE
あるいは平行四辺形を対角線できると同じ形の三角形2つになる
それを>>774で直角三角形2つにしてくっつければ長方形に変わる
0778132人目の素数さん
垢版 |
2019/12/17(火) 09:42:03.02ID:/04vhOiY
>>725
>>766
 s = 1/a + 1/b + 1/c,
 S = 1/aa + 1/bb + 1/cc,
とおくと
 (x, y, z) = (k(S-s/a), k(S-s/b), c+k(S-s/c))
 k = ±1/√{S(3S-ss)},
 符号は x>0, y>0 となるようにとる。
 最大値 ks - (ss-2S)(kc + 1/2S).
0780132人目の素数さん
垢版 |
2019/12/17(火) 11:49:50.34ID:/04vhOiY
>>766 訂正
a=b の場合
 最大値 2ac/√(2aa+4cc) - a(4c-a)/(2aa+4cc),

a=b=c の場合
 (x, y, z) = (1/√6, 1/√6, c-2/√6)
 最大値 2c/√6 - 1/2.
0783132人目の素数さん
垢版 |
2019/12/17(火) 15:07:15.26ID:/04vhOiY
>>734
欠損箇所を周辺のデータで補間せよ、ってこと?
 x(3) ≒ {Σ[i=1,5] w(i)x(i)}/{Σ[j=1,5] w(j)}
と仮定すると
 x(3) ≒ {Σ[i≠3] w(i)x(i)}/{Σ[j≠3] w(j)}
  = (0.5*2 + 2*7 + 2*5 + 0.5*3)/(0.5+2+2+0.5)
  = 26.5/5
  = 5.3

*欠損データそのものは分からない。
0787132人目の素数さん
垢版 |
2019/12/17(火) 15:24:57.04ID:LABsjLoU
なにがわからないのかまるでさっぱりわからない、ということは良く分かりました
0788132人目の素数さん
垢版 |
2019/12/17(火) 15:35:57.27ID:WjI+OJsd
https://i.imgur.com/1C4rt1F.jpg
この真ん中らへんに書いてある。
何を言いたいのかさっぱりわからないからとりあえず何か形あるもので見たくて…
もしわかる人居たらカッコのnと外のnを別に別けて指数でどうなってるのかご教授ください。
0790132人目の素数さん
垢版 |
2019/12/17(火) 15:45:59.87ID:WjI+OJsd
なるほ。
でも全然何をしようとしたらこうなるのか全然イメージできなくてワロタ
0792132人目の素数さん
垢版 |
2019/12/17(火) 18:14:17.94ID:mhe8tnec
aを0<aなる実定数とするとき、
定積分
I = ∫[0,a] [1/{x(x+1)} - exp(-x)/x] dx
について以下の問に答えよ。

(1)Iはa→+∞で有限の値に収束することを示せ 。

(2)Iをaの関数と見てI=f(a)とおくとき、f(a)の増減を調べよ。
0794132人目の素数さん
垢版 |
2019/12/17(火) 22:09:42.20ID:gjJSfhwE
>>793
無理しないでちゃんと学校の数学から進めたほうがいいぞ!
背伸びして全く理解できないレベルで難しい本をやってみても、別に自慢できないし
数年後恥ずかしさでのたうちまわるぞ
0796132人目の素数さん
垢版 |
2019/12/17(火) 23:09:06.36ID:MbtYd6vw
リーマン積分で値が求められるがルベーグ積分では求値困難な関数を求めよ
0797132人目の素数さん
垢版 |
2019/12/17(火) 23:32:38.23ID:WjI+OJsd
>>794
loglog(n)=log(logn)=loge^k=kなので
(logn)^2・k=(logn)(logn)k=k・e^2k
それで何をしようとしてるのです?
グラフか何か図示できます?
数学科じゃないのでわからないから禿げそう
0798132人目の素数さん
垢版 |
2019/12/18(水) 00:26:08.50ID:v511nzTO
n以下の素数のうち、一の位の数がkのものの個数N(k)とおく。

極限
lim[n→∞] N(9)/N(3)
を求めよ。
0801132人目の素数さん
垢版 |
2019/12/18(水) 16:20:47.14ID:Qc1Gu641
f_[1](x)=x^3-x
f_[k+1](x)=f(f_[k](x))
とする。
方程式f_[n](x)=0の解で、区間[-1,1]に含まれるものの個数を求めよ。
0802132人目の素数さん
垢版 |
2019/12/18(水) 17:00:06.78ID:1hlpZcRY
確率分布の賞金の平均を求める問題の解答に、単位である円が付いていなかったのですが、
付けないものなんですか?付けてはいけないんですか?
0803132人目の素数さん
垢版 |
2019/12/18(水) 20:02:57.97ID:GQllmBub
>>802
確率の歴史からは、円とは限らんからね。


フランスの数学者パスカル(1623〜1662)が1654年にフェルマーにあてた手紙が、現在の確率

論の始まりだと言われている。当時の有名な賭博師メレがパスカルに以下のような問題を持ち

込み、その問題についてがその手紙のやりとりの中で論じられているそうである。

甲乙二人がおのおの32ピストル(当時のお金の単位)の金を賭けて勝負したとする。

 そしてどちらかが先に3点を得たものを勝ちとし、勝った方がかけ金の総額64ピストルをもら

 えるとする。ところが甲が2点、乙が1点を得たとき、勝負が中止になってしまった。

 このとき、二人のかけ金の総額64ピストルを甲と乙にどのように分配すればよいだろうか。

 ただし二人の力は互角で、勝つ確率はそれぞれ1/2ずつだとする。
0804132人目の素数さん
垢版 |
2019/12/18(水) 23:19:42.39ID:RT2CFTx+
>>802
0805132人目の素数さん
垢版 |
2019/12/18(水) 23:24:52.99ID:RT2CFTx+
>>803

甲が1点1/2で甲勝ち
乙が1点1/2のときは
その次
甲が1点1/2で甲勝ち
乙が1点1/2で乙勝ち
甲勝ちは1/2+1/2・1/2=3/4
乙勝ちは1/2・1/2=1/4
64ピストルのうち
甲へは48ピストル
乙へは16ピストルを配分するのが
正当
0806132人目の素数さん
垢版 |
2019/12/19(木) 07:04:38.27ID:kVFzAP55
日本シリーズは先に4勝した方が覇者となる。
両チームの実力が互角であったときに先勝したチームが覇者となる確率はいくらか?

残りの6試合での勝ち負けを虱潰しに数え上げて42/64= 0.65625になったけど、あってる?

一般化するとこういう問題になる。

先にw点を得たものが勝ち、甲がA点、乙がB点を得たとき、勝負が中止となった
このまま続けていたとして甲の勝った確率は?
0807132人目の素数さん
垢版 |
2019/12/19(木) 07:14:57.41ID:kVFzAP55
>>805
わざわざのレスありがとうございます。
100万回シミュレーションしてみました

sim <- function(A=2,B=1,w=3){ # A wins the game, T/F ?
while(A < w & B < w){
g = rbinom(1,1,p=0.5)
if(g==1){
A=A+1
}else{
B=B+1
}
}
A > B
}
mean(replicate(1e6,sim()))*64

> mean(replicate(1e6,sim()))*64
[1] 47.98048
0808132人目の素数さん
垢版 |
2019/12/19(木) 07:18:45.93ID:kVFzAP55
>>805
俺が乙だったら、点数は2:1だからその比で配分しようよ、と主張して64/3= 21.33333ピストルくれというww
0810132人目の素数さん
垢版 |
2019/12/19(木) 08:25:04.93ID:0xpDbvgy
>>803
昔の人はそんなこともわからないバカしかいなかったんだな。
(もちろん発展は先人のおかげとはいえ)
科学以降人類はどれだけ賢くなったのか検討もつかないな
0811132人目の素数さん
垢版 |
2019/12/19(木) 08:43:01.58ID:3dNIVTIi
nを自然数の定数とする。
実数a,bを10進法表記したときの、それぞれの小数点以下n桁目の数をa[n],b[n]とする。
以下の場合について、b[n]をa[n]で表せ。

(1)a=20/91, b=1/13
(2)a=1/(√2+1), b=√2+1
0813132人目の素数さん
垢版 |
2019/12/19(木) 09:52:27.17ID:Z0fx1z4F
>>809
0815132人目の素数さん
垢版 |
2019/12/19(木) 12:43:17.24ID:Z0fx1z4F
>>814
0816132人目の素数さん
垢版 |
2019/12/19(木) 12:53:54.58ID:kVFzAP55
>>812
速攻のレスありがとうございます。
シミュレーション解・虱潰し解しか出せなかったので助かります。
0817132人目の素数さん
垢版 |
2019/12/19(木) 13:31:38.39ID:kVFzAP55
>>808
俺が乙なら、勝敗は決着してないから、32ピストルくれというぞ。

格言:理屈と膏薬はどんなところにもつく。
0818132人目の素数さん
垢版 |
2019/12/19(木) 16:15:56.48ID:kVFzAP55
>>812
最後はAが得点して勝負が決定するのを忘れずにAが勝者になるまでのBの点数で場合分けすれば良かったのですね。


NS <- function(w,A,B,p){ # 先にw点得点した方が勝者、A,B:現在の点数 ,p:甲の勝率
ans=0
for(k in 0:(w-B-1)){ # k: Aが勝者になるまでのBの点数
ans=ans+choose(w-A-1+k,w-A-1)*(1-p)^k*p^(w-A)
}
return(ans)
}

> NS(3,2,1,0.5)
[1] 0.75
> NS(4,1,0,0.5)
[1] 0.65625
0819132人目の素数さん
垢版 |
2019/12/19(木) 17:38:39.48ID:3dNIVTIi
xy平面上の3つの閉曲線
C : x^2+y^2=1
C_1: x^2+{x^2/(1+x^2)}+y^2=1
C_2: {x^4/(1+x^2)}+y^2=1
を、周長が短い順に並べよ。
0820132人目の素数さん
垢版 |
2019/12/19(木) 18:18:12.06ID:3dNIVTIi
n≥3とする。
ラーメン屋Aでn人の行列ができるとき、列の中に「(先頭側から)男、女、女」の順で並ぶ部分が少なくとも1つ存在する確率がn/(n+10)であるという。
この情報のみを用いて、Aの来店客の男女比を推定せよ。
0821132人目の素数さん
垢版 |
2019/12/19(木) 18:20:24.23ID:3dNIVTIi
6辺の長さが全て整数で、
4面の面積が全て整数で、
体積が整数である

このような四面体の例を1つあげよ。
0824132人目の素数さん
垢版 |
2019/12/19(木) 22:28:29.45ID:4lnbLN0r
>>813>>814
P144 節末問題 4.
10円,100円,500円の3枚の硬貨を同時に投げるとき,表が出る硬貨の金額の合計を賞金とする。
賞金の平均を求めよ。
P168 解答 4.
305
0825132人目の素数さん
垢版 |
2019/12/19(木) 22:37:59.76ID:Z0fx1z4F
>>824
>>802
0826132人目の素数さん
垢版 |
2019/12/19(木) 23:47:23.93ID:557jgt78
>>823
天才ですか?
0827132人目の素数さん
垢版 |
2019/12/19(木) 23:50:56.03ID:557jgt78
今見て気がつきました。
じゃ、正四面体なんかじゃどうでしょうか?
0828132人目の素数さん
垢版 |
2019/12/20(金) 00:13:25.91ID:8dg6uySp
>>827
一辺の長さaの正四面体の体積は(√2/12)*a^3
辺と体積が同時に整数になることはない
0830132人目の素数さん
垢版 |
2019/12/20(金) 00:31:15.36ID:pVRKr0X7
じゃあ、もういっそネタばらししちゃって下さいよ
0832132人目の素数さん
垢版 |
2019/12/20(金) 12:08:33.45ID:kvc0Ke/4
10^n-1のSnってどう求めるの?
0833132人目の素数さん
垢版 |
2019/12/20(金) 15:04:09.45ID:ipZ1Vjdr
二変数対称多項式f(x,y)でf(f(x,y),z)が三変数対称多項式になるようなものを
すべて求めよ。
0834132人目の素数さん
垢版 |
2019/12/20(金) 15:13:46.65ID:ipZ1Vjdr
>>833 二次の対称式の場合は分かるのですが三次以上が手が出ません。。
f(x,y)=axy+b(x+y)+c (ただしb^2-b=ac)
0836132人目の素数さん
垢版 |
2019/12/20(金) 23:10:47.95ID:4Ek/LvdL
無限ホテルのパラドックス読んでてわからないことがあって、新しい宿泊客のために既存の客が部屋を一つづつずらすってあるけど、あれは何でそうなるの?
ネットで調べたけどそれらしい答えが無くて困ってる
無限ホテルが集合論のお話で、ホテルは可算無限集合、無限に居る宿泊客全員も可算無限集合で、どっちも無限としての大きさが合うから部屋は過不足なく用意されるって話だってところまではネットで読んだ
で、Wikipediaには順序数? の計算ルールが書いてあって、1+ωとω+1は違うってあったからこれが部屋移動の理由かと最初は思った
でも無限ホテルって無限人の来客があってもokってあるから、これってω+ωでどこに客をぶちこんでも意味変わらないなと
だからこの予想は違うと今は思ってる
この疑問のしっくり来る(理解できる)解説が見つからなくてずっとモヤモヤしてるので、誰か教えてくれるとありがたいです
0837132人目の素数さん
垢版 |
2019/12/20(金) 23:28:22.52ID:noPcPYwC
新しい宿泊客が1人の場合はn号室の人を(n+1)号室に映せば1号室が空く
新しい宿泊客が可算無限人の場合は、
n号室の人を2n号室に写せば奇数号室が空くのでやはり収容可能
非可算無限人の場合は知らん
0839132人目の素数さん
垢版 |
2019/12/21(土) 02:09:56.13ID:jLO75EBV
>>792
(1)
f(a) = ∫[0,a] {1-exp(-x)}/x dx - ∫[0,a] 1/(x+1) dx
 = log(a) - Ei(-a) + γ - log(a+1)
 = log(a/(a+1)) - Ei(-a) + γ
 → γ = 0.5772156649 (a→∞)
ここに Ei(-a) = ∫[a,∞] (1/x)exp(-x) dx → 0 (a→∞)

(2)
 exp(x) > x+1 より
 1/(x+1) > exp(-x),
 f '(a) > 0,
 f(a) は x>0 で単調増加。
0841132人目の素数さん
垢版 |
2019/12/21(土) 02:37:08.92ID:jLO75EBV
>>819

C : xx+yy = 1,
C_1 : xx+yy = 1 - xx/(1+xx) < 1, (円内)
C_2 : xx+yy = 1 + xx/(1+xx) > 1, (円外)

 C_1 < C < C_2
0843132人目の素数さん
垢版 |
2019/12/21(土) 03:06:48.33ID:jLO75EBV
>>840
溶かしたら(キュリー温度以上では)もう強磁性体ではない。
常磁性だから自発磁化は無いが、周囲の磁場に応じて弱く磁化している。
それが冷えて固まれば(キュリー温度以下で)再び強磁性体に戻る。
強磁性体はヒステリシスが強いため、固化した時の磁化が良く保たれる。
たとえば火山の溶岩の磁化の向きから、噴火当時の子午線に対する方位が分かる。
これを元にして、過去に大陸が移動したことが分かった。(ウェゲナー)
0844132人目の素数さん
垢版 |
2019/12/21(土) 04:15:36.85ID:nUb7ormp
>>843
結局溶かして丸く固めた瞬間はただの鉄で、冷えていけば磁石になるってことですか?
どこがS極でどこがN極になるんですか?
0845132人目の素数さん
垢版 |
2019/12/21(土) 05:52:50.74ID:wum1jR1j
高校の数学の質問です。
よろしくお願いいたします。

10進数で表された5/9を3進数になおせという問題があって、答は0.12だったのですが、分子と分母をそれぞれ3進数になおして、12/100でもあってますか?
自分は、あっていると思うのですが。

で、その理屈でいうと、例えば10進数で表された3/5は、8進数でも3/5になるのですが、あってますか?

友達が、それは違うのではないかというのですが、自分は正しいと思っています。詳しい方に教えていただけたらと思います。
自分の考えは、どこかおかしいのでしょうか?
0846132人目の素数さん
垢版 |
2019/12/21(土) 10:43:48.22ID:OUD6tpgc
>>845
正しい
10進法で5/9というのは5を9=3^2で割ること
それは、3進法では5(10)=12(3)の小数点を左に2つずらすことに対応する

あと、n進法とn進数は似て非なるもの
0847132人目の素数さん
垢版 |
2019/12/21(土) 11:00:20.65ID:hqOu2kUy
>>844
地球だって球体の磁石だし(永久磁石じゃないけど)、形は関係ないんじゃね?
0848132人目の素数さん
垢版 |
2019/12/21(土) 11:33:52.88ID:ya2KQLM2
>>845
合っている。
ただ先生が言っているのは5/9(10進法)を3進法の小数で書けということだと思う

それだと10進法で書いて
5/9
=5 * 3^-2
=(3+2)*3^-2
=1*3^-1 + 2*3^-2

つまり3進法で書いて0.12(3)となる

もちろん3進法で12/100を計算しても同じ結果になる
0849132人目の素数さん
垢版 |
2019/12/21(土) 11:36:55.07ID:ya2KQLM2
n進法の割り算を筆算でやるのは基本的に大変なのでおすすめしない

(2,3進法くらいまでならその場でできそうだけど)

ただabcd…xyz(n)を100…00(n)で割る時は簡単で、10進法のときと同じく小数点をずらすだけで良い
0851132人目の素数さん
垢版 |
2019/12/21(土) 12:41:10.90ID:ejpcme5D
n進法の質問をした者です。
回答を、ありがとうございます。

よくわかりました。
質問をしてよかったです。
0852132人目の素数さん
垢版 |
2019/12/21(土) 13:47:07.77ID:wmcVYWUB
以下の3条件を全て満たす四面体が存在することを示せ。

・6辺の長さが全て整数
・4つの面の面積が全て整数
・体積が整数
0853132人目の素数さん
垢版 |
2019/12/21(土) 15:00:45.93ID:yZH+pOFG
4次元ミンコフスキー空間内の曲線f:[a,b]→Wは単射かつ連続微分可能、逆も連続微分可能であるとする
(e[0],e[1],e[2],e[3])をローレンツ基底として
f(t)=Σf^i(t)e[i]
と展開する

この状況で、とある本に「fの単射性からf^0も単射」とあるのですが、これは成り立たないような気がします(f(t)=te[1]が反例)
もしfが時間的曲線(つまり<df/dt,df/dt>が常に正)であると仮定したときは成り立ちますか?もし成り立つなら証明を教えて頂きたいです
0854132人目の素数さん
垢版 |
2019/12/21(土) 16:14:18.70ID:jLO75EBV
>>844
キュリー温度になったときの磁場 (地球磁場) の方向に磁化する。
 (純鉄で 770℃、フェライトで 500℃ ぐらい)
ただし、磁化は非常に弱く、「磁石」とは呼べない。
再びパルス磁場を加えて強く磁化させれば磁石になる。

>>843
 ウェゲナーより50年も前に大陸移動を主張した人もいたが、
証拠不十分で保留になった。
0855132人目の素数さん
垢版 |
2019/12/21(土) 16:25:46.24ID:jLO75EBV
>>801
x_0 = 1,
f_[k](x_k) = 1
とおくと
f_[1](x_k) = (x_k)^3 - (x_k) = x_(k-1),
これより
x_0 = 1,
x_1 = {((27-3√69)/2)^(1/3) + ((27+3√69)/2)^(1/3)}/3
  = 1.32471795724475
x_2 = 1.39603524561538
x_3 = 1.41056662315775
x_4 = 1.41348372262564
x_5 = 1.41406757634046
x_6 = 1.41418436444319
x_7 = 1.41420772275818
x_8 = 1.41421239444895
x_9 = 1.41421332878822
x_10= 1.41421351565612
x_k → √2  (k→∞)
0857132人目の素数さん
垢版 |
2019/12/21(土) 22:25:28.32ID:vMc6M+31
>>824
基本的に、最後に単位付けるの忘れるなよでやってきて、
今回は単位付けんなよでは困ります。
0859132人目の素数さん
垢版 |
2019/12/22(日) 02:22:23.16ID:fKVXFhjD
行列環 M n(R) は右自由 R 加群の自己準同型環と同一視できるのはなぜ
0861132人目の素数さん
垢版 |
2019/12/22(日) 04:23:00.95ID:fKVXFhjD
すいませんどうゆう同型写像を作ればいいにでしょうか
0862132人目の素数さん
垢版 |
2019/12/22(日) 04:55:25.21ID:fKVXFhjD
すいませんわかりました
0863132人目の素数さん
垢版 |
2019/12/22(日) 12:33:14.60ID:l9w4vljZ
次の関数F(s)を求めよ。
F(s) = ∫[0,∞] e^(-st)√(1-t^2) dt
注)t>1における積分路は実軸上とせよ。
0865132人目の素数さん
垢版 |
2019/12/22(日) 12:50:12.82ID:hxEyFLd0
>>853
補足ですが、<x,y>はミンコフスキー計量で、(e[i])がローレンツ基底とは正規直交基底で
<e[0],e[0]>=1, <e[j],e[j]>=-1(j≠0)
を満たすもののことです

もし付加条件を仮定しても成り立たなければ、その反例を教えてください
0872132人目の素数さん
垢版 |
2019/12/23(月) 00:39:52.95ID:fjpmrYbL
これはBessel関数なんか使わんでもいけそう・・・・
 r = 1/(1+d)   (d>0)
 r^n = 1/(1+d)^n = 1/{1 +nd + n(n-1)dd/2 + ・・・・} ≦ 1/[nd + n(n-1)dd/2],
 n・r^n ≦ 1/[d + (n-1)dd/2] → 0  (n→∞)
0873132人目の素数さん
垢版 |
2019/12/23(月) 03:26:46.56ID:fjpmrYbL
これはBessel関数なんか使わんでもいけそう・・・・
a_n = n・r^n とおく。

n > 1/(1-r) ならば
a_(n+1) / a_n = (n+1)r/n < (2-r)r = R,
ここに R = 1 - (1-r)^2 < 1,

N = [1/(1-r)] + 1 として
a_n / a_N < R^(n-N) → 0 (n→∞)
 (比較判定法)
0874132人目の素数さん
垢版 |
2019/12/23(月) 18:07:47.83ID:hSNh7p4P
次の極限が収束することを示し、その値を求めよ。nは自然数である。

lim[n→∞] ∫[0,n] e^(-x)*{Σ[k=1,n] (x^k)/k!} dx
0875132人目の素数さん
垢版 |
2019/12/23(月) 19:36:36.70ID:fjpmrYbL
I_n = ∫[0,n] e^(-x)*{Σ[k=1,n] (x^k)/k!} dx
とおく。
nが小さいときは
I_1 = 1 - 2 e^(-1),
I_2 = 2 - 8 e^(-2),
I_3 = 3 - (51/2)e^(-3),
I_4 = 4 - 76 e^(-4),
I_5 = 5 - (1765/8)e^(-5),
I_6 = 6 - (3162/5)e^(-6),
I_7 = 7 - (431851/240)e^(-7),
I_8 = 8 - (178296/35)e^(-8),
I_9 = 9 - (64366227/4480)e^(-9),
I_10 = 10 - (7635340/189)e^(-10),
0878132人目の素数さん
垢版 |
2019/12/23(月) 20:29:45.55ID:Iu2lFBcp
球の体積についでですが
円錐の集まりで体積を求める方法と三角錐で求める方法がありましたが同じように感じました
これはどれでも同じなんでしょうか?
例えば星型のように一部が出っ張っていたり
または底面πR^2の中にその半分の半径の空洞があったりしても同じなんでしょうか?

多分同じだとは思うのですがそこを理解するためには何を学べばいいでしょうか?
0879132人目の素数さん
垢版 |
2019/12/23(月) 20:36:54.08ID:/K57AvEV
>>803
俺が甲だったら

>二人の力は互角で、勝つ確率はそれぞれ1/2ずつだとする。

これはおかしい
二人の力は全く不明なので通算成績を得点する確率として推測するべきだと主張して


現時点で甲が得点する確率は2/3
甲が勝つのは
(1)次に得点する
(2)次は乙が得点してその次に甲が得点する
場合である。
(1)の確率はそれまでの通算成績から2/3
(2)の確率は(1-2/3)*(2/4)
よって甲が勝つ確率は2/3+(1-2/3)*(2/4)
> 2/3+(1-2/3)*(2/4)
[1] 0.8333333
>
それゆえ
> (2/3+(1-2/3)*(2/4))*64
[1] 53.33333
をよこせ、主張する。
0881132人目の素数さん
垢版 |
2019/12/23(月) 22:31:09.38ID:fjpmrYbL
>>875
I_1 = 1 - 0.735758882342885
I_2 = 2 - 1.0826822658929
I_4 = 4 - 1.3919885555438
I_8 = 8 - 1.7089041344145
I_16 = 16 - 2.15344281643
I_32 = 32 - 2.79771778864
I_64 = 64 - 3.7206
I_128 = 128 - 5.034
I_256 = 256 - 6.898
I_512 = 512 - 9.537
I_1024 = 1024 - 13.27
I_2048 = 2048 - 18.56

I_n ≒ n - 0.40√n  (n>>1)
どう見ても収束しない。
0882132人目の素数さん
垢版 |
2019/12/23(月) 22:50:05.00ID:CsxCFjif
xについての2次方程式
x^2-bx+p=0…(*)
を考える。

(1)pを実数の定数とする。
bが実数全体を動くとき、(*)の解が取りうる値の範囲を複素数平面上に図示せよ。

(2)bを実数の定数とする。
pが実数全体を動くとき、(*)の解が取りうる値の範囲を複素数平面上に図示せよ。

(3)kを実数の定数とする。
bが実数全体を動き、かつp=kbであるとき、(*)の解が取りうる値の範囲を複素数平面上に図示せよ。
0884132人目の素数さん
垢版 |
2019/12/24(火) 01:15:59.69ID:Vbfn7Z0E
>>877
→に関する推論規則が無い論理体系
0885132人目の素数さん
垢版 |
2019/12/24(火) 01:16:53.29ID:Vbfn7Z0E
>>879
勝手に言ってろ
0886132人目の素数さん
垢版 |
2019/12/24(火) 04:03:13.29ID:HK5MCHSc
>>871
相乗-相加平均で
 (n+1) r^n < r^(n/2) (1+r+r^2+・・・・+r^n) < r^(n/2) /(1-r) → 0  (n→∞)
0887132人目の素数さん
垢版 |
2019/12/24(火) 18:37:27.73ID:UKNiwghI
>>885
理屈と膏薬はどんなところにもつく。

実力は不明だから、対戦前の甲が得点する確率を一様分布と仮定して計算すると、
2点対1点での甲の得点する確率はβ(3,2)のβ分布になる。
期待値を用いて計算すると
(3/5 + (1-3/5)*3/6)*64 = 51.2ピストルを甲が要求できることになる。
0888132人目の素数さん
垢版 |
2019/12/24(火) 23:43:31.29ID:QTPsAdUJ
確率変数Xの確率密度関数がf(x)=a-{(1/2)x}(0≦x≦2)で表されるとき,次の問いに答えよ。

(1)定数aの値,および確率P(1<X≦2)を求めよ。

先生、なぜa≧1ではなくa=1となるのでしょうか?限定できるのでしょうか?
何が盲点突かれているのかわかりません。お教え願えませんでしょうか?
0890132人目の素数さん
垢版 |
2019/12/25(水) 02:55:16.21ID:oEKznZ6+
>>888
定義域での確率の総和(確率密度関数の面積)が1になるから
f(x)の不定積分をF(x)とすれば
F(x)=ax-x^2/4
F(2)-F(0)=1
a=1
0891132人目の素数さん
垢版 |
2019/12/25(水) 03:42:57.86ID:eSEVzzoW
>>842

周長 = 2∫[-1,1] √{1 + (dx/dy)^2} dy
だから |dx/dy| を比べよう。

C: 2xdx + 2ydy = 0 より
  dx/dy = -(y/x),

C_1: 2x{1 + 1/(1+xx)^2} + 2ydy = 0 より
  dx/dy = -(y/x)/{1 + 1/(1+xx)^2},

C_2: 2x{1 - 1/(1+xx)^2} + 2ydy = 0 より
  dx/dy = -(y/x)/{1 - 1/(1+xx)^2},

∴ |dx/dy| を比べると
  C_1 < C < C_2
0892132人目の素数さん
垢版 |
2019/12/25(水) 04:58:32.23ID:eSEVzzoW
まちがえた。

L = 2∫[-1,1] √{1+(dx/dy)^2} dy
だから |dx/dy| を比べよう。

C: 2xdx + 2ydy = 0 より
  |dx/dy| = |y|/√(1-yy),

C_1: 2x{1 + 1/(1+xx)^2} + 2ydy = 0 より
  dx/dy = -y/{x + x/(1+xx)^2},
 ここで xx ≦ 1/φ < 1 より
  |x|{1 + 1/(1+xx)^2} > |x|{1 + 1/[2(1+xx)]}
  ≧ |x|√{1 + 1/(1+xx)}
  = √(1-yy),
  |dx/dy| > |y|/√(1-yy),

∴ |dx/dy| を比べると
  C_1 < C
0893132人目の素数さん
垢版 |
2019/12/25(水) 05:06:41.06ID:eSEVzzoW
>>842
 C_2 の内部の面積 S_2 > π,
∴ 等周問題から、
 C_2 の周長 ≧ √(4πS_2) > 2π = Cの周長。
0894132人目の素数さん
垢版 |
2019/12/25(水) 11:24:30.57ID:1bVwZBvf
>>889>>890
a≦スモールx≦b
a≦ラージX≦b
α≦ラージX≦β
の違いまったくわからないのですが、とりあえずお疲れのところありがとうございました。
0895132人目の素数さん
垢版 |
2019/12/25(水) 12:15:56.57ID:7aUY9Wdu

i , j = 1 , 2のときクロネッカーのデルタδijを求めよ

質問
「i , j = 1 , 2」のとき の読み方としては
「i , jが1もしくは2のとき」でよいのでしょうか?
それとも「i = 1 かつ j = 2のとき」でしょうか?
以上、よろしくお願い致します
0898132人目の素数さん
垢版 |
2019/12/25(水) 19:31:25.97ID:4qjKLfBB
小林の曲線と曲面の微分幾何のp48で、(u,v)平面に曲線u=u(t), v=v(t)をとり...とあるのですが、この定義がよくわからないので教えてください。
この(u,v)平面のある開集合はR^3内の曲面のパラメーターとして使われています。
0899132人目の素数さん
垢版 |
2019/12/25(水) 20:21:51.19ID:oEKznZ6+
>>894
確率変数Xが0〜2の値ととりその確率密度関数がf(x)なら
Xが1〜2になる確率は∫[1,2]f(x)dx という話ではないの?
0901132人目の素数さん
垢版 |
2019/12/25(水) 20:36:38.96ID:7nMOghr0
aを実数の定数,P(x)をxに関する命題として、∀r∈(0, a) ∀x∈[0, r] P(x) ⇒ ∀x∈[0, a) P(x)は一般に正しい?
もし正しいとすると次のことが言えてしまって、困惑しています。
[0, a)上の連続関数列f_n(x)とその各点収束先の[0, a)上の連続関数f(x)で、lim[x→a-0]f(x)=∞なるものを考える。これは一様収束ではないが、r∈(0, a)を任意に取れば[0, r]上では一様収束する⇒[0, a)上でこれは一様収束しているといえてしまい、矛盾する。
0902132人目の素数さん
垢版 |
2019/12/25(水) 20:44:03.35ID:Xl2uuUVl
>>901

> aを実数の定数,P(x)をxに関する命題として、∀r∈(0, a) ∀x∈[0, r] P(x) ⇒ ∀x∈[0, a) P(x)は一般に正しい?

正しい。

> もし正しいとすると次のことが言えてしまって、困惑しています。

ダウト。
↑を用いて↓を示すために何をP(x)とすればできるのか、直感ではなく具体的に書き下してみる。

> [0, a)上の連続関数列f_n(x)とその各点収束先の[0, a)上の連続関数f(x)で、lim[x→a-0]f(x)=∞なるものを考える。これは一様収束ではないが、r∈(0, a)を任意に取れば[0, r]上では一様収束する⇒[0, a)上でこれは一様収束しているといえてしまい、矛盾する。
0903132人目の素数さん
垢版 |
2019/12/25(水) 20:56:32.67ID:7nMOghr0
>>902
P(x)をこう取れると考えました。

∀ε>0 ∃N>0 ∀n≧N ∀x∈[0, r) ┃f_n(x) - f(x)┃<ε

あっ、例の論理式を用いて出てくるのは
∀x∈[0, a) ∀ε>0 ∃N>0 ∀n≧N ┃f_n(x) - f(x)┃<ε、つまり各点収束の定義であって、∀ε>0 ∃N>0 ∀n≧0 ∀x∈[0, a) ┃f_n(x) - f(x)┃<ε、つまり一様収束の定義は出てこないということですか。なるほど〜、ありがとうございます。
0904132人目の素数さん
垢版 |
2019/12/25(水) 23:58:19.23ID:zffetm6f
f(x.y)=ye^x
df=?
0905132人目の素数さん
垢版 |
2019/12/26(木) 00:00:28.64ID:Da50t6Td
>>904
dx=ye^x
dy=e^x
df=ye^xdx+e^xdy
じゃないの?
0906132人目の素数さん
垢版 |
2019/12/26(木) 00:01:10.35ID:Da50t6Td
>>904>>905は自分なんだけど
誰か頼む
0907132人目の素数さん
垢版 |
2019/12/26(木) 00:03:52.63ID:YVgI+UyN
dfは合ってる。
dxとdyは違う。
左辺が微分形式て右辺がスカラーということはあり得ない。
0909132人目の素数さん
垢版 |
2019/12/26(木) 12:21:51.95ID:HEbCqewL
>>907
905ではないが代わって
∂f/∂x=... , ∂f/∂y=...
と書こうとしてうまく出なかったんじゃないの
0911132人目の素数さん
垢版 |
2019/12/26(木) 13:59:23.28ID:gQUazjvf
>>910
ベクトルに対しての数値
0915132人目の素数さん
垢版 |
2019/12/26(木) 17:44:21.69ID:vHJ/AHVv
O(0,0,0),A(1,0,0),B(1,1,0),C(0,1,0)
P(cosθ,sinθ,t-sinθ)
Q(sinθ+1,cosθ,t+sinθ)

tを実数の定数とする。
0≤θ≤π/2のとき、
OP+PA+BQ+QCの最大値を求めよ。
0916132人目の素数さん
垢版 |
2019/12/26(木) 23:18:47.56ID:u+uHVqcC
>>914
?=30
0917132人目の素数さん
垢版 |
2019/12/26(木) 23:45:50.82ID:R3NxxcTg
>>914
tan(?) = {sin(24゚)+sin(36゚-24゚)}/{1-cos(24゚)+cos(36゚-24゚)}
 = {sin(24゚)+sin(12゚)}/{1-cos(24゚)+cos(12゚)}
 = ・・・・
0919132人目の素数さん
垢版 |
2019/12/27(金) 01:28:55.12ID:BC11RaU1
>>916の補足。
補助線を引いて頂角が36°の二等辺三角形を考えてみる(底角72°)。
等しい2辺の長さを1とすると、この二等辺三角形の底辺の長さは正弦定理から、
sin(36°)/sin(72°)=1/2cos(36°)
この底辺を共有するもう一つの三角形の頂角?°に対して、底角は96°と84-?°
になるのが、これに正弦定理を適用すると、
底辺の長さ=sin(?°)/sin(84°-?°)=1/2cos(36°)
これを?について解けばよいのだが、?=30とすれば、この方程式を満たすことは
明らか。
0920132人目の素数さん
垢版 |
2019/12/27(金) 02:38:36.59ID:yVLOQFDS
ABCの三組(一組の上限人数10人)に組分けをする
成績1位〜30位までの30人が、1位の人から順番にルーレットを回してABCに割り振られていく
途中で上限人数に達したら締切で、残りの組のみからなるルーレットを回す

これって、最後の方に回すことになる成績悪い人が同じ組に固まりやすかったりしますか?最後にどこかの組が空いてたらそこになだれ込んじゃうわけですし
0921132人目の素数さん
垢版 |
2019/12/27(金) 08:58:13.03ID:BC11RaU1
>>920
A,B,Cと書いた紙が10枚ずつ入った箱から順番にとっていくの
と同じことでしょ(順番に関係なく特定の組を引く確率は1/3
だよね?)。

抽選でランダムに分けたら、どうやっても偏りが生じうるのは
しょうがないんじゃないの?偏るのがいやなら、成績順にA,B,C.
C,B,A,,,と振り分けるとかしないと。
0922132人目の素数さん
垢版 |
2019/12/27(金) 09:01:12.18ID:FqqlMh9P
>>920
20回シミュレーションしてみた

> sim <- function(n=30){
+ r=numeric(n)
+ for(i in 1:n){
+ j=which(c(sum(r==1)<10 , sum(r==2)<10 , sum(r==3)<10 ))
+ r[i]=sample(j,1)
+ }
+ cat(c('A','B','C')[r],'\n\n')
+ }
> for(i in 1:20) sim()
A B A B A A B B A B C A B A C B A A B A C C C C C C C B C A

A B C C B A B B B C C B B B C B A A B A A A A A C C C A A A

A A B C C A B A A B B C B A B B C A A C C B B B A A B A B B

C C A A B A A A B A A C A B A B B A B C B B B C B A B C B B

A C C A A A C C C A C C B B C C C A B A B B B B A A B B B A

A C B B B B B B B A C A A B A A B A A B A C C A B A C B A A

A C C B A B B A B C A C B B B A A B C B A B A A C C A B B C

A C B C A B C B C C C C A B A C C A C B A B A B B A A A B A

B B C C A B B C B B B C A B B A A A B A A C A A C C A B B A

A A C C A C C C C C A B C B C B C B B A B B B B A B A A A A

A C A A A C A A A C B C B C B C A C B B A B A B B B C C C B

A C C C A B A C B C B C B B A A B A B B B A A A A C B B C C

A B C A C B B A C B A C C A B B C B B A A C B A A C B C C A

B B C C A C B B B B A C C B A C C B C B C A C B A A A A A A

C A A B C B A C A B A A B A A C C A C A C B B B C B C C A B

C A C C A B C A C C B B B B C C A A B C A A B C A B B A B A

B B B A B C A C A B A C B B A C C A C A B B A A C C B A C B

A C C C A B B C C A A B A C A C B B C A A B C B B C A B B A

A C C B A C A B B B C B C C A A C B B C B B A A C C B A A A

B A A C C A A A A A B C B B B A B B A C C C C C B C A C A A
0923132人目の素数さん
垢版 |
2019/12/27(金) 09:22:36.25ID:8Ftk2h9g
>>921
>順番に関係なく特定の組を引く確率は1/3
>だよね?
つまり偏らないということ
0924132人目の素数さん
垢版 |
2019/12/27(金) 09:28:06.07ID:bnpG+BjS
>>921
それとはちょこっと違うんじゃないかな
その場合は、一人目がAを引いたら二人目はAを引く確率が少し下がる
しかし、>>920の設定だと一人目がAを引いても二人目がAを引く確率は同じ
ABCが全て残っていれば内訳がいくつであろうとそれぞれ1/3
こういう設定でも29番目に引く人と30番目に引く人が同じ組になる確率は1番目と2番目が同じ組になる確率と同じだろうかっていう質問なんじゃないか?
0925132人目の素数さん
垢版 |
2019/12/27(金) 09:35:40.72ID:m7wze3DH
ちゃんと計算しないとだけど一番の人と2番の人が同組になる確率と29番目の人と30番目の人が同組になる確率は違う気はする。
0926132人目の素数さん
垢版 |
2019/12/27(金) 09:48:04.03ID:FqqlMh9P
>>922
デバッグして各部屋の成績順と平均を出すように変更

> sim <- function(){
+ n=30
+ r=numeric(n)
+ for(i in 1:n){
+ j=which(c(sum(r==1)<10 , sum(r==2)<10 , sum(r==3)<10 ))
+ r[i]=as.numeric(sample(as.character(j),1))
+ }
+ A=c(which(r==1))
+ B=c(which(r==2))
+ C=c(which(r==3))
+  print(as.matrix(rbind(A,B,C)))
+
+ c(mean_A=mean(A),mean_B=mean(B),mean_C=mean(C))
+
+ }
> sim()
[,1] [,2] [,3] [,4] [,5] [,6] [,7] [,8] [,9] [,10]
A 2 5 6 9 11 16 17 18 19 22
B 3 4 7 8 10 13 14 20 23 25
C 1 12 15 21 24 26 27 28 29 30
mean_A mean_B mean_C
12.5 12.7 21.3
0927132人目の素数さん
垢版 |
2019/12/27(金) 10:45:35.34ID:FqqlMh9P
>>925
その直感を体感するために
29番と30番が同室になる確率を10万回のシミュレーションで出してみた。

sim <- function(n=30,a=29,b=30){
r=numeric(n)
for(i in 1:n){
j=which(c(sum(r==1)<10 , sum(r==2)<10 , sum(r==3)<10 ))
r[i]=as.numeric(sample(as.character(j),1))
}
A=c(which(r==1))
B=c(which(r==2))
C=c(which(r==3))
room=rbind(A=A,B=B,C=C)
ans=FALSE
for(i in 1:3){
if(any(room[i,]==a) & any(room[i,]==b)) {
ans=TRUE
break
}
}
ans
}
mean(replicate(1e5,sim(n=30,a=29,b=30)))

> mean(replicate(1e5,sim(n=30,a=29,b=30)))
[1] 0.74055
0928132人目の素数さん
垢版 |
2019/12/27(金) 11:03:15.88ID:BC11RaU1
>>924
>一人目がAを引いたら二人目はAを引く確率が少し下がる
そうだけど、一人目がどうなろうが、二人目がAを引く確率だけに注目すると、
1/3だよね。P(1stA|2ndA)+P(1stNotA|2ndA)=1/3*9/29+2/3*10/29=1/3

ああ、でも、11人目よりあとになると、連続してAって場合がありえないから
違ってくるのか。

あと、一人目と二人目が同組になる確率は9/29、二人目と三人目めが同組
になる確率も9/29,,,となるけど、途中で札が枯渇するから、あとの方は同じ
確率にならんよな気がするね、たしかに。
0929132人目の素数さん
垢版 |
2019/12/27(金) 11:09:04.01ID:BC11RaU1
>>927
1と2,2と3,,,でどこからどう違ってくるかやってみて欲しい。
0930132人目の素数さん
垢版 |
2019/12/27(金) 11:16:12.58ID:BC11RaU1
>>923
いや、それでも偏るでしょってこと。
1位から10位まで同じ組っていうことも 10!/30^10≒6/10^9
の確率で起きるし、まったく偏らない組分けになる確率は
かなり低いと思う。
0931132人目の素数さん
垢版 |
2019/12/27(金) 11:22:34.06ID:BC11RaU1
>>930
すまん、間違えてた。A,B,Cと書いた紙を10枚ずついれて引かせた場合には、
1位から10位までが同じ組になる確率は3*C(30,10)≒1/10^7でした。
0932132人目の素数さん
垢版 |
2019/12/27(金) 11:29:31.42ID:xhuWrzo5
数を減らして6人を3人ずつ二組に分ける場合を考えると
>920の設定の場合、1位と2位が同組になる確率は1/3で5位と6位が同組になる確率は17/54じゃないかな
意外だが後者の方が確率が低い
計算間違えてるかな
0933132人目の素数さん
垢版 |
2019/12/27(金) 11:40:29.96ID:BC11RaU1
何度もすまん、ルーレット方式だとAに当たる確率は10人目までは1/3だけど、
11人目以降が1/3より小さくなるんだな。くじ引き方式だと最後まで1/3なので、
やっぱり別問題だったわ。
一人目と二人目が同組になる確率についてもルーレット方式だと1/3なので
はなから違う。
0934132人目の素数さん
垢版 |
2019/12/27(金) 12:00:03.25ID:BC11RaU1
>>932
5位と6位が同組になる確率はその組を1位から4位まで誰も選ばない
確率だから、3*(2/3)^4=16/27じゃないの?
0935132人目の素数さん
垢版 |
2019/12/27(金) 12:34:37.63ID:BC11RaU1
>>932
くじ引き方式(>>921)だと、1位と2位が同組になる確率も5位と6位が同組
になる確率も等しく1/5だね。
0936132人目の素数さん
垢版 |
2019/12/27(金) 12:36:37.81ID:FqqlMh9P
>>914
複素平面で考えた方が楽だった。

https://i.imgur.com/YLQ3cJY.jpg

a=1 # length of 0P
b=1 # length of PQ
alpha=24/180*pi # angle of 1-0-P
beta=36/180*pi # angle of 0-P-Q
P=a*(cos(alpha)+1i*sin(alpha))
Q=P+b*(cos(alpha+pi-beta)+1i*sin(alpha+pi-beta))
(pi-Arg(Q-1))/pi*180 # degree of Q-1-0
abs(Q-1) # length of Q1

> (pi-Arg(Q-1))/pi*180 # degree of Q-1-0
[1] 30
> abs(Q-1) # length of Q1
[1] 1.229297
0939132人目の素数さん
垢版 |
2019/12/27(金) 13:29:36.09ID:FqqlMh9P
>>936
点線のつくるもう一つ角は18°となった。長さは実線分の1.23倍
# 長さL,M,NのZ尺を角度A°(LとMのなす角)、B°(LとMのなす角)で折り曲げたとき
# 先端と終端を結ぶ線とZ尺の作る角度および先端と終端の距離
# https://i.imgur.com/YLQ3cJY.jpg
Zoo <- function(L=1,M=1,N=1,A=36,B=24){ # L=QP,M=P0,N=01,A=Q-P-0 B=P-0-1
alpha=B/180*pi
beta=A/180*pi
a=M/N
b=L/N
P=a*(cos(alpha)+1i*sin(alpha))
Q=P+b*(cos(alpha+pi-beta)+1i*sin(alpha+pi-beta))
Langle=(pi-Arg(Q-1))/pi*180 # degree of Q-1-0
Uangle=Arg(Q-P)/pi*180 - Arg(Q-1)/pi*180 # degree of P-Q-1
length=abs(Q-1)*N # length of Q1
c(Langle,Uangle,length)
}
Zoo(1,1,1,36,24)

> Zoo(1,1,1,36,24)
[1] 30.000000 18.000000 1.229297
0942132人目の素数さん
垢版 |
2019/12/27(金) 13:43:52.82ID:FqqlMh9P
>>941
Rだと数値演算でこういうのが起こる
> (1-1+1/10)==1/10
[1] TRUE
> (1+1/10-1)==1/10
[1] FALSE

Wolframでは上記のようなのは起こらない。
0943132人目の素数さん
垢版 |
2019/12/27(金) 13:45:52.09ID:FqqlMh9P
Pythonでも同じ誤差がでる
(1.2-1)*5==1
Out[1]: False

(1.2-1)*5
Out[4]: 0.9999999999999998
0944132人目の素数さん
垢版 |
2019/12/27(金) 13:56:56.80ID:xhuWrzo5
>>934
1位と2位が同じ組になった場合、3位4位は1/2ルーレットを使うことになるからちょっと違ってくる
0945132人目の素数さん
垢版 |
2019/12/27(金) 13:57:31.91ID:m7wze3DH
まぁどうせ計算機何てとりあえず答えの数値出しといて後でじっくり30°になるんだからいいんだけど。
今回からのも30°ってわかってしまえば後はチェックするの簡単だし。
0946132人目の素数さん
垢版 |
2019/12/27(金) 14:09:19.51ID:BC11RaU1
>>945
いちおう正弦定理を使えば方程式を導けて(>>919)、
sin(x)*sin(54°)=sin(30°)*sin(84°-x)
と変形できるので、x=30°が厳密解になることがわかる。
0947132人目の素数さん
垢版 |
2019/12/27(金) 14:28:41.81ID:m7wze3DH
>>946
そう。
一応そこからtan(x)=‥にして整理していけば原理的にはとけるけどとてもやる気がしない。
それに30°くらいならいいけど覚えてない数になった時は右片の最小多項式の次元見て総当たりするしかない。
(計算機なら一瞬でやってくれるけど手計算ではほぼ無理)
でもtan(x)=の形になるから解の一意性は明らかなので勘であたりをつけて成立する事を確認する方が実用的。
計算機はその "あたり" をつけるために使うだけだから数値計算してくれれば、まぁ実用には耐えうる。
0948132人目の素数さん
垢版 |
2019/12/27(金) 14:31:06.82ID:BC11RaU1
>>944
あ、そうですね。
P(AABB)=P(BBAA)=1/36,P(ABAB)=P(ABBA)=P(BAAB)=P(BABA)=1/54
となるので、P(****CC)=2/36+4/54=7/54
よって、P(****AA)+P(*****BB)+P(****CC)=7/18か...
1/3=6/18よりちょっと大きいだけですね。
0949132人目の素数さん
垢版 |
2019/12/27(金) 14:53:15.46ID:BC11RaU1
>>947
まあ、>>946の方程式を眺めても自明だけど、積を和に直して、

1/2{cos(x-54°)-cos(x+54°)}=1/2{cos(x-54°)-cos(114°-x)}
と変形して整理すれば
cos(x+54°)=cos(114°-x)
を解けばいいだけ。したがって、x+54°=114°-x より、x=30°となる。
(0°<x <180°ではこれ以外に解はない)
0950イナ ◆/7jUdUKiSM
垢版 |
2019/12/27(金) 15:24:39.30ID:zPJPlDKd
>>403
>>531平行四辺形ABCDの面積をSとすると、
EC=(1/4)BCより△DEC=(1/2)(1/4)S=S/8
△ECF=(1/3)△DEF=S/24
△DEF=△DEC-△ECF=S/8-S/24=S/12
Dを起点にメネラウスの定理より、
(DG/GE)(EB/BC)(CF/FD)=1
(DG/GE)(3/4)(1/2)=1
DG/GE=8/3
△GEF=(3/11)△DEF=(3/11)(S/12)=S/44
四角形GFCE=△ECF+△GEF
=S/24+S/44=17S/264
∴四角形GFCEは平行四辺形ABCDの17/264倍
0951132人目の素数さん
垢版 |
2019/12/27(金) 16:08:09.73ID:m7wze3DH
>>949
おお、なるほど。
そんな手がありましたか。
あくまで>>947はこの手の初等幾何の問題を大人気なく解く時の一般論ね。
本問なら確かにそれで一撃ですな。
0953132人目の素数さん
垢版 |
2019/12/27(金) 18:44:41.26ID:KwJt/dWM
>>930
偏らないよ
1回目からk回目までをまとめて確率事象としたものと
a+1回目からa+kl回目までをまとめて確率事象としたものとで
同じ配分になる確率は同じ
0954132人目の素数さん
垢版 |
2019/12/27(金) 18:46:12.96ID:KwJt/dWM
>>944
それだと条件付き確率で
当然ながら異なってくる
1〜kとa+1〜a+kとで比較するときに
条件付き確率で考えてはダメ
0957132人目の素数さん
垢版 |
2019/12/27(金) 19:03:22.99ID:KwJt/dWM
>>955
アホ金
A×3+B×3を1列に並べる総数は6C3通り
AA**** 4C1通り
BB**** 4C1通り
****AA 4C1通り
****BB 4C1通り
12人目が同一組になる確率2*4C1/6C3=1/10
56人目が同一組になる確率2*4C1/6C3=1/10
というか計算する必要もないほど自明
0958132人目の素数さん
垢版 |
2019/12/27(金) 19:07:35.22ID:KwJt/dWM
>>957
>2*4C1/6C3=1/10
アホ金
2/5
0959132人目の素数さん
垢版 |
2019/12/27(金) 19:12:43.62ID:KwJt/dWM
君ら
くじ引きが平等だってのを
条件付き確率計算するとか本質的でない理解しかしてないのでない?
どこでも同じなのはどこでも本質的に同じだからだよ
0961132人目の素数さん
垢版 |
2019/12/27(金) 19:45:55.18ID:KwJt/dWM
>>960
だって本質的にどこでも同じやン
0962132人目の素数さん
垢版 |
2019/12/27(金) 20:20:10.28ID:m7wze3DH
6人フタ部屋のルーレット方式でシミュ作ってやってみたらいい。
1,2番目が同部屋は明らかに確率1/2。
5,6番目が同部屋も果たしてそうか?
0963132人目の素数さん
垢版 |
2019/12/27(金) 20:29:23.94ID:KwJt/dWM
>>962
なんでルーレット?
条件付き確率で考えたとしても
12が同部屋になるのは
1が何を引いたとしても2がその残りから同じ部屋番を引くとき
1が引いた時点でその部屋番は1つ減っているから
2がそれを引く確率は2/(2+3)=2/5
0964132人目の素数さん
垢版 |
2019/12/27(金) 20:32:12.46ID:FqqlMh9P
>>927
1〜29番が30番と同室になる確率を各々1万回のシミュレーションで求めてみた。
シミュレーション回数不足かもしれないが、一定の傾向は認められる。

https://i.imgur.com/QfZf9M1.jpg

> rbind(aa,p30)
[,1] [,2] [,3] [,4] [,5] [,6] [,7] [,8] [,9] [,10] [,11] [,12] [,13] [,14]
aa 1.0000 2.0000 3.0000 4.0000 5.0000 6.0000 7.000 8.0000 9.0000 10.0000 11.0000 12.0000 13.0000 14.0000
p30 0.2421 0.2559 0.2457 0.2539 0.2434 0.2477 0.248 0.2529 0.2469 0.2537 0.2479 0.2441 0.2559 0.2497
[,15] [,16] [,17] [,18] [,19] [,20] [,21] [,22] [,23] [,24] [,25] [,26] [,27]
aa 15.0000 16.0000 17.0000 18.0000 19.0000 20.0000 21.0000 22.0000 23.0000 24.0000 25.0000 26.000 27.000
p30 0.2488 0.2433 0.2538 0.2554 0.2668 0.2729 0.2835 0.2995 0.3255 0.3463 0.3886 0.452 0.531
[,28] [,29]
aa 28.0000 29.0000
p30 0.6285 0.7436
0966132人目の素数さん
垢版 |
2019/12/27(金) 20:43:28.09ID:KwJt/dWM
56が同部屋になるのを条件付き確率で考えた場合
5が引く時点で同部屋のくじしか残っていないということ
AAAB 3/6*2/5*1/4*3/3
AABA 3/6*2/5*3/4*1/3
ABAA 3/6*3/4*2/4*1/3
BAAA 3/6*3/5*2/4*1/3
ABBB 3/6*3/5*2/4*1/3
BABB 3/6*3/5*2/4*1/3
BBAB 3/6*2/5*3/4*1/3
BBBA 3/6*2/5*1/4*3/3
8*3!*3/6*5*4*3=2/5
0967132人目の素数さん
垢版 |
2019/12/27(金) 20:45:12.37ID:KwJt/dWM
>>965
ああ分かったルーレットなのか
>>957以下は撤回
0968132人目の素数さん
垢版 |
2019/12/27(金) 20:58:26.95ID:KwJt/dWM
ABが復元抽出の場合になるので
12が同室になるのは
AA**** 1/2*1/2=1/4
BB**** 1/2*1/2=1/4
1/4+1/4=1/2
56が同室になるのは満室が出ると以下必然となるため
AAAB 1/2*1/2*1/2*1/1=1/8
AABA 1/2*1/2*1/2*1/2=1/16
ABAA 1/2*1/2*1/2*1/2=1/16
BAAA 1/2*1/2*1/2*1/2=1/16
ABBB 1/2*1/2*1/2*1/2=1/16
BABB 1/2*1/2*1/2*1/2=1/16
BBAB 1/2*1/2*1/2*1/2=1/16
BBBA 1/2*1/2*1/2*1/1=1/8
1/8+6*1/16+1/8=10/16=5/8
ということか
0969イナ ◆/7jUdUKiSM
垢版 |
2019/12/27(金) 21:00:00.64ID:zPJPlDKd
>>950
>>617正方形45°回転の菱形クロス回転だから、共通部分の立体は円錐を稜線と平行な鉛直に対し45°に切りこんだ立体4個分だから、
円錐の体積Vを斜め45°の稜線で切る問題の記憶から、
V=V1+V2として、
V1/V2=(3π-4)/(3π+4)
V1=(3π-4)r^2h/18
V2=(3π+4)r^2h/18
V2-V1=2r^2h/9
r=h=√2/2
求める体積は、
4(V2-V1)=8r^2h/9
=4√2/9(<1)
あってるかも。
0970イナ ◆/7jUdUKiSM
垢版 |
2019/12/27(金) 21:10:01.84ID:zPJPlDKd
>>969訂正。
>>617
求める体積は、
4(V2-V1)=8r^2h/9
=2√2/9
共通部分はだいぶ小さいのかな?
0971132人目の素数さん
垢版 |
2019/12/27(金) 21:11:34.56ID:KwJt/dWM
A×n+B×nで同様にした場合
12が同室となる場合は1/2
last12が同室となる場合は
満室がどの時点で出るかで分類して
2{(1/2)^n+(n-1,1)(1/2)^(n+1)+(n-1,2)(1/2)^(n+2)+…+(n-1,n-2)(1/2)^(2n-2)}
かな
0972132人目の素数さん
垢版 |
2019/12/27(金) 21:19:07.54ID:KwJt/dWM
くじ引きの場合は最初の2名と最後の2名で同確率となるのは当たり前なので
一方の部屋により集まりやすいルーレット式の場合は
最後の2名が同室になる確率が1/2より大きくなるのは当然か
0973132人目の素数さん
垢版 |
2019/12/27(金) 22:02:08.66ID:bnpG+BjS
>>972
6人を2人ずつ3組分ける場合を計算してみると最後の2名が同室になる確率は1/3より小さくならない?
0974132人目の素数さん
垢版 |
2019/12/27(金) 22:20:50.24ID:FqqlMh9P
9人を定員3人の3部屋にわける場合を考える
部屋割りのやり方を列挙すると

[,1] [,2] [,3] [,4] [,5] [,6] [,7] [,8] [,9]
[1,] 1 1 1 2 2 2 3 3 3
[2,] 1 1 1 2 2 3 2 3 3
[3,] 1 1 1 2 2 3 3 2 3
[4,] 1 1 1 2 2 3 3 3 2
[5,] 1 1 1 2 3 2 2 3 3
[6,] 1 1 1 2 3 2 3 2 3
で始まり
[,1] [,2] [,3] [,4] [,5] [,6] [,7] [,8] [,9]
[1675,] 3 3 3 2 1 2 1 2 1
[1676,] 3 3 3 2 1 2 2 1 1
[1677,] 3 3 3 2 2 1 1 1 2
[1678,] 3 3 3 2 2 1 1 2 1
[1679,] 3 3 3 2 2 1 2 1 1
[1680,] 3 3 3 2 2 2 1 1 1
で終わる 1680通り

このうち、8番と9番が同じものは
[,1] [,2] [,3] [,4] [,5] [,6] [,7] [,8] [,9]
[1,] 1 1 1 2 2 2 3 3 3
[2,] 1 1 1 2 2 3 2 3 3
[3,] 1 1 1 2 3 2 2 3 3
[4,] 1 1 1 2 3 3 3 2 2
[5,] 1 1 1 3 2 2 2 3 3
[6,] 1 1 1 3 2 3 3 2 2
で始まり
[,1] [,2] [,3] [,4] [,5] [,6] [,7] [,8] [,9]
[415,] 3 3 3 1 2 1 1 2 2
[416,] 3 3 3 1 2 2 2 1 1
[417,] 3 3 3 2 1 1 1 2 2
[418,] 3 3 3 2 1 2 2 1 1
[419,] 3 3 3 2 2 1 2 1 1
[420,] 3 3 3 2 2 2 1 1 1
で終わる420通り

> 420/1680 = 0.25は直感に反する
1680通りが同様に確からしいという前提が間違いなのだと思う。
0975132人目の素数さん
垢版 |
2019/12/27(金) 22:41:07.89ID:FqqlMh9P
>>973

6人を定員3人の3部屋にわける場合を考える
部屋割りのやり方を列挙すると

[,1] [,2] [,3] [,4] [,5] [,6]
[1,] 1 1 2 2 3 3
[2,] 1 1 2 3 2 3
[3,] 1 1 2 3 3 2
[4,] 1 1 3 2 2 3
[5,] 1 1 3 2 3 2
[6,] 1 1 3 3 2 2
で始まり
[,1] [,2] [,3] [,4] [,5] [,6]
[85,] 3 3 1 1 2 2
[86,] 3 3 1 2 1 2
[87,] 3 3 1 2 2 1
[88,] 3 3 2 1 1 2
[89,] 3 3 2 1 2 1
[90,] 3 3 2 2 1 1
で終わる 90通り

このうち、5番と6番が同じものは
[,1] [,2] [,3] [,4] [,5] [,6]
[1,] 1 1 2 2 3 3
[2,] 1 1 3 3 2 2
[3,] 1 2 1 2 3 3
[4,] 1 2 2 1 3 3
[5,] 1 3 1 3 2 2
[6,] 1 3 3 1 2 2
[7,] 2 1 1 2 3 3
[8,] 2 1 2 1 3 3
[9,] 2 2 1 1 3 3
[10,] 2 2 3 3 1 1
[11,] 2 3 2 3 1 1
[12,] 2 3 3 2 1 1
[13,] 3 1 1 3 2 2
[14,] 3 1 3 1 2 2
[15,] 3 2 2 3 1 1
[16,] 3 2 3 2 1 1
[17,] 3 3 1 1 2 2
[18,] 3 3 2 2 1 1
の18通り
> 18/90 = 0.2は直感に反する
90通りが同様に確からしいという前提が間違いなのではないかと思う。
0976132人目の素数さん
垢版 |
2019/12/27(金) 23:06:27.27ID:FqqlMh9P
sim <- function(n=6,a=5,b=6){ # n人を定員3人の3部屋にわける場合にa, bが同室かT/Fを返す
r=numeric(n) # 1〜6人の部屋番号(1〜3)の配列
for(i in 1:n){
j=which(c(sum(r==1)<n/3 , sum(r==2)<n/3 , sum(r==3)<n/3 )) # 定員に達していない部屋から
r[i]=as.numeric(sample(as.character(j),1)) # ランダムに割り当てる
}
A=c(which(r==1)) # room 1に割当てられた人の順位番号
B=c(which(r==2))
C=c(which(r==3))
room=rbind(A,B,C)
ans=FALSE
for(i in 1:3){# 3部屋のどれかに同室ならば TRUEを返して終了
if(any(room[i,]==a) & any(room[i,]==b)){# if((a %in% room[i,]) & (b %in% room[i,]))と同意
ans=TRUE
break
}
}
ans
}
mean(replicate(1e5,sim(n=6,a=5,b=6))) # 10万回のシミュレーションをして頻度を求める

> mean(replicate(1e5,sim(n=6,a=5,b=6)))
[1] 0.39076
> mean(replicate(1e5,sim(n=6,a=5,b=6)))
[1] 0.3874

とシミュレーションでは1/3より大きい。
自分の直感はシミュレーションを支持。
0977132人目の素数さん
垢版 |
2019/12/27(金) 23:13:50.09ID:KwJt/dWM
>>973
12が同室1/3
56が同室は
AABB 1/3*1/3*1/2*1/2=1/36
ABAB 1/3*1/3*1/3*1/2=1/54
ABBA 1/3*1/3*1/3*1/2=1/54
BAAB 1/3*1/3*1/3*1/2=1/54
BABA 1/3*1/3*1/3*1/2=1/54
BBAA 1/3*1/3*1/2*1/2=1/36
3*(2*1/36+4*1/54)=(18+24)/108=42/108=7/18>1/3
0978イナ ◆/7jUdUKiSM
垢版 |
2019/12/27(金) 23:25:58.22ID:zPJPlDKd
>>970
>>914題意の図を内角が左上A72°左下B96°右下C78°右上D84°となるよう4頂点を決め、ABの中点をE、ADの延長線とBCの延長線の交点をF、ACとBDの交点をGとし、AE=BE=1、BG=xとすると、
ADは一辺ABの正五角形の対角線だから1+√5
AD=BD=BC=1+√5
Aを起点にメネラウスの定理より、――@
Bを起点にメネラウスの定理より、――A
Fを起点にメネラウスの定理より、――B
@ABより、x=2
△ABGはAB=GBの二等辺三角形で∠BAG=∠BGA
84°-?=?+24°
2?=84°-24°=60°
∴?=30°
たぶん。
0980132人目の素数さん
垢版 |
2019/12/27(金) 23:40:29.09ID:KwJt/dWM
A×n+B×n+C×nで同様にした場合
12が同室となるのは1/3
last12が同室となるのは
1部屋目の満室と2部屋目の満室がどこで出るかで分類して・・・・
面倒だなあ
0981132人目の素数さん
垢版 |
2019/12/28(土) 00:25:20.97ID:T6yZsGIV
f(f(x))-x=0
を満たす、実数xについての関数f(x)について以下の問いに答えよ。

(1)f(x)が1次関数ならばf(x)=xであることを示せ。

(2)f(x)をすべて決定せよ。
0982132人目の素数さん
垢版 |
2019/12/28(土) 00:31:05.18ID:PhuGZyqF
これはひどい
0983132人目の素数さん
垢版 |
2019/12/28(土) 00:49:19.36ID:djVdRhtS
f(x) = a*x + b
f(f(x)) = a*(a*x + b) + b = a^2 * x + a*b + b = x

a^2 = 1
a*b + b = 0

a = 1 ⇒ b = 0
a = -1 ⇒ b は任意の実数

f(x) が1次関数 ⇒ f(x) = x or f(x) = -x + c
0985132人目の素数さん
垢版 |
2019/12/28(土) 01:07:58.90ID:p2O6LJwx
>>976,977
とっくに既出ですよ。 >>944
0986132人目の素数さん
垢版 |
2019/12/28(土) 01:09:12.40ID:p2O6LJwx
>>976,977
アンカー間違えた。つ>>948
0987イナ ◆/7jUdUKiSM
垢版 |
2019/12/28(土) 02:53:00.72ID:GFHwIJTI
>>978ABとCDが平行じゃないし3つの等しい線分もジグザグ。すなわち遠方になってもかならず頂点Fを結ぶ。メネラウスとチェバだろ。比が出てx=2で二等辺三角形の底角が等しいから、違うのか?
 ̄ ̄]/\______∩∩_
____/\/ .,~、 (___))|
 ̄ ̄\/ 彡-_-ミっ / |
 ̄ ̄|\_U,~⌒ヽ、| |
□ | ‖ ̄ ̄U~~U | / )
____| ‖ □ ‖ |/ /|
_____`‖______‖ノ / |
 ̄ ̄ ̄ ̄ ̄ ̄ ̄ ̄ ̄‖ |
□  □  □  ‖ /
__________________‖//
 ̄ ̄ ̄ ̄ ̄ ̄ ̄ ̄ ̄ ̄_/__/__/__/__/__/__/__/__/__/__/__/__/__/__/__/__/__/__/__/__/__/__/__/__/__/__/__/__/__/__/__/__/__/__/__/__/__/__/__/__/__
0988132人目の素数さん
垢版 |
2019/12/28(土) 04:17:08.40ID:Q7tXw4P7
(2) たとえば、 (1)の解を F(x) として
 f(x) = g^(-1){F(g(x))},
これがすべてぢゃなかろうが・・・・

ところで次スレはまだ?
0990132人目の素数さん
垢版 |
2019/12/28(土) 22:15:37.88ID:p6r3EJNl
射影幾何学はユークリッド幾何学と非ユークリッド幾何学に共通の性質を抽出した幾何学である

1 その通り
2 半分正しいけど半分間違っている
3 完全に間違い

どれですか?
0991132人目の素数さん
垢版 |
2019/12/29(日) 00:42:23.56ID:8OjZTw/B
ユークリッドと非ユークリッドに共通した性質ってそれ非ユークリッドそのものじゃね
0994イナ ◆/7jUdUKiSM
垢版 |
2019/12/29(日) 15:33:56.47ID:YfxvMMZF
 ̄ ̄]/\前>>987訂正。_
____/\/,,、、     )
 ̄ ̄\/彡-_-ミ    /
 ̄ ̄|\_U,~⌒ヽ___/|
□ | ‖ ̄~U~U~ ̄‖ |
____| ‖ □ □ ‖ |/
_____`‖_________‖/ベン図が描けんの? 共有部分がないんやないが?
0995132人目の素数さん
垢版 |
2019/12/29(日) 20:00:35.03ID:ktrDgrgt
>>917

 -1 + e^(24゚i) - e^(-12゚i) = - e^(-30゚i){e^(30゚i) - e^(54゚i) + e^(18゚i)}

 Im{e^(30゚i) - e^(54゚i) + e^(18゚i)} = 1/2 - sin(54゚) + sin(18゚)
 = {1 + 2sin(234゚) + 2sin(18゚)}/2
 = {sin(90゚) + sin(306゚) + sin(234゚) + sin(162゚) + sin(18゚)}/2
 = 0,   (←正5角形)
より
 e^(30゚i) - e^(54゚i) + e^(18゚i) = AC (=実数)

 -1 + e^(24゚i) - e^(-12゚i) = - AC・e^(-30゚i)
0996132人目の素数さん
垢版 |
2019/12/30(月) 01:02:27.01ID:4FN+HhkB
↑ [次スレ.051]

ヴェクトルによる方法
 Z字に沿って A,B,C,D とおき、主軸の向きをxとする。
各辺となす角は
↑AB: x-12゚
↑BC: x+204゚
↑CD: x
∴ ↑AD の主軸垂直成分は
 sin(x-12゚) + sin(x+204゚) + sin(x),
あるいは
 sin(192゚-x) + sin(336゚-x) + sin(x),
平均して
 {sin(x-12゚) + 2sin(x) + sin(192゚-x) + sin(x+204゚) + sin(336゚-x)}/2,
x=30゚ とおけば左辺は
 {sin(18゚) + sin(90゚) + sin(162゚) sin(234゚) + sin(306゚)}/2 = 0  ←正5角形
となり↑AD に平行。
[次スレ.066]
0997132人目の素数さん
垢版 |
2019/12/30(月) 05:01:13.95ID:wh5s35zC
aを実数の定数とする。
実数xについての関数
f(x)=x^3-a[x]-1
について、以下の問いに答えよ。
ただし[x]はxを超えない最大の整数を表す。

(1)a=2のとき、方程式f(x)=0の解をすべて求めよ。

(2)以下の(i)(ii)の条件を満たすようなaの範囲を答えよ。
(i)方程式f(x)=0が重複を込めて3つの実数解を持つ。
(ii)方程式f(x)=0が重複を込めて2つの虚数解を持つ。
10011001
垢版 |
Over 1000Thread
このスレッドは1000を超えました。
新しいスレッドを立ててください。
life time: 113日 15時間 15分 35秒
10021002
垢版 |
Over 1000Thread
5ちゃんねるの運営はプレミアム会員の皆さまに支えられています。
運営にご協力お願いいたします。


───────────────────
《プレミアム会員の主な特典》
★ 5ちゃんねる専用ブラウザからの広告除去
★ 5ちゃんねるの過去ログを取得
★ 書き込み規制の緩和
───────────────────

会員登録には個人情報は一切必要ありません。
月300円から匿名でご購入いただけます。

▼ プレミアム会員登録はこちら ▼
https://premium.5ch.net/

▼ 浪人ログインはこちら ▼
https://login.5ch.net/login.php
レス数が1000を超えています。これ以上書き込みはできません。

ニューススポーツなんでも実況